You are on page 1of 346

Nikhil Agarwal

(FIRST EDITION)
e
ki
oo

42 SOLVED
pC
ar
Sh

PREVIOUS YEAR
PAPERS

Nikhil Agarwal
Authors: Kanishka Halder ,
Aniruddha Chattopadhyay,
Aadi Swadipto Mondal ,
Parth Sarathi Roy

Owner & Compiler


Nikhil Agarwal

e
ki
oo
pC
ar

Copyright © 2018 by Sharp Cookie. All Rights reserved.


Sh

Copyright © 2018, exclusive rights by Sharp Cookie, India for manufacture,


circulation of the book beyond IIT Kharagpur premises is strongly advised against.

No part of this publication may be reproduced, stored in a retrieval system or


transmitted in any form without the consent of the authors’ or the authority
in charge of Sharp Cookie. Any such wrong doer without prior written permission from
the publishers might face prosecution.

In case of any doubts or query visit our website: www.sharpcookie.ml


M.R.P. : ₹400 (first edition)
*Since the authors are themselves students in particular and human beings in general
It should clearly resonate that the book is not error proof. In case of any doubts or ambiguities
get in touch with your professors.
*Some questions were not available and has been made from recollection only.
We deeply apologize for any other inconvenience caused.
Acknowledgement

This book is a cumulative effort of lot of individuals. Practically it is not possible to


include each of the concerned person’s names but we extend our heartiest gratitude to
each individual involved in the process. To all our beloved professors whose constant
guidance and motivation facilitated the making of the book and to our lovely parents for
being so approving, THANK YOU.
We would like to grab this opportunity to extend our gratitude to some of the
noteworthy people --- Honourable Director, IIT Kharagpur Professor Partha Pratim
Chakraborty; Honourable Dean UGS, IIT Kharagpur Professor Sudhir Kumar Barai;
Professor Jitendra Kumar, Professor Animesh Mukherjee, Professor Atul Jain, Professor
ie
Pranab K. Dutta, Professor Ratna Dutta, Professor Sugata Pratik Khastgir and Professor
ok

Swagata Dasgupta.
o
pC
ar
Sh

Special Thanks:
Professor Koeli Ghoshal
( Department of Mathematics )
Preface
Students have been solving previous year question papers to prepare for exam throughout their school
life. Solving previous year questions doesn’t only help in revision but also boosts confidence. So why stop
after school? This was the question in the authors’ minds while starting this project. But unsupervised
practice only embeds one’s errors. So it is necessary to have a solution guide for checking the answers
and also clear any misconceptions. Many freshers in IIT Kharagpur have felt the need of such a question
bank-solution guide. This book was launched to fill this void.

This book is a compilation of 42 question papers and the solutions of previous semesters of IIT Kharag-
pur. It covers question papers of Physics, Maths, Electrical Technology, Chemistry, Programming and Data
Structures and Mechanics of the 1st year course structure.
e
ki
This book is meant for every 1st year student in IIT Kharagpur looking for well organized solutions
oo

for previous year question papers. Solving papers enhances a student’s perspective about the pattern of the
upcoming exams.Different exams give different weightage to different topics. This can be tackled by solving
pC

previous year question papers. This book is also meant for last-minute revision as each paper consists of
ar

questions and their solutions from the important topics.


Sh
The dassi fundae ...

 Always keep in mind that in absolute marking, one who gets 90 and one who
gets 100 are same and equivalently in relative marking. To score a high CGPA,
you have to score more no of EX rather than getting more no of full marks. So
do not focus unnecessarily.

 This is not your home where you will get tutors from your coaching centre, your
home-tutors and several others. Here professors are the only hope. So never
miss classes as there will be none to help you. Be physically and mentally
present at the class.
ie
ok

 Leaving some exceptions, professors are always quite friendly and


o

approachable. Sometimes they are more like friends than teachers. They are
pC

always to help you. So never miss a chance to exploit their favour.


ar

 This is not JEE where you have to solve hundreds of problems. Solve your
Sh

tutorial sheets first to get an idea of the chapters and then jump for this book.
There is no need of solving several problem books.

MAKE YOU CONCEPTS CLEAR


AND
BEAT THE EXAMS!!!
CONTENTS
1) MATHS I
• MIDSEM
➢ 2017-18 1
➢ 2016 7
➢ 2015 12
• ENDSEM
➢ 2017 16
➢ 2015-16 e 21
ki
oo

➢ 2011-12 30
pC

2) MATHS II
ar
Sh

• MIDSEM
➢ 2018 39
➢ 2017 45
➢ 2016 51
• ENDSEM
➢ 2018 55
➢ 2017 63
➢ 2016 73
3) PHYSICS
• MIDSEM
➢ 2017-18 Spring 79
➢ 2017-18 Autumn 83
➢ 2016-17 Spring 86
• ENDSEM
➢ 2017-18 Autumn 88
➢ 2017-18 Spring 96
➢ 2016-17 Spring 103
4) ELECTRICAL TECHNOLOGY
• MIDSEM
e
ki
oo

➢ 2017-18 Autumn 108


pC

➢ 2016-17 Spring 118


ar
Sh

➢ 2015-16 Autumn 129


• ENDSEM
➢ 2017-18 Autumn 137
➢ 2015-16 Spring 148
➢ 2015-16 Autumn 154
5) PROGRAMMING AND DATA STRUCTURE
• MIDSEM
➢ 2017-18 Spring 162
➢ 2017-18 Autumn 175
➢ 2016-17 Spring 183
• ENDSEM
➢ 2017-18 Spring 193
➢ 2017-18 Autumn 203
➢ 2016-17 Autumn 212
6) CHEMISTRY
• MIDSEM
➢ 2017-18 Spring 225
➢ 2016-17 Spring 233
➢ 2016-17 Autumn
e 240
ki
oo

• ENDSEM
pC

➢ 2016-17 Spring 247


ar
Sh

➢ 2015 Autumn 263


➢ 2012 Autumn 280
7) MECHANICS
• MIDSEM
➢ 2017-18 Spring 291
➢ 2017-18 Autumn 297
➢ 2016-17 Autumn 305
• ENDSEM
➢ 2017-18 Spring 311
➢ 2017-18 Autumn 318
➢ 2016-17 Spring 324

*KEY CONCEPT AND FORMULAE

e
ki
oo
pC
ar
Sh
MATHS I
MID SEMESTER 2017-18

Ans 1a): Let f (x) = (1 − x) − e−x


f 0 (x) = −1 + e−x < 0 e
ki
⇒ f is a decreasing function for x > 0
oo

Also f (0) = 0
∴ f (x) < f (0) ⇒ (1 − x) < e−x . . . . . . (1)
pC

x2
Let g(x) = e−x − (1 − x + )
2
ar

g 0 (x) = −e−x − (−1 + x) < 0 [from (1)]


Sh

⇒ g decreasing for x > 0


Also g(0) = 0
∴ g(x) < g(0) = 0
x2
⇒ e−x < 1 − x +
2
Hence,
x2
(1 − x) < e−x < (1 − x + ) for x > 0
2
Ans 1b): By MVT we have,
f (b) − f (a)
= f 0 (c1 ) . . . . . (1) for c1 ∈ (a, b)
b−a
Let g(x) = x2
Now by applying Cauchy’s MVT on f (x) and g(x) we say that ∃ c2 ∈ (a, b) such that,
f (b) − f (a) f 0 (c2 )
= 0
g(b) − g(a) g (c2 )
f (b) − f (a) f 0 (c2 )
⇒ =
b2 − a2 2c2
f 0 (c1 ) f 0 (c2 )
⇒ = [by applying (1)]
b+a 2c2
Ans 1c): f (x) = (ax + 5)ex
f 0 (x) = (ax + 5 + a)ex
f 0 (x) = (ax + 5 + 2a)ex
..
.

1
f n (x) = (ax + 5 + na)ex
x2
∴ f (x) = f (0) + xf 0 (0) + f 00 (0) + . . .
2!
100 80
Co-efficient of x =
100!
5 + 100a 80
⇒ =
100! 100!
80 − 5 75
⇒a= = = 0.75
100 100

 2
 sin (x − y) , f (x, y) 6= (0, 0)
Ans 2a): f (x, y) = |x| + |y| e
ki
0 , f (x, y) = (0, 0)

oo

|f (x, y) − f (0, 0)|


sin2 (x − y)
pC


= − 0

|x| + |y|
ar

(x − y)2


|x| + |y|
Sh

|x − y||x − y|

|x| + |y|
≤ |x| + |y|
p
< 2 x2 + y 2
< 2δ < 

∴δ<
2
Hence f (x, y) is continuous at (0,0).

Ans 2b): f (x, y) = x2 − 5xy + 2y 2


f (1, 1) = −2
fx (1, 1) = −3
fy (1, 1) = −1
fxx (1, 1) = 2
fxy (1, 1) = −5
fyy (1, 1) = 4

F (x, y) = −2 − 3(x − 1) − (y − 1) + (x − 1)2 − 5(x − 1)(y − 1) + 2(y − 1)2


2 2
Ans 2c): f (x, y) = ex +y
u(x, y) = ln f = x2 + y 2
u(x, y) is homogeneous function of degree 2.
By using Euler’s theorem

2
∂u ∂u
x +y = 2u
∂x ∂y
1 ∂f 1 ∂f
x +y = 2u
f ∂x f ∂y
∂f ∂f
x +y = 2f ln(f )
∂x ∂y

e
ki
oo
pC
ar
Sh

and

3
Sh
ar
pC
oo
ki
e

4
 
x+e−x x
Ans 4a): lim e −e (∞ − ∞ form)
x→+∞
e−x
e −1
= lim ( 00 form)
x→+∞ e−x
−x
e (−e−x )
e
= lim
x→+∞ (−e−x )
e−x e
ki
= lim e = e0 = 1
x→+∞
oo

∂f 2 2
= 2x − x(x2 − y 2 ) e(−x −y )/2
 
Ans 4b): and
pC

∂x
∂f 2 2
= − 2y − y(x2 − y 2 ) e(−x −y )/2
 
∂y
ar

So critical points are solutions of the simultaneous equations.


Sh

2x − x(x2 − y 2 ) = 0 and −2y − y(x2 − y 2 ) = 0


i.e. x[2 − (x2 − y 2 )] = 0 and √ y[−2 − (x2 −√y 2 )] = 0
They have solutions, (0,0) , (± 2,0) , (0,± 2)
2 2
Here, fxx = (2 − 5x2 + x2 (x2 − y 2 ) + y 2 )e(−x −y )/2
2 2
fyy = (5y 2 − 2 + y 2 (x2 − y 2 ) − x2 )e(−x −y )/2
2 2
fxy = xy(x2 − y 2 )e(x −y )/2
Taking D = [fxx (xo , yo )] × [fyy (xo , yo )] − [fxy (xo , yo )]2
We calculate,
Point fxx fxy fyy D Type
(i) (0,
√ 0) 2 0 −2 −4 Saddle
(ii) (± 2, √0) −4/e 0 −4/e 16/e2 Max
(i) (0, ± 2) 4/e 0 4/e 16/e2 Min
Ans 4c): Case I: Here fx = 6x2 , fy = 4y 3 . So (0,0) is a candidate for absolut maximum or minimum
inside {(x, y)|x2 + y 2 < 1}
Case II: Let us find maxm or minm value of f (x, y) such that g(x, y) = (x2 + y 2 ) = 1
3 4 2 2
Consider L(x, y, λ) = 2x + y + λ(x + y − 1)
Here Lx = 6x2 + 2λx , Ly = 4y 3 + 2λy , Lλ = x2 + y 2 − 1
Lx = 0 ⇒ x(3x + λ) = 0 , Ly = 0 ⇒ y(2y 2 + λ) = 0 , Lλ = 0 ⇒ x2 + y 2 − 1 = 0
If x = 0 , y = ±1 , λ = −2
If y = 0 , x = ±1 , λ = ±3
3
If x 6= 0 , y 6= 0 then 3x = 2y 2 ⇒ y 2 = x ,
2

5
3 1
Now, x2 + y 2 = 1 ⇒ x2 + x = 1 ⇒ x = −2 or x =
2 2
x = −2 is discarded√
1 3
∴x= andy = ±
2 2 √
1 3 13
Here, f (0, ±1) = 1 , f ( , ± )= , f (−1, 0) = −2 , f (1, 0) = 2
2 2 16
∴ Absolute maximum at (1,0) and value is 2
and absolute minimum is at (-1,0) and value is -2.

e
ki
oo
pC
ar
Sh

6
MATHS I
MID-SPRING SEMESTER 2016

e
ki
oo
pC
ar
Sh

xa y b
Ans 1a): f (x, y) =
x2 + xy + y 2
We need to check for continuity at (x, y) ⇒ (0, 0)
Hence let x = r cos θ y = r sin θ where r → 0
(cos θ) a (sin θ)b
lim ra+b−2
r→0 1 +cos θ sin θ 
 (cos θ)a (sin θ)b 
⇒ lim ra+b−2 
r→0 sin 2θ 
1+
2
So if it must be continuous then conditions are a + b ≥ 2 since ra+b−2 will make the function discontinuous.
sin 2θ 1
Also the [. . .] part must be bounded. Since 1 + ≥ it is bounded when a ≥ 0 , b ≥ 0
2 2
Any other condition causes discontinuity.
xy
Ans 1b): f (x, y) = p
x2 + y 2
df
f (h, 0) − f (0, 0)
So = lim =0
dx x=0 h→0 h

(x + h)y (x)y
p −p
df (x + h)2 + y 2 x2 + y 2
= lim
dx h→0 h
y3 df x3
= p Similarly, = p
( x2 + y 2 )3 dy ( x2 + y 2 )3

7
f (15) − f (6)
Ans 2a): = f 0 (c)
9
where c lies between (6,15) according to LMVT.
⇒ f (15) − f (6) ≤ 9 × (10) [since f 0 (c) < 10]
⇒ f (15) ≤ 92
92 is the max value.

f (3) − f (3)
Ans 2b): = f 0 (c) where c lies between (-3,3) according to LMVT.
6
f 0 (c) = 1
f (3) − f (0)
≤ 1 ⇒ 3 − f (0) ≤ 3 . . . . (1)
3
f (0) − f (−3)
≤ 1 ⇒ f (0) − 3 ≤ 3 . . . (2)
3
Hence ⇒ f (0) = 0 (from (1) and (2)) e
ki
oo
pC
ar
Sh

!x
1 + ln(1 + a)
Ans 3a): lim
x→∞ x
ln(1 + a2 )
x.
=e x
=1+a 2

Now, 1 + a2 = 2a cos2 t
⇒ 1 + a2 ≥ 2a (by AM-GM since a>0)
⇒ cos2 t = 1
⇒ t = nπ ± 0

Ans 3b): (x − 1)ex = f (x)


According to Taylor’s Theorem
(x − a)2 00
f (x) ⇒ f (a) + (x − a)f 0 (a) + f (a) + . . .
2!
3
(x − a) 000
So compute the terms till f (a) to get the series.
3!
8
 2 2

(xy) x − y , if (x, y) 6= (0, 0)

Ans 4a): f (x, y) = x2 + y 2


0 if (x, y) = (0, 0)

According to definition :
4
lim − dz4f should exist
4f →0 z
4
= lim . 4 y(4x2 − 4y 2 )(4x2 + 4y 2 )3/2
(4x,4y)→(0,0) x e
ki
⇒ Let 4y = m 4 x
oo

m(1 − m2 )
= −→ depends on m
(1 + m2 )3/2
pC

⇒ not differentiable.
ar

Ans 4b): exact = f (1.01, 1.01) − f (1, 1) = 4.02 × 10−2 Total = f x.dx + f y.dy = 0.04
Sh

4.02 − 4 0.02
error = × 100 = × 100 = 0.5%
4.02 4.02

 2 2
 (x y + xy ). sin(x − y) (x, y) 6= (0, 0, )
Ans 5a): f (x, y) = x2 + y 2
0

∂f
f (h, y) − f (0, y)
= lim
∂x (0,y) h→0 h

9
(h2 y + hy 2 ) sin(h − y)
= lim
h→0 h2 + y 2
=0
∂f
=0
∂y 0,0

Similarly solve the other.
Ans 5b): x2 y 2 z 3 + zx sin y = 5
x2 y 2 z 3 + zx sin y − 5 = 0
∂z ∂z
We have to find and
∂x ∂y
2 3 2 2 2 ∂z ∂z
⇒ 2xy z + 3x y x + x sin y + z sin y = 0
∂x ∂x
∂z 2xy 2 z 3 + z sin y
⇒ =− 2 2 2
∂x 3x y z + x sin y
Similarly solve the other part.

e
ki
oo
pC
ar

Ans 6a): f (x, y) = sin(x, y)


(x − 1)2 (y − π/2)2
Sh

f (x, y) = sin(π/2) + (x − 1) sin0 (y) + (y − π/2) sin0 (xπ/2) + sin00 (y) + sin00 (xπ/2)
2! 2

+2(x − 1)(y − π/2) (sin0 (xπ/2))
∂x
Ans 6b): u(x, y) = xφ(y/x) + ψ(y/x)
∂u −1 −1
= φ(y/x) + φ0 (y/x)( )y + ψ 0 (y/x)( 2 )y
∂x x x
∂u 0 1 0 1
= xφ (y/x) + ψ (y/x)
∂y x x
∂u y
y. = y.φ0 (y/x) + ψ 0 (y/x)
∂y x
∂u −1
x. = xφ(y/x) − φ0 (y/x).y + ψ 0 (y/x)( )
∂x x
∂u ∂u 0
⇒ y. +x = xφ (y/x)
∂y ∂x
f(x) = x
By using Euler theorem,
φ and ψ are homogeneous functions of order 0. Hence,
∂φ ∂φ ∂ψ ∂ψ
x +y =0 x. + y. =0
∂x ∂y ∂x ∂y
⇒ So the only part that gets differentiated is x part .
So lets take φ(y/x) = k
∂k ∂k
f = xk [differentiation of ’k’ in the form x +y =0]
∂x ∂y

10
∂f ∂f
Hence, x +y
∂x ∂y 
∂k ∂k
= k.x + x +y
∂x ∂y
= kx
⇒ f (x) = x

e
ki
oo
pC

Ans 7a): f = x2 (2 − y) − y 3 + 3y 2 + 3y
ar

∂f
= 0 ⇒ 2x(2 − y) = 0
Sh

∂x
⇒ x = 0 or y = 2
∂f
= 0 ⇒ x2 (−1) − 3y 2 + 6y + 9 = 0
∂y
∂f
for x = 0 ⇒ ⇒ −y 2 + 2y + 3 = 0
∂y
−y 2 + 3y − y + 3 = 0
(y + 1)(y − 3) = 0
⇒y=3;y=1
(0,3) , (0,1)
for y = 2 ⇒ −18 + 12 + 9 − x2 = 0
⇒ −x2 +√ 3=0
⇒√ x = ± 3√
(2, 3) , (2,- 3)
Check the values for maximum and minimum.

Ans 7b): (x − 2)2 + (y − 1)2 ≤ 20


(x − 2)2 + (y − 1)1 − 20 + λ(x2 + y 2 ) = z
dz dz
find = 0 and =0
dx dy
and then check for which pair it is maximum.

11
MATHS I
MID SEMESTER 2015

sin 2x + a sin x
Ans 1a): lim
x→0 x3
Using Taylor’s series,
(2x)3 (ax)3
2x + + · · · + ax + + ...
lim 3! 3!
x→0 x3
rest of the terms will be zero.
⇒ 23 + a = 0
⇒ a = −8

Ans 1b): ex = sec x e


ki
ex − sec x = 0
oo

f (x) = e−x − cos x


if 2 roots exist then
pC

for f 0 (x) = 0 one root exists by Rolle’s Theorem.


⇒ −e−x + sin x = 0
ar

Ans 1c): Let f (x) = ex


Sh

ex − e0
≥ ec for c ∈ (0,∞)
x−0
ec is increasing function then
min e0 = 1
∴ ex − 1 > x [proved]

Question 2(a) is not in syllabus.


Ans 2b): To prove : ax ≥ xa for x ≥ a ≥ e
Tanking log of both sides
⇒ x ≥ a loga x
Lets take, f (x) = x − a loga x
a
∴ f 0 (x) = 1 −
x ln a
Now, x > a and ln a > 1 (as a > e)
a
Thus, <1
0
x ln a
∴ f (x) > 1 which means f (x) is an increasing function.
f (a) = 0
∴ f (x) ≥ 0 for any x ≥ a
∴ x − a loga x ≥ 0

12
∴ x ≥ a loga x
∴ ax ≥ xa [proved]

Question 3 is not in syllabus.

x4 − x2 y 6
Ans 4a):
(x2 + y 6 )2
Let x = my 3 e
ki
m4 y 1 2 − m2 y 1 2

oo

(y 6 + m2 y 6 )2
m4 − m2

pC

(1 + m2 )2
Value is dependent on m, thus functions is not continuous.
ar

4z − dz
Ans 4b): lim
Sh

p
(x,y)→(0,0) x2 + y 2

4x. 4 y − 4x.fx (0, 0) − 4y.fy (0, 0)
lim p
(4x,4y)→(0,0) 4x2 + 4y 2
f (h, 0) − f (0, 0)
fx (0, 0) = lim =0
h→0 h
fy (0, 0) = 0
4y = m 4 x r
m
⇒ lim
(4x,4y)→(0,0) 1 + m2
Hence not differentiable.

13
1
Ans 5a): f = x3 sin( )
y2
∂f 1
= 3x2 sin( 2 ) = fx
∂x y
∂f 1 −1
= 2x3 cos( 2 ) 3 = fy
∂y y y
4x2 + 4y 2 < 42
1
Now, 3x2 sin( 2 ) < 3x2
y
1
2
∴ 3x sin( 2 ) < 3x2 + 3y 2
y
1
∴ 3x2 sin( 2 ) < 342
y
Hence fx is continuous. [Proved by delta-epsilon method]
Ans 5b): x3 + z 2 + xeyz + z cos x = 0
⇒ differentiating with respect to x
dz dy dz
⇒ 3x2 + 3z 2 + eyz + xeyz .y + cos x − z sin x = 0
dx dx dx
dz 2 yz
3x + e − z sin x
=− 2
dx (0,0,0) 3z + xyeyz + cos x

e
ki
oo
pC
ar
Sh

Ans 6b): u = x2 f (y/x) − g(y/x)


∂2u ∂2u ∂2u
⇒ x2 . 2 + y 2 2 + 2xy = 0 (as n=0 )
∂x ∂y ∂x∂y
2
∂ f 2
∂ f 2
∂ f
⇒ x2 . 2 + y 2 2 + 2xy = 0 (as n=0 )
∂x ∂y ∂x∂y
∴ u = h(x, y) − g(x, y)4
So (n)(n-1) = (2)(2-1)
and α = 2 according to Euler theorem.

14
∂f 2 2
= 2x − x(x2 − y 2 ) e(−x −y )/2
 
Ans 7): and
∂x
∂f 2 2
= − 2y − y(x2 − y 2 ) e(−x −y )/2
 
∂y
So critical points are solutions of the simultaneous equations.
2x − x(x2 − y 2 ) = 0 and −2y − y(x2 − y 2 ) = 0
i.e. x[2 − (x2 − y 2 )] = 0 and √ y[−2 − (x2 −√y 2 )] = 0
They have solutions, (0,0) , (± 2,0) , (0,± 2)
2 2
Here, fxx = (2 − 5x2 + x2 (x2 − y 2 ) + y 2 )e(−x −y )/2
2 2
fyy = (5y 2 − 2 + y 2 (x2 − y 2 ) − x2 )e(−x −y )/2
2 2
fxy = xy(x2 − y 2 )e(x −y )/2
Taking D = [fxx (xo , yo )] × [fyy (xo , yo )] − [fxy (xo , yo )]2
We calculate,
Point fxx fxy fyy D Type
(i) (0,
√ 0) 2 0 −2 −4 Saddle
(ii) (± 2, 0) −4/e 0 −4/e 16/e 2 Max
√ 2
(i) (0, ± 2) 4/e 0 4/e 16/e Min
This question was repeated in 2017-18 Midsem.

e
ki
oo
pC
ar
Sh

15
MATHS I
END SEMESTER EXAMINATION 2017

1. (a) Solve the given differential equation


d2 y dy
x2 2 − 3x + 5y = 0
dx dx
(b) Solve and write the general solution of the given differential equation:
d3 y d2 y dy
3
− 2
+3 + 5 = ex cos x
dx dx dx

Ans 1a): x2 y 00 − 3xy 0 + 5y = 0


Let x = ez ⇒ z = log x
d
x2 y 00 = D1 (D1 − 1) where = D1
dz
xy 0 = D1

Thus the given Differential Equation gives,


(D1 2 − 4D1 + 5)y = 0
D1 = 2 ± i
General solution: e2z (c1 cos z + c2 sin z)
=x2 (c1 cos(log x) + c2 sin(log x)) e
ki
oo

Ans 1b): (D3 − D2 + 3D + 5)y = ex cos x


So, m3 − m2 + 3m + 5 = 0
pC

m = −1, (1 ± 2i)
⇒ Complementary solution: c1 e−x + ex {c2 cos 2x + c3 sin 2x}
ar

for particular integral,


Sh

1
P.I. = 3 2
[ex cos x]
D − D + 3D + 5
1
=ex cos x
(D1 + 1) − (D1 + 1)2 + 3(D1 + 1) + 5
3
1
=ex 3 cos x [replace all D2 with -1]
D + 2D2 + 4D + 8
1
=ex cos x
−D − 2 + 4D + 8
1
=ex cos x
3D + 6
3D − 6
=ex 2 − 36
cos x
x
9D
e
= {sin x + cos x}
15
General Solution: CF + PI
ex
= c1 e−x + ex {c2 cos 2x + c3 sin 2x} + {sin x + cos x}
15
2. (a) Solve the given equation,
(x2 + y 2 + 2α)dx + 2ydy = 0
(b) Solve:
dy
+ y cos x = y 3 sin 2x
dx

16
Ans 2a): (x2 + y 2 + 2x).dx + 2ydy = 0
M = x2 y 2 + 2x
N = 2y
∂M ∂N
⇒ = 2y =0
∂y ∂x
1 ∂M ∂N
⇒ − =1
N ∂y ∂x R
Integrating Factor = e 1.dx = ex
⇒ eZx (x2 + y 2 + 2x)dx + ex .2y.dy = 0
⇒ ex (x2 + y 2 + 2y).dx + 0 = c
⇒ ex (x2 + y 2 ) = c

dy
Ans 2b): + y cos x = y 3 sin 2x
dx
dy
⇒ y3. + y −2 cos x = sin 2x
dx
dy dv
Let y −2 = v ⇒ −2y −3 =
dx dx
dv
(−2 cos x)v = sin 2x
dx R
IF = e −2 cosZx.dx = e−2 sin x
v.e−2 sin x = e−2 sin x sin 2x.dx + c
Let sin x = Zt ⇒ cos x.dx = dt
ve−2 sin x = e−2t .2t.dt + c e
ki
1
oo

=−te2t − e−2t + c
2
1
pC

y −2 e−2 sin x = − sin x.e−2 sin x − e−2 sin x + c


2
ar

3. a)Test whether f (z) is differentiable at x = 0 where,


Sh

2)

Im(z
z 6= 0

f (z) = z
0 z=0
Are CR equations satisfied at z=0

(b) (i) Prove that u(x, y) = e−x(x sin y−y cos y)


(ii) Find v(x,y) such that f (z) = u + iv is analytic.
(iii) Find f (z) in terms of z.

 2xy

(x, y) 6= (0, 0)
Ans 3a): f(z) = x − iy
0 (0, 0)
 2xy(x + iy)

(x, y) 6= 0
= x2 + y 2
0 z=0

Differentiability at z=0,
f (h) − f (0)
lim
h→0 h
2xy
= lim 2 Let y = mx
h→0 x + y 2
2m2
= lim
(x,y)→(0,0) 1 + m2
Limit depends on m, hence it does not exist.
C.R equations :

17
(u(h, 0) − u(0, 0))
ux (0, 0) = lim =0
h→0 h
Similarly,
uy (0, 0) = ux (0, 0) = vy (0, 0) = 0 at z=0
C.R. equations are satisfied,
ux = vy = 0
uy = −vx = 0

Ans 3b(i): u(x, y) = e−x (x sin y − y cos y)


uxx = −2e−x sin y + e−x (x sin y − y cos y)
uyy = e−x (−x sin y + 2 sin y + y cos y)
uxx + uyy = 0
Hence u is harmonic.
Ans 3b(ii): dv = vx .dx + vy .dy
=−uy .dx + ux .dy
Z =e−x (x cos y − cos y + y sin y).dx + e−x (sin y − x sin y + y cos y).dy
v=− e−x (x cos y − cos y + y sin y).dx
=e−x (x cos y + y sin y) + c
Ans 3b(iii): f (z) = f (x + iy) = u(x, y) + iv(x, y)
=u(z, 0) + iv(z, 0)
=e−z (z sin 0 − 0) + ie−z (z cos 0 + 0) + ie
=ize−z + ie

(x − 3)2 y2e
ki
4. a) Let C be the arc of ellipse + = 1 lying on the quadrant oriented in counter
4 9
oo
Z
1
clockwise direction. Evaluate 4
dz
C z
pC

z 3 + 2z
(b) Let C be the circle |z| = 3 oriented in counter-clockwise direction. If g(w) = dz then
(z − w)3
ar

find g(2) and g(4i)


Sh

Ans 4a): Let L br the joining point z=1 to z=5


1 1 5
Z Z  
1 1 124
Then 4
.dz = − 4
.dz = 3
=−
c z L z 3 z 1 375
Ans 4b): Let f (z) = zZ3 + 2z which is analytic in the domain.
f (z)
f 00 (z) = 6z and g(z) = 3
.dz
c (z − 2)
2ni 00
= f (z)
2!
Scince z = 4i lies outside domain
g(4i) = 0

5. (a) Find the Laurent series in the annulus 1 < |z| < 4 for the following function about z = 0
z+2
f (z) = 2
z − 5z + 4
(b) Classify the singularities of the following function in the finite complex plane: f (z) =
1
ze z 2 + 1
sin z
z+2
Ans 5a): f (z) =
z2− 5z + 4
2 −1
= +
(z − 4) z − 1

18
∞ ∞  j
1 X  z k X 1
=− −
2 4 z
k=0 j=1

X
= ak z k
k=−∞
where, 
− 1 4−k , k = 0, 1, 2, . . .
ak = 2
−1, k = −1, −2, . . .
1
z 2+1
ze
Ans 5b): f (z) =
sin z
(i) z=0 is removable singularity as,
lim f (z) = e
z→0

(ii) z= nπ where n∈I are simple poles since,


lim f (z) = ∞
z→nπ

(iii) z=±i are essential singularities since,


lim f (z) does not exis.t
z→±i

6. (a) Prove that 5x + 12x < 13x for all x ∈ (2, ∞)


(b) Find the value of the limit if it exists,
ex sin x − x(1 + x) e
ki
lim
x→0 x3  x  x
oo

5 12
Ans 6a): Let f (x) = + −1
13 13
pC

f (2) = 0
It has real root at say (2,α).
ar

∃c ∈ (2, α) such that f 0 (c) = 0


Sh

but f 0 (x) < 0 ∀ x

x2 x3
(1 + x + + . . . )(x − + . . . ) − x(1 + x)
Ans 6b): lim 2! 3!
 x→0  x3
3 1 1
x −
2 6 1
= lim 3
=
x→0 x 3

7. (a) Solve the given differential equation by the method of variation of parameters,
dy dy
+ 2 + 1 = et ln t
dt2 dt
(b) Solve the following system of linear differential equation,
dx
=x+y Given y=5 and x=10 when t=0
dt
dy
= 4x − 2y
dt

Ans 7a): Auxiliary equation : m2 + 2m + 1 = 0


m1 , m2 Z= −1
t2 t2
u1 = − t ln t.dt = − ln t +
2 4

19
Z
u2 = ln t.dt = t ln t − t
t2 3
yp = u1 y1 (t) + u2 y2 (t) = ( ln t − t2 )e−t
2 4
Ans
7b): Auxiliary
equation:
1 − k 1
=0 ⇒ k = −3, 2
4 −(2 + k)
x = c1 e−3t + c2 e2t
y = −4c1 e−3t + c2 e2t
Using given condition,
x = e−3t + 9e2t
y = −4e−3t + 9e2t

8. a) f (x, y) = x2 − 2xy + 2y Find the maximum value of f (x, y) given 0 ≤ x ≤ 3 and 0 ≤ y ≤ 2 .


−xy 3
(b) Given the function f (x) = 2
x + y2
(i)Prove that f (x) is continuous at the point (0,0)
(ii) Check whether fxx is continuous at the point (0,0)
(iii) Check whether the function f (x, y) is differentiable or not at the point (0,0)

Ans 8a): Critical point is interior


fx = 0 and fy = 0 ⇒ (1, 1) is critical point.
f (1, 1) = 1
Along boundaries,
x = 0 f (0, y) = 2y 0 ≤ y ≤ 2 e
min f = 0 , maxf = 4;
ki
2
y = 2 f (x, 2) = x − 4x + 4 0 ≤ x ≤ 3 min f = 0 , maxf = 4;
oo
pC

x = 3 f (3, y) = 9 − 4y, 0 ≤ y < 2 min f = 0 , maxf = 9;


y = 0 f (x, 0) = x2 , 0 ≤ x ≤ 3 min f = 0 , maxf = 9;
ar

Absolute Maximum = 9
Sh

Absolute Minimum = 0

Ans 8b(i): fx (0, 0) = 0


−x2 y 3 + y 5
fz (x, y) =
(x2 + y 2 )2
lim fx (x, y) = 0
(x,y)→(0,0)
2xy 3 (x2 − 3y 2 )
(ii) fxx =
(x2 + y 2 )
lim along y = mx depends on m so limit does not extend.
(x,y)→(0,0)

(iii) Since fx is continuous and fy exists at (0,0) the function is differentiable .

*****

20
MATHS I
END SEMESTER 2015-16

Ans 1a): (x + λ2 )ex


x2 x3 x4
=(x + λ2 )(1 + x + + + + . . .)
2! 3! 4!
1 λ2 e
ki
∴ coefficient of x64 = +
63! 64!
oo

5 1 λ2
∴ = +
63! 63! 64 × 63!
pC

λ 2
or 5 = 1 +
8
ar

∴ λ = ±16
Sh

Ans 1b): y = ex (sin x + cos x)


dy
or = ex (sin x + cos x + cos x − sin x) = ex (2 cos x)
dx
dy
∴ For =0
dx
π 3π
x= ,
2 2
d2 y
∴ 2 = 2ex (cos x − sin x)
dx  
d2 y π
∴ 2 = 2eπ/2 (−1) ∴ convex downwards
dx 2
2
d y
∴ 2 = 2e3π/2 (1) ∴ convex upwards.
dx
Question 1(c) is not in syllabus.

21
f (0, k) − f (0, 0)
Ans 2a): fy (0, 0) = lim =0
k→0 k−0
f (h, k) − f (h, 0)
fy (h, 0) = lim
k→0 k−0
1 hk(h2 + 2k 2 )
= lim ×
k→0 k (h2 + k 2 )
h(h + 2k 2 )
2
= lim =h
k→0 h2 + k 2
fy (h, 0) − fy (0, 0)
fxy (0, 0) = lim =1
h→0 h−0
fx (0, 0) = lim
f (h, 0) − f (0, 0)
=0 e
ki
h→0 h−0
f (h, k) − f (0, k)
oo

fx (0, k) = lim
h→0 h−0
pC

1 hk(h2 + 2k 2 )
= lim ×
k→0 h (h2 + k 2 )
ar

k(h + 2k 2 )
2
= lim = 2k
Sh

k→0 h2 + k 2
fx (0, k) − fx (0, 0)
fyx (0, 0) = lim =2
k→0 k−0
64
Ans 2b): f (x, y) = 2 × (x + z) + xz
xz
 
1 1
= 128 + + xz
z x
−128 −128
fx = 2
+z fz = +x
x z2
128 128
z= 2 x= 2
x z
128
z=  
128 128
z2 z2
z 4
z=
128
z 3 = 27
∴ z = 27/3
27
∴ x = 14/3
2 14
∴ x = 27− 3
∴ x = 27/3
Now , x × z × y = 64
∴ 27/3 × 27/3 × y = 26

22
24 − 7 − 7
( )
∴y=2 4
∴ y = 210/4 = 25/2
Reason for minima :
−128 −128
fxx = 3
(−2) fzz = (−2)
z z3
fzx = 1
128 × 2
fxx = =2
27
128 × 2
fzz = =2
27
∴ fxx fzz − fzx = 4 − 1 = 3 > 0
Thus its a point of local minimum.
∴ x = 27/3 , y = 25/2 , z = 27/3

e
ki
oo
pC
ar

dy
Sh

Ans 3a): (2xey + 3y 2 ) + (3x2 + λey ) = 0


dx
For exact , M dxN dy = 0
My = Nx where 2xey + 3y 2 = N
3x2 + λey = M
∴ My = λe y

∴ Nx = 2ey
∴λ=2
∴ (2xey + 3y 2 )dy + (3x2 + 2ey )dx = 0
or 3y 2 dy + 3x2 dx + d(2xey ) = 0
or d(x3 + y 3 ) + d(2xey ) = 0
or x3 + y 3 + 2xey = c where c = integration constant

d
Ans 3b): y + (xy) = x(sin x + ln x)
dx
or ydx + xdy + ydx = x(sin x + ln x)dx
dy 2y
or + = (sin x + ln x)
dx x
R dx
!
2
∴ IF = eZ x
∴ yx2 = x2 (sin x + ln x)dx
 
2 sin x x ln x 2 x c
∴y= + + 2
+ 1 cos x − + 2
x 3 x 9 x

23
ln y 2
 
dy y
Ans 3c): + ln y = y
dx x x
ln y 2
 
1 dy ln y
or + =
y dx x x
Let ln y = z
dz 1 dy
=
dx y dx
dz z  z 2
∴ + =
dx x x
z
Let = u
x
z = ux
dz du
=u+x
dx dx
du
u+x + u = u2
dx
du
or x = u2 − 2u
dx
du dx
or 2 =
u − 2u x
du dx
or (u − (u − 2)) =
u(u − 2)  x
1 1 2dx
or du − =
u−2 u x
du du dx
or − =2
u−2 u x e
ki
du du 2dx
or − =
u−2 u x
oo

u − 2
or ln = 2 ln |x| + ln |c|
pC

u
2
or ln 1 − = ln |cx2 |
u
ar

2
∴ 1 − = cx2
Sh

u
2
or u =
1 − cx2
ln y 2
or =
x 1 − cx2
2x
or ln y =
1 − cx2

24
d2 y
Ans 4a): + 4y = 3cosecx
dx2
2
∴ (D + 4)y = 3cosecx
∴ General solution are sin2x and cos 2x (y1 and y2 )
sin 2x cos 2x
∴ w = = −2(sin2 2x + cos2 2x) = −2
2 cos 2x −2 sin 2x
−y2 f (x)
Z Z
y1 f (x)
y = c1 y1 + c2 y2 + y1 dx + y2 dx
w w
− cos 2x
Z Z
sin 2x
= c1 sin 2x + c2 cos 2x + sin 2x (cosecx)dx + cos 2x (cosecx)dx
Z −2 Z−2
sin 2x cos 2x
= c1 sin 2x + c2 cos 2x + (cosecxdx − 2 sin xdx) − 2 sin x cos xcosecxdx
2  2
1
= c1 sin 2x + c2 cos 2x + (sin 2x) cos x − ln |cosecx + cot x| − sin x cos 2x
2
4
Ans 4b): (D + 2D + 1)y = 02

Roots of auxiliary equation :


m4 + 2m2 + 1 = 0
or (m2 + 1)2 = 0
or m = ±i, ±i
∴ y = (c1 + c2 x) sin x + (c3 + c4 x) cos x
y(0) = 0 y(π/2) = −1
or c3 = 0 or c1 + c2 (π/2) = −1
c1 = c2 (−π/2) − 1
c1 + c4 = −1 and c2 − c4 (π/2) = −1
∴ c2 (−π/2) − 1 + c4 = −1 e
ki
or c2 (−π/2) + c4 = 0
oo

πc2
or c4 =
2
pC

πc2 π
∴ c2 − × = −1
2 2
ar

π 2
or c2 ( − 1) = 1
Sh

4
4
or c2 = 2
π −4

or c4 = 2
π −4
2π 4 − 2π − π 2
and c1 = −1 − 2 =
π − 4 π 2 − 4
2
  
4 − 2π − π 4x 2πx
∴y= + 2 sin x + cos x
π2 − 4 π −4 π2 − 4

25
d2 y dy
Ans 5a): (3 + 2x)2 + 2(3 + 2x) + 4y = 8x
dx2 dx
Let (3 + 2x) = u
dy dy du 2dy
= × =
dx du  dx  du
d2 y d dy du 4dy
= 2 =
dx2  du du dx du2
2
 
2 4d y dy u−3
∴u + 2(2u) + 4y = 8
duu2 du 2
or u2 u00 + uu0 + y = u − 3
Let u = ez
∴ (D(D − 1) + D + 1)y = ez − 3
or (D2 + 1)y = ez − 3
ez 3 ez − 6
or y = − = [complimentary function]
2 1 1
ez − 6
y = c1 sin z + c2 cos z +
2
3
= c1 sin x(ln(3 + 2x)) + c2 cos(ln(3 + 2x)) + x − [integration constant]
2
dy1
Ans 5b): = y1 + 4y2
dx
dy2
= 2y1 − y2
dx
By the method of undetermined coefficients,
1 − α 4
∴ =0
2 −1 − α e
ki
or (α − 1)(α + 1) − 8 = 0
oo
or α = ±3
∴ y1 =  c1 e3x + c2 e−3x
pC

1 dy1
y2 = − y1
4 dx
ar

1
= (3c1 e3x − 3c2 e−3x − c1 e3x − c2 e3x )
Sh

4
1 3x
e (2c1 ) − (4c2 )e−3x

=
4

Ans 6a): For function f (z) = arg(z)


Let z = x ± iy
For x > 0
lim arg(z) = 0
y→0

26
For x < 0
lim arg 1(z) = π and lim arg(z) = −π
y→0+ y→0−
∴ function is not continuous for z → x where x is real.
Ans 6b): Given uxx + uyy = 0 and vxx + vyy = 0
∴ F (x, y) = (uy − vx ) + i(ux + vy )
For F (x, y) to be analytic,
∂ ∂ ∂ ∂
(uy − vx ) = (ux + vy ) and (uy − vx ) = − (ux + uy )
∂x ∂y ∂y ∂x
For the 1st one:
uy = −2y vx = 6xy ux = 2x vy = 3x2 − 3y 2
uxy = 0 vxx = 6y uyx = 0 vyy = −6y

∴ uxy − vxx = −6y = uyx + vyy


For the 2nd one:
uy = −2y vx = 6xy ux = 2x vy = 3x2 − 3y 2
uyy = −2 vyx = 6x uxx = 2 vxy = 6x

∴ uyy − vyx = −6x − 2 = −(uxx + vxy )


∴ F (x, y) is analytic [C-R equations are satisfied]

Ans 6c): z 3 = (x + iy)3


= x3 + (iy)3 + 3x2 (iy) + 3x(iy)2
= x3 − iy 3 + 3ix2 y − 3xy 2
= (x3 − 3xy 2 ) + i(3x2 y − y 3 )
x3 − 3xy 3 + iy 3 e
ki
∴ f (x, y) =
x2 + y 2
oo

x(x − 3y 2 )
2 y3
∴u=
x2 + y 2 x2 + y 2
pC

f (h, 0) − f (0, 0)
∴ ux (0, 0) = lim
ar

h→0 h
h(h2 )
Sh

= lim =0
h→0 h2
f (0, k) − f (0, 0)
vy (0, 0) = lim =0
k→0 k
f (0, k) − f (0, 0)
uy (0, 0) = lim =0
k→0 k
f (h, 0) − f (0, 0)
ux (0, 0) = lim =0
h→0 h
∴ ux = vy and uy = −vx
f (z) − f (0)
For lim
z→0 z−0
(x3 − 3xy 2 ) + iy 3
−0
x2 + y 2
= lim
(x,y)→(0,0) (x + iy)
Let y = mx
(x3 − 3xm2 x2 ) + i(m3 x3 )
∴ lim
x→0 ((1 + im)x)(x2 )(1 + m2 )
(1 − 3m2 ) + i(m3 )
= which is a function of m.
(1 + im)(1 + m2 )
∴ the function is non-differentiable at origin.

27
e
ki
oo
pC

Z dz 1
ar

Ans 7(a): To prove: ≤ √



2
z +1
Γ 2 5
Sh

Length of Γ =11
M = M ax 2

1 z 1+ 1

   
1 1 1 1
= 2 = = ≤ max .max

2
z +1 z +1 |(z + i)(z − i)|  |(z +  i)||(z − i)| |z + i| |(z − i)|
1
In the path between 2 to 2 + i max occurs at z = 2 + i
  |z − i|
1 1 1
∴ max = =
|z − i| |2| 2
1 1 1
∴ 2 ≤ ×√

z +1 2 5
1
∴M = √
2 5
byZ ML theorem of inequality,
dz Z dz 1
≤ ML ⇒ ≤ √

2 2
Gamma z + 1 z +1

2 5
Z πiz
e dz
Z
eπiz dz
Ans 7b): 2
=
Γ (2z − 4z − z + 2) ZΓ
(2z(z − 2) − (z − 2))
eπiz dz
=
Γ 2(z Z− 2)(z − 1/2)
1 2 ((z − 1/2) − (z − 2))
= × eiπz dz
2 3 Γ (z − 1/2)(z − 2)
Z iπz
eiπz
Z
1 e dz 1
= − dz
3 Γ z−2 3 Γ z − 1/2
28
1
= 0 − eiπ(1/2) = −i/3 [There is only one point of discontinuity]
3
Ans 7c):

Function is not analytic at z = −1 which is outside the domain of the given curve. D ≡ Im(z) > 0
Let Γ be the straight line joining (−1 + 2i) to (1 + 2i)
∴ zZ = x + 2i where x ∈ [−1, 1] e
ki
z+1−1
∴ dz
oo

ZΓ z +Z1
dz
pC

= dz −
Γ Γ z+1
= [z]1+2i − [ln|z + 1|]1+2i
ar

−1+2i −1+2i
2 + 2i
Sh

= [1 + 2i + 1 − 2i] − ln

2i

i − 1
= 2 − ln

−1

= 2 − ln |1 − i|

29
MATHS I
END SEMESTER 2011-12

Ans 1a): By LMVT e


ki
f (b) − f (a)
= f 0 (c) where c ∈ [a, b]
oo

b−a
For ∀ c ∈ (a, b), f 0 (c) = 0
pC

f (b) − f (b)
∴ =0
b−a
ar

or f (b) = f (a)
Sh

x(1 − p(1 − x2 /2! + x4 /4!)) + q(x − x3 /3! + x5 /5!) 1


Ans 1b): lim 3
=
 x→0 x 3
x2 x2

1−p 1 1 q 1
or lim ( 2 ) + − + q( 2 ) − + q( ) =
x→0 x 2! 4! x 3! 5! 3
1
∴ coefficient of 2 = 0
x
1 q 1
∴ 1 − p + q = 0 and − =
2! 3! 3
∴q=1
∴p=2

Part (c) is not in syllabus.

Ans 1d): f (x) = 3x3 − 4x2 + 5x − 1 f (3) = 59


0 2
f (x) = 9x − 8x + 5 0
f (3) = 62
f 00 (x) = 18x − 8 f 00 (3) = 46
f 000 (x) = 18 f 000 (3) = 18
(x − 3)2 (x − 3)3
f (x) = f (3) + f 0 (3)(x − 3) + f 00 (3) + f 000 (3)
2! 3!
= 59 + 62(x − 3) + 23(x − 3)2 + 3(x − 3)3

30
Ans 2a): eu = x3 + y 3 + z 3 − 3xyz
∴ u = ln(x3 + y 3 + z 3 − 3xyz)
= ln(x + y + z) + ln(x + ωy + ω 2 z) + ln(x + ω 2 y + ωz)
du 1 1 1
= + 2
+ 2
dx x + y + z x + ωy + ω z x + ω y + ωz
du 1 ω ω2
= + +
dy x + y + z x + ωy + ω 2 z x + ω 2 y + ωz e
ki
du 1 ω2 ω
= + +
oo

dz x + y + z x + ωy + ω 2 z x + ω 2 y + ωz
d2 u 1 1 1
=− − −
pC

dx 2 (x + y + z) 2 2
(x + ωy + ω z) 2 (x + ω y + ωz)2
2
2
d u 1 ω 2 ω
ar

2
=− 2
− 2 2

dy (x + y + z) (x + ωy + ω z) (x + ω y + ωz)2
2
Sh

2
d u 1 ω ω2
= − − −
dz 2 (x + y + z)2 (x + ωy + ω 2 z)2 (x + ω 2 y + ωz)2
d u d2 u d2 u
2 3
∴ 2 + 2 + 2 =−
dx dy dz (x + y + z)2
Ans 2b): z(x, y) = xn φ(y/x)
Let u be homogeneous function
u = xn φ(y/x)

differentiating u with respect to x ( )
∂x
n−1 −y 0 n
ux = nx φ(y/x) + 2 × φ (y/x)x
x
xux = nx φ(y/x) − yxn−1 φ(y/x) . . . . . . . . . . (1)
n

differentiating with respect to y,


uy = xn−1 φ(y/x)
yuy = yxn−1 φ(y/x) . . . . . . . . . . . (2)
(1) + (2)
yuu + xux = nxn φ(y/x) − yxn−1 φ(y/x) + yxn−1 φ(y/x)
⇒ xux + yuy = nxn φ(y/x)
⇒ xux + yuy = nu [proved]
 
x y
z = sin−1 + tan−1
y x
It is an homogeneous equation of order 0.
∴ nzx + yzy = 0

31
Dividing by xy, we get , zx /y + zy /x = 0

dψ dψ dx dψ dy
Ans 2c): = × + ×
dγ dx dγ dy dγ
dψ dψ
= cos θ + sin θ
dx  dy
d2 ψ

d dψ d dψ
2
= cos x cos x + ( sin θ) sin θ
dγ dx dx dy dy
d2 ψ 2
2 θ + d ψ sin2 θ
= cos
dx2 dy 2
dψ dψ dψ
= (−γ sin θ) + (γ cos θ)
dθ dx dy
d2 ψ d2 ψ 2 2 dψ d2 ψ 2 dγ
2
= 2
(γ sin θ) − (γ cos θ) + 2
(γ cos2 θ) − (γ sin θ)
dθ dx dx dy dy
d2 ψ 2 2 2
2 d ψ = d ψ + d ψ − γ dψ
∴ + γ
dθ2 dγ 2 dx2 dy 2 dγ
2
d ψ 2
d ψ dψ d2 ψ d2 ψ
∴ +γ 2 +γ = + 2
dθ2 dγ 2 dγ dx2 dy

e
ki
oo
pC
ar
Sh

 (Z̄)2

, Z 6= 0
Ans 3a): f (Z) = Z
0 , Z = 0
(Z̄)2 (Z̄)3
=
Z |Z|2
3
(Z̄) = (x  − iy)3 = (x3+ 3xy 2 2y + y3)
3 3 2
 ) + i(−3x
−3x y + y 3
2

(Z̄) x + 3xy
∴ = +i
|Z|2 x2 + y 2 x2 + y 2
2
x(x + 3y ) 2 y(−3x2 + y 2 )
u= v =
(x2 + y 2 ) x2 + y 2
u(h, 0) − u(0, 0)
ux (0, 0) = lim =1
h→0 h

32
u(0, k) − u(0, 0)
uy (0, 0) = lim =0
k→0 k
v(h, 0) − v(0, 0)
vx (0, 0) = lim =0
h→0 h
v(0, k) − v(0, 0)
vy (0, 0) = lim =1
k→0 k
∴ ux =vy and uy =  −vx [C-R
 2equations  satisfied]
2
x + 3y 2 y − 3x 2
lim x + iy
(x,y)→0 x2 + y 2 x2 + y 2
Let x = γ cos θ , y = γ sin θ
∴ lim [γ(cos3 θ + 3 sin2 θ cos θ) + iγ(sin3 θ − 3 cos2 θ sin θ)] = 0
γ→0
∴ continuous at origin.
For differentiability,
¯ 3
(4Z)
−0
| 4 Z|2
lim
4Z→0 4Z
x2 + 3y 2 y 2 − 3x2
 
x( 2 ) + iy( )
x + y2 x2 + y 2
= lim
(x,y)→(0,0) (x + iy)
x2 + 3y 2 y 2 − 3x2
 
1
= lim x( 2 ) + iy( 2 )
(x,y)→(0,0) (x + iy) x + y2 x + y2
Let y = mx
x2 + 3m2 x2 m2 x2 − 3x2
 
1
= lim x( 2 ) + imx( 2 e
)
ki
(x,y)→(0,0) x(1 + im) x + m 2 x2 x + x2 m2
oo

1 + 3m2
  2 
1 m −3
= + (im)
(1 + im) 1 + m2 1 + m2
pC

It is a function of m.
∴ not differentiable.
ar
Sh

Ans 3b): u = 6x − 2xy


ux = 6 − 2y uy = −2x
uxx = 0 uyy = 0
∴ uxx + uyy = 0
Also for analytic function ux = vy and uy = −vx
∴ vy = 6 − 2y
or v = 6y − y 2 + f (x)
vx = f 0 (x) = −uy = 2x
or f (x) = x2
∴ v = 6y − y 2 + x2
∴ u + iv = (6x − 2xy) + i(6y − y 2 + x2 )
 2
∂2


Ans 3c): + (u2 + v 2 ) ux = vy and uy = −vx where f (Z) = u + iv
∂x2 ∂y 2
2uuxx + 2u2x + 2vvxx + 2vx2 + 2uuyy + 2u2y + 2vvyy + 2vy2
= 2u(uxx + uyy ) + 2v(vxx + vyy ) + 2(u2x + vx2 + u2y + vy2 )
As f (Z) is analytic, then it is harmonic
∴ uxx + uyy = vxx + vyy = 0
Also ux = vy and uy = −vx
∴ 4(u2y + vy2 ) = 4|f 0 (Z)|2

33
Ans 4a): (x2 − iy 2 ) is analytic in the domain.
∴ it is not path dependent.
∴ z = x + iy
= x + i(2x2 )
∴ dz = dx + i(4x)dx = dx(1 + i4x)
Z 2
∴ (x2 − i4x4 )(1 + i4x)dx
1 Z 2
= (x2 + i4x3 − i4x4 + 16x5 )dx
1  
1 16 1 1
= [7] + [64 − 1] + 4i [15] − [32 − 1] e
ki
3 6  4  5
oo
7 8 15 31
= + × 63 + 4i −
3 3 4 5
pC

511 49i
= −
3 5
ar

Z
z + 3i
Ans 4b):
Sh

(z 2 − iz + 2)3
c
z 2 − iz + 2 = z 2 + iz − 2iz + 2
= z(z + i) − 2i(z + i)
= (z + i)(z − 2i)
C : |z − 1 − 2i| = 2
For z = −i √
| − i − 1 − 2i| = |1 + 3i| = 10 > 2
For z = 2i √
|2i − 1 − 2i| = 1√< 2
Z Not analytic 2

(z + 3i)dz
c (z + i)3 (z − 2i)3
z + 3i z 3i
f (z) = 3
= 3
+
(z + i) (z + i) (z + i)3
3
(z + i) − 3z(z + i) 2 3i × 3
f 0 (x) = 6

(z + i) (z + i)4
1 3z 9i
= 3
− −
(z + 1) (z + i)  (z + i)4
4

(z + i)4 − z 4 (z + i)3

00 −3 9i(−4)
f (z) = − −3
(zi )4 (z + i)5 (z + i)8
36i 3 3 127
= − − +
(z + i)5 (z + i)4 (x + i)4 (z + i)5
36i 3 3 12(3i)
∴ f 2 (2i) = 5 4 − 4 4 − 4 4 + 5 5
3 i 3 i 3 i 3 i
34
9×4 4×9 3 3
= + − −
9 × 27 9 × 27 3 × 27 3 × 27
8 2 2
= − =
27 27 9
2 2 z + 3i
∴ f (2i) = φc dz
2πi (z + i)3
(z + 3i)dz 2πi
∴ φc 2 =
(z − iz + 2)3 9
Ans 4c): |z − 2| = 1
For z = −1 : | − 1 − 2| = 3 > 1
Z z 2= 2 : |2 − 2| = 0 < 1
For
z − 5z + 3
dz
(z + 1)(z − 2)2
z 2 − 5z + 3
f (z) =
z+1
(2z − 5)(z + 1) − (z 2 − 5z + 3)
f 0 (z) =
(z + 1)2
2z + 2z − 5z − 5 − z 2 + 5z − 3
2
=
(z + 1)2
2
z + 2z − 8 (z + 1)2 − 9 9
= 2
= 2
=1−
(z + 1) (z + 1) (z + 1)2
9
f 0 (2) = 1 − = 0
9
0 1! H
f (2) =
2π c
c e
ki
I 2
z − 5z + 3
oo
or 2
dz = 0
c (z + 1)(z − 2)
pC
ar
Sh

dy y
Ans 5a): = (y ln x − 1)
dx x
(y 2 ln x − y)dx − xdy = 0
M dx + N dy = 0
y 2 lnxdx = ydx + xdy
ydx + xdy
or ln xdx =
y2
(ln x) d(xy)
=
x2 (xy)2
(xy) −1 − ln x − 1
or = −c
−1 x
1 ln x + 1
or = +c
xy x
Ans 5b): xdy − ydx = (x2 + y 2 )dx

35
 
xdy − ydx y2
or = (1 + )dx
x2 x
y
d( )
or x
 y 2 = dx
(1 + )
xy 
or tan−1 =x+c
x
π
or tan−1 (1) = + c
2
π
or c = −
4y  π
∴ tan−1 =x−
x 4
Ans 5c): (D5 − D)y = |2ex + 3 sin x − 2x|
to get complimentary function we solve
(D5 − D)y = 0
m4 (m − 1) = 0
⇒ m = 1, 0, 0, 0, 0
∴ C.F. = c1 ex + c2 + c3 x + c4 x2 + c5 x3
To find the particular integral,
1
P.I. = 4 [12ex + 8 sin x − 2x]
D (D −  1) 
x 1 1 2
= 12e × x + − × 8(sin x + cos x) + 4 (1 + D + D + . . .)(2x)
2 D
1
= 12ex .x − 4 sin x + 4 (2x + 2) − 4 cos x e
ki
D  5
x4

x
oo

x
P.I. = 12e .x − 4 sin x − 4 cos x + 2 +
120 24
pC

x5 x4
 
General Solution = c1 ex x2
+ c2 + c3 x + c4 + c5 x3 + 12ex .x − 4 sin x − 4 cos x + 2 +
120 24
ar


f (x) g(x)
Sh

Ans 5d): Wronskian(W) = 0


f (x) g 0 (x)

2
f (x) = x2 , g(x) = e−x
f 0 (x) = 2x , g 0 (x) = −2xe −x2

2
e−x
2
x

2 2
∴W= 2 = −2x3 e−x − 2xe−x
2x −2xe−x
2
W = −2xe−x (x2 + 1)

log x sin(log x)
Ans 6a): x2 y 00 − 3xy 0 + y =
x

36
Put x = ez or z = log x
z sin z
(D0 (D0 − 1) − 3D0 + 1)y =
ez
Complimentary function,
(D2 − 4D + √
1)y = 0 √
y = c1 e (2+ 3)x + c2 e(2− 3)x
Particular Integral:-
 
1 z sin z
D3 − 4D + 1 ez
1 1
Using (eax V ) = eax V
f (D) f (D + a)
1
= (e−z ) 2 (z sin z)
D − 6D + 6
1 1 f 0 (D)
Using (xV ) = x. V − V
 f (D) f (D) {f (D)}2 
1 1 (2D − 6)
= z z 2 (sin z) − (sin z)
e  D − 6D + 6 (D2 − 6D + 6)2 
1 1 (2D − 6)
= z z (sin z) − (sin z)
e  5 − 6D (25 − 36D2 − 60D) 
1 (5 + 6D) (2D − 6)
= z z (sin z) − (sin z)
e  25 − 36D2 (60D + 11) 
1 z (2D − 6)(60D − 11)
= z (5 + 6D) sin z − (sin z)
e  61 (3600D2 − 121)
(120D2 − 382D + 66)

1 z
= z
e  61
(5 sin z + 6 cos z) +
−3721 esin z
ki

1 z (66 sin z − 382 cos z − 120 sin z)
oo
= z (5 sin z + 6 cos z) − sin z
e 61 3721
pC

∴ General Solution = C.F. + P.I.


√ √
 
(2+ 3)x (2− 3)x 1 z (66 sin z − 382 cos z − 120 sin z)
= c1 e + c2 e + z (5 sin z + 6 cos z) − sin z
ar

e 61 3721
Sh

2
Ans 6b): y 00 − y =
1 + ex
Complimentary solution :
(D2 − 1)y = 0
m2 − 1 = 0
⇒ m = ±1
y = c1 ex + c2 e−x
u = ex , v = e−x
P.I. : y = L1 u + L2 v
u v ex e−x

W = 0 0 = x = −1 − 1 = −2
u v e −e−x
2e−x
 
1
L01 = − x
×−
Z 1+Ze 2
e−x
⇒ L01 = dx
1 + ex
⇒ L1 = ln(ex + 1) − e−x − x
2ex 1 −ex
L02 = × − =
Z 1 + exZ 2 x 1 + ex
e
⇒ L02 = −
1 + ex
⇒ L2 = − ln(e + 1)x

P.I. = (ln(ex + 1) − e−x − x)ex + (− ln(ex + 1))e−x


= ex ln(ex + 1) − 1 − xex − ln(ex + 1)e−x

37

dx dy
Ans 6c): + 2y + sin t = 0 − 2x − cos t = 0 −→(2)

dt dt
Differentiating with respect to t
d2 x 2dy
+ + cos t = 0 −→ (1)
dt2 dt
(1) - 2x(2)
d2 x
+ 4x + 3 cos t = 0
dt2
(D2 + 4)x = −3 cos t
P.I. :-
1
x= 2
(−3 cos t)
(D + 4)
x = − cos t
Solution : x = c1 cos 2t + c2 sin 2t − cos t
Putting t = 0 and x = 1
⇒ c1 = 2
∴ x = 2 cos 2t − cost+ c2 sin 2t
1 dx
y=− + sin t
2 dt
1
y = − (−4 sin 2t + sin t + 2c2 cos 2t + sin t)
2
1
y = − (2c2 cos 2t − 4 sin 2t + 2 sin t)
2
Putting t = 0 and y = 1
⇒ c2 = −1
e
ki
∴ x = 2 cos 2t − sin 2t − cos t
oo
∴ y = 2 sin 2t − sin t + cos 2t
pC
ar
Sh

38
MATHS II
MID SEMESTER EXAMINATION 2017-2018

Ans a): Let a,b,c be scalars (real) such that


a(2x3 + x2 + x + 1) + b(x3 + 3x2 + x − 2) + c(x3 + 2x2 − x + 3) = 0
(2a + b + c)x3 + (a + 3b + 2c)x2 + (a + b − c)x + (a − 2b + 3c) = 0
Equating coefficients of like powers of x, e
ki
2a + b + c = 0
oo

a + 3b + 2c = 0
a+b−c=0
pC

a − 2b + 3c = 0    
2 1 1 1 3 2
ar

1 3 2  ∼ 2 1 1

Coefficient matrix ⇒ A =  1 1 −1 1 1 −1 . . . . . . . . . . .[R1 ∼ R2 ]
Sh

1 −2 3 1 −2 3
 
1 3 2
0 −5 −3 R2 = R2 − 2R1
∼ 0 −2 −3
 R3 = R3 − R1
R4 = R4 − R1
0 −5 1
 
1 3 2
0 1 3/5
∼ 0 −2 −3 
 R2 = (−1/5)R2
0 −5 1
 
1 3 2
0 1 3/5
∼ 0 0 1 
 R3 = (−5/9)R3
0 0 4
So clearly linearly independent.
b): Let V = R2 be the set of all ordered pairs (x,y) of real numbers.
Operations: (x1 , y1 ) + (x2 , y2 ) = (x1 + x2 , y1 + y2 )
c(x, y) = (|c|x, |c|y)
In this case,
(a + b)α = aα + bα . . . . .[where a, b ∈ R and α ∈ V ]
So let α = (x, y) and [a,b∈ R]
⇒ (a + b)(x, y) = |a + b|(x, y)
but,

39
(a + b)(x, y) = a(x, y) + b(x, y)
= (|a|x, |a|y) + (|b|x, |b|y)
= (|a| + |b|)(x, y)
But we know,
|a + b| ≤ |a| + |b| triangle inequality
hence V(R) is not a vector space.

 
1 −1 2 3
Ans a): A = 11 24 29 61
2 3 5 10
 
1 −1 2 3
∼ 0 5 1 4 e
ki
0 0 0 0
oo

∴ dim(V ) = 2 . . . . . . . .[since dim(V) = number of linearly independent vectors in the echelon form]
Any 2 vectors in S
 form basis; for example (1,-1,2,3),(2,3,5,10)
pC

−7/5
b): N(T) = span 1/5 
ar

1
Sh

 
−7/5
Basis of N(T) =  2/5 
1
any non-zero vector in N(T) is its basis.
By RN theorem , dim(R(T)) + dim(N(T)) = dim(V) ⇒ dimR(T) = 2

Ans 3a): f (−1) = −a + b − c + d = 5


f (−2) = −3a + 4b − 2c + d = 7
f (2) = 8a + 4b + 2c + d = 11
f (3) = 27a + 9b + 3c + d = 37

40
    
−1 1 −1 1 a 5
−8 4 −2 1  b   7 
 3 4 2 1  c  = 11
    

27 9 3 1 d 37
on
 solving the augmented matrix,
−1 1 −1 1 | 5
−8 4 −2 1 | 7 
 
 8 4 2 1 | 11
27 9 3 1 | 37
we find; a = 1, b = 2, c = 3, d = 1
f (x) = x3 + 2x2 − 3x + 1

b): x = 1-a; y = 1+a; z = a;


⇒x+y=2
⇒x+z=1

e
ki
oo
pC
ar
Sh

Ans 4a): Finding


 eigenvalues;

−λ 1 0
 1 −λ 0  ⇒ (1 − λ)(λ2 − 1) = 0
0 0 1−λ

 λ = 1, 1, −1  

−1 1 0 x
 1 −1 0 y  = 0
0 0 0 z
   
1 0
From here we get eigenvectors 1 and 0
0 1
 
1
And from the other eigenvalue we get the eigenvector −1
0
   
1 0 1 0.5 0.5 0
Thus D = P −1 AP . . . . . . . where P = 1 0 −1 and P −1 =  0 0 1
0 1 0 0.5 −0.5 0
 
1 0 0
From here we get D = 0 1 0 
0 0 −1
41
b): For ∗ −1
√A to √be a unitary
√ matrix A has to be equal to A
1/ 2 i/ 2 1/ √2 ia
⇒ =I
−ia b −i/ 2 b
 √ √ 
√ 1 √ i(a/ 2 + b/ 2)
⇒ =I
−i(a/ 2 + b/ 2) (a2 + b2 )
⇒ a = −b and√a2 + b2 √ =1
⇒ (a, b) = (1/ √2, −1/√2)
⇒ (a, b) = (−1/ 2, 1/ 2)

e
ki
oo
pC
ar
Sh

 
0.45 0.3 −15.00 14.28
coln
1.57  . . . . . . . .[Diagonally dominant means |ai1 | ≥ no.of
P
Ans 5a):  4.5 0.15 0.3 aij ]
j=1
0.15 −10.5 0.45 −3.86
Making
 it diagonally dominant and solving,

1.00 0.03333 0.0667 0.3489
−0.0143 1.00 −0.0429 0.3676 
−0.0300 −0.0200 1.00 −0.9520

5b): Using Gauss Jacobi


(n) (n)
xn+1
1 = 0.3489 − 0.0333x2 − 0.0667x3
(n) (n)
xn+1
2 = 0.3676 + 0.0143x1 + 0.0429x3
(n) (n)
xn+1
3 = 0.9520 + 0.0300x1 + 0.0200x2

42
5c):
(n) (n) (n)
n x1 x2 x3
0 0.0000 0.0000 0.0000
1 0.3489 0.3676 −0.9520
2 0.4002 0.3317 −0.9342
3 0.4002 0.3332 −0.9334
5b): Using Gauss Siedal method
(n) (n)
xn+1
1 = 0.3489 − 0.0333x2 − 0.0667x3
(n+1) (n)
xn+1
2 = 0.3676 + 0.0143x1 + 0.0429x3
(n+1) (n+1)
xn+1
3 = 0.9520 + 0.0300x1 + 0.0200x2

Ans a): Using Newton Rhapson, e


ki
f (xn )
xn+1 = xn − 0
oo

f (xn )
e2xn − xn − 6
pC

xn+1 = xn −
2e2xn − 1
Given xo = 0.97
ar

We get root = 0.9705


Sh

6b): f (0.5) = −0.4375


f (1.0) = 1
(0.5 + 1)/2 = 0.75; f (0.75) = 0.06640
∴ root lies between 0.5 and 0.75
(0.5 + 0.75)/2 = 0.625 f (0.75) = −0.2224
So root lies between (0.625,0.75)

Ans 7a): Usinmg Lagrange’s Interpolation


(x − x1 )(x − x2 ) (x − x2 )(x − xo ) (x − xo )(x − x1 )
Px (x) = yo + y1 + y2
(xo − x1 )(xo − x2 ) (x1 − x2 )(x1 − xo ) (x2 − xo )(x2 − xo )

43
∴ we get,
f(x) = 6x + 2

|f (2) (t)|
b): max |f (x) − p(x)| ≤ max × max|(x − xo )(x − x1 )|
0≤x≤1 0≤t≤1 2!
1 1
= max max |(x − 0)(x − 1)|
2 0≤t≤1 (1 + t)2 0≤x≤1
1
⇒ max =1
0≤t≤1 (1 + t)2

1
⇒ max |(x − 0)(x − 1)| = [since x lies between 0 and 1 (x-0)(x-1) is -ve and min value of f(x)=-1/4
0≤x≤1 4
at x=1/2. Thus |min| becomes the maximum value]
1
∴ max |f (x) − p(x)| ≤
0≤x≤1 8

e
ki
oo
pC
ar
Sh

44
MATHS II
MID SEMESTER EXAMINATION 2017
1. a) Find the dimension and basis of a cubic polynomial P(x) given that P(1)=P(2)=P(3).
Ans 1a): P(x) is a cubic polynomial with P(1)=P(2)=P(3). We can assume,
P (x) = α(x − 1)(x − 2)(x − 3) + β
So the basis of this polynomial space, [ 1,(x-1)(x-2)(x-3) ]
Hence the dimension of the polynomial space = 2
Alternate Solution (lengthy process): Assume a cubic polynomial P (x) = ao + a1 x + a2 x2 + a3 x3
Now, P(1)=P(2)=P(3)
⇒ ao + a1 + a2 + a3 = ao + 2a1 + 4a2 + 8a3 = ao + 3a1 + 9a2 + 27a3
taking two at a time,
⇒ a1 + 3a2 + 7a3 = 0 . . . . .(1)
⇒ a1 + 5a2 + 19a3 = 0 . . . . . .(2)
⇒ 2a1 + 8a2 + 26a3 = 0 . . . . . . .(3)
Solve this via Gaussian Elimination method or it is clear that (3)=(1)+(2). So there will be only two
independent equations for 4 variables, namely, ao , a1 , a2 and a3 .
Thus Dimension = 2, and basis can be found via Gaussian Elimination as [ 1, (x-1)(x-2)(x-3) ].
2. a) Let T : R3 −→ R3 be a linear transformation defined by T(1,0,0)=(1,0,1), T(0,1,0)=(1,1,0)
and T(0,0,1)=(0,1,1). Find the form of linear transformation T. Also find the null-space N(T)
of T and its dimension.
Ans 2a): Let (x, y, z) ∈ R3 Then, e
ki
(x, y, z) = x(1, 0, 0) + y(0, 1, 0) + z(0, 0, 1)
oo

Then T (x, y, z) = xT (1, 0, 0) + yT (0, 1, 0) + zT (0, 0, 1)


= x(1, 0, 1) + y(1, 1, 0) + z(0, 1, 1)
pC

= (x + y, y + z, x + z)
∴ T (x, y, z) = (x + y, y + z, x + z), ∀(x, y, z) ∈ R3
ar

Let (x, y, z) ∈ N (T )
Sh

Then T (x, y, z) = θ, where θ is the zero element in R3 .


(x + y, y + z, x + z) = (0, 0, 0)
(x + y) = 0, (y + z) = 0, (x + z) = 0
∴ the solution is x=0, y=0, z=0.
∴ null space of T, N(T) = {θ}
∴ dimension of N(T) = 0.
2 b)Let T : P3 (R) −→ P2 (R) be the linear transformation defined by
T (f (x)) = f 0 (x)
where the prime(’) denotes derivative with respect to x. Find the matrix representation of T
w.r.t the standard ordered basis {1, x, x2 , x3 } for P3 (R) and {1, x, x2 } for P2 (R) respectively.
Ans 2b): Given T : P3 (R) −→ P2 (R) defined by T(f(x)) = f’(x)
Now, T (1) = f 0 (1) = 0 = 0 × 1 + 0x + 0x2
T (x) = f 0 (x) = 1 = 1 × 1 + 0x + 0x2
T (x2 ) = f 0 (x2 ) = 2x = 0 × 1 + 2x + 0x2
T (x3 ) = f 0 (x3 ) = 3x2 = 0 × 1 + 0x + 3x2
∴ Matrix
 representation
 of T wrt. the basis for both P3 (R) and P2 (R) is
0 1 0 0
A=0 0 2 0
0 0 0 3
3. a) Consider the following equations
x + 2y + 3z = 6
y + (λ − 3)z = 3
y − z = µ − 12

45
What are the values of λ and µ for
(i) No solution
(i) Unique solution
(i) Infinitely many solutions
Ans 3a):  Augumented matrix, 
  1 2 3 6
A|b = 0 1 λ − 3 3 
0 1 −1 µ − 12
R3 −→ R3 −  R2 
  1 2 3 6
⇒ A|b = 0 1 λ − 3 3  (row echelon form)
0 0 2 − λ µ − 15
(i)No solution: λ = 2, µ 6= 15
(ii)Unique solution: λ 6= 2, µ ∈ R
(iii)Infinitely many solutions: λ = 2, µ = 15
3 b)  Write the given matrix in row-echelon form and thus find its rank.
3 0 1 2
 6 1 0 0
 
A= 12 1 2 4

 6 0 2 4
9 0 1 2
 
3 0 1 2
 6 1 0 0
 
Ans 3b): A= 12 1 2 4

e
ki
 6 0 2 4
oo

9 0 1 2
R2 −→ R 2 − 2R 1 ,  R3 −→ R3 − 4R1 , R4 −→ R4 − 2R1 , R5 −→ R5 − 3R1
pC


3 0 1 2
0 1 −2 −4
ar

 
A= 0 1 −2 −4 (row-echelon form)

Sh

0 0 0 0 
0 0 0 0
Rank of A = 3.

4 a)For
 the given
 matrix A, find all its eigen values and their respective geometric multiplicity.
−1 −2 2
A=−2 −1 4
−1 1 2
Ans
4a): |A − λI| = 0
−1 − λ −2 2

−2 −1 − λ 4 = 0

−1 1 2 − λ
2
⇒ λ(λ − 9) = 0
⇒ λ = 0, −3, 3
For λ = 0     

−1 −2 2 x 0
−2 −1 4 y  = 0
−1 1 2 z 0
Solving for x,y and z using Gaussian Elimination.
R2 → R2 − (R1 +R3)  
1 2 −2 x 0
⇒ 0 0 0  y  = 0
1 −1 −2 z 0
R3 −→ R3 − R1 and R2 ↔ R3

46
    
1 2 −2 x 0
⇒ 0 −3 0  y  = 0 (row-echelon form)
0 0 0 z 0
assuminf z to be free variable, let z = α
3y = 0 ⇒ y = 0
x = 2z − 2y = 2α
 eigen
∴  vecor,
 
x 2
y  = α 0 where α ∈ R − {0}
z 1
foe λ = 3,
    
−4 −2 2 x 0
−2 −4 4  y  = 0
−1 1 −1 z 0
Similarly solving (x,y,z) as shown in the case of λ = 0 via Gaussian Elimination,
eigen vector, (x,y,z) = β(0,1,1) where β ∈ R − {0}
 λ = 3,     
for
2 −2 2 x 0
−2 2 4 y  = 0
1 1 5 z 0
Similarly solving (x,y,z) as shown in the case of λ = 0 via Gaussian Elimination,
eigen vector, (x,y,z) = γ(0,1,1) where γ ∈ R − {0}
4 b)The following
  matrix satisfies the equation, A3 = aA2 + bA + c
3 1 1
e
ki
−1 5 −1
1 1 3
oo

Find the values of a,b and c.


Ans 4b): We can solve this question by finding A2 and A3 and then putting them in the equation to get the
pC

desired value, but Caley Hamilton’s method gives


 us an easier way out.
ar


3−λ 1 1
Characteristic equation:  −1 5 − λ −1  = 0
Sh

1 1 3−λ
3 2
λ − 11λ + 40λ − 44 = 0
Every matrix satisfies its own characteristic equation.
⇒ A3 − 11A2 + 40A − 44I = 0
⇒ A3 = 11A2 − 40A + 44I
Comparing with A3 = aA2 + bA + c we get,
a=11, b=-40 and c=44

5 a) Check the diagonizability


 of the matrix
6 −2 2
A=−2 3 −1
2 −1 3
If A is diagonizable then find an invertable matrix P such that P −1 AP = D where D is the
diagonal matrix of A.
Ans 5a): |A − λI| = 0
6 − λ −2 2

⇒ −2 3 − λ −1 = 0
2 −1 3 − λ
⇒ λ = 2, 2, 8
for λ = 2,
    
4 −2 2 x 0
−2 1 −1 y  = 0
2 −1 1 z 0

47
R3 −→ R3 
 + R2andthen  R2 −→ R2 + (1/2)R1
4 −2 2 x 0
0 0 0 y  = 0
0 0 0 z 0
Assuming y = α and z = β
α−β
4x = 2y - 2z ⇒ x =
2
Eigen
  vectors,   
x 1/2 −1/2
y  = α  1  + β  0  α, β ∈ R
z 0 1
Algebraic multiplicity of λ = 2 is 2.
Geometric multiplicity of λ = 2 is 2. (2 eigen vectors)
for λ = 8,
    
−2 −2 2 x 0
−2 −5 −1 y  = 0
2 −1 −5 z 0
R
 3 −→ R 3 − (R 2 − 2R
    1 )
−2 −2 2 x 0
−2 −5 −1 y  = 0
0 0 0 z 0
R2 −→ R2 − R1     
−2 −2 2 x 0
⇒ 0 −3 −3
   y = 0
 
0 0 0 z 0 e
ki
Assuming z = γ
oo

y = −γ
2y − 2z
x= = z − y = 2γ
pC

−2
(x, y, z) = γ(2, −1, 1) where γ ∈ R
ar

Algebraic multiplicity for λ = 8 is 1.


Sh

Geometric multiplicity for λ = 8 is 1.


As the geometric multiplicity and algebraic multiplicity of the eigen values i.e. 2,2,8 are respectively same so
the matrix
 A is diagonizable.

1/2 −1/2 2
P= 1 0 −1 (Any scalar multiples of the eigen vectors as columns will also be acceptable)
0 1 1
(You can interchange the first two columns among themselves and also place the 3rd column first.)
Thusdiagonalmatrix,
2 0 0
D = 0 2 0
0 0 8
and the
 invertible matrix, 
1/2 −1/2 2
P= 1 0 −1
0 1 1
5 b)If A and B are similar matrices then show that A and B has the same eigen values.
Ans 5b): It is sufficient to show that A and B have the same characteristic polynomials.
Let P −1 AP = B (A and B are similar)
Now let, (B − λI) = det(P −1 AP − λP −1 P )
= det(P −1 (A − λI)P )
= det(P −1 )det(A − λI)det(P )
= det(A − λI)
∴ det(B − λI) = det(A − λI)

48
Therefore A and B have same eigen values if they are similar.

6. a)Make the following matrix strictly diagonally dominant by rows :


x1 + 2x2 − 8x3 − 3x4 = 6
x1 + 7x2 + 3x3 − x4 = 9
5x1 − x2 − x3 − 8x4 = 10
4x1 + x2 + x3 + x4 = 5
Hence apply Jacobi’s method to perform three iterations with the initial guess (0,0,0,0)
Ans 6a): diagonally dominant :
(k+1) 1 (k) (k) (k)
4x1 + x2 + x3 + x4 =5 | x1 = (5 − x2 − x3 − x4 )
4
(k+1) 1 (k) (k) (k)
2x1 + 7x2 + 3x3 − x4 = 9 | x2 = (9 − 2x1 − 3x3 + x4 )
7
(k+1) 1 (k) (k) (k)
x1 + 2x2 − 8x3 − 3x4 = 6 | x3 = − (6 − x1 − 2x2 + 3x4 )
8
(k+1) 1 (k) (k) (k)
5x1 − x2 − x3 − 8x4 = 10 | x4 = − (10 − 5x1 + x2 + x3 )
8
(1) (2) (3)
x1 = 1.25 | x1 = 1.4286 | x1 = 1.0670
(1) (2) (3)
x2 = 1.2857 | x2 = 1.0714 | x2 = −0.7168
(1) (2) (3)
x3 = −0.75 | x3 = 0.1964 | x3 = −0.1027
(1) (2) (3)
x4 = −1.25 | x4 = −0.5357 | x4 = −0.5156

6. b) Consider 2x1 − x2 = 7
e
ki
−x1 + 2x2 − x3 = 1
−x2 + 2x3 = 1
oo

Use Gauss-Seidel method to perform 2 iterations.


pC

(k+1) 1 (k)
Ans 6b): x1 = (7 + x2 )
ar

2
(k+1a) 1 (k+1) (k)
Sh

x2 = (1 + x1 + x3 )
2
(k+1) 1 (k+1)
x3 = (1 + x2 )
2
using initial guess as (0,0,0)
(1) (2)
x1 = 3.5 x1 = 4.625
(1) (2)
x2 = 2.25 x2 = 3.625
(1) (2)
x3 = 1.625 x3 = 2.3125

Ans 7a): f (x) = cos x − xex

49
f 0 (x) = −(sin x + (1 + x)ex )
Newton Raphson formula,
f (xn )
xn+1 = xn − 0
f (xn )
cos xn − xn exn
So xn+1 = xn +
sin xn + (1 + xn )exn
0.0532
Let xo = 0.5 then x1 = 0.5 + = 0.5183
2.9525
−0.00082958
Putting x1 = 0.5183 we get, x2 = 0.5183 + = 0.517757
3.043513
Putting x2 = 0.517757 we get, x3 = 0.517757
Now rounding off to 3 decimal places we get, 0.517757 ≈ 0.518
Ans : 0.518

Ans 7b): To find the root of f (x) = 0 by using Newton Raphson method, we have the formula
f (xn )
xn+1 = xn − 0
f (xn )
On comparing with fixed point iteration method (ie. x = φ(x)) we see that,
f (x)
φ(x) ≡ x − 0
f (x)
Hence this method will converge if |φ0 (x)| < 1
⇒ |f (x)f 00 (x)| < |f 0 (x)|2

Ans 7c): Let n be the number of iterations required by bisection method to achieve an accuracy of > , then
corresponding formula is e

ki
b − a
2n ≤ 

oo

 
h |b − a| i
i.e. n ≥ loge ÷ loge 2
pC


Here |b − a| = 3,  = 0.005
ar

Thus n ≥ 9.2289 i.e. n=10


Sh

50
MATHS II
MID SEMESTER EXAMINATION 2016
1. By using row-echelon form, determine the value of k so that the following system of linear
equations
kx + y + z = 1
x + ky + z = 1
x + y + kz = 1
has (i) a unique solution (ii) No solution (iii) Infinite number of solutions.

Ans 1): kx + y + z = 1
x + ky + z = 1
x + y + kz = 1
So we form
 augmentedMatrix
k 1 1 | 1
A|B = 1 k 1 | 1

1 1 k | 1
 
1 k 1 | 1
∼ 0 1 − k k−1 | 0 
0 0 2
−k − k + 2 | 1 − k
(i) for k 6= 1 and k 6= -2
rank(A) = rank(A|B) = 3 e
ki
Therefore unique solution.
oo

(ii) For k = -2, rank(A) 6= rank(A|B)


pC

therefore no solution.
(iii) For k=1; rank(A) = rank(A,b) = 1<3
ar

2. (i) Find the dimension of the subspace V of R4 , where V = [(2x+y+z, −x+y−2z, x−z, 2x+2y) ∈
Sh

R4 ] where (x, y, z) ∈ R
(ii) Verify whether
W = {(x, y) : x, y ∈ R eitherx = 0 or y = 0 or both x = 0 = y}
is a subspace of R2

Ans 2(i): V = (2x + y + z, −x + y − 2z, x + z, 2x + 2y)


Thus, if v ∈ V , then,
v = x(2, −1, 1, 2) + y(1, 1, 0, 2) + z(1, −2, 1, 0)
[i.e. the subspaces are generated by vectors
v1 = (2, −1, 1, 2); v2 = (1, 1, 0, 2); v3 = (1, −2, 1, 0)]
To
 check
 linear dependence,

v1 2 −1 1 2
v2  = 1 1 0 2
v3 1 −2 1 0
 
2 −1 1 2
∼ 0 −3 1 −2
0 0 0 0
Since there are 2 non-zero rows ⇒ dim(V ) = 2.
2(ii): Note that (0,0)∈W
Let (a,0), (0,b) ∈ W . . . . [where a 6= 0; b 6= 0 ]
but (a,0) + (0,b) = (a,b) 6∈ W

51
∴ W is not a subspace of R2 .

3. Find the matrix of linear transformation on R3 ↔ R3 w.r.t the basis (1, 1, 0), (0, 1, 0) and (0, 0, 1)
given T (1, 1, 0) = (3, 4, 0) ; T (0, 1, 0) = (2, 1, 0) and T (0, 0, 1) = (5, −4, 1).

Ans 3): (x, y, z) = α(1, 1, 0) + β(0, 1, 0) + γ(0, 1, 1)


⇒ α = x, β = y − x − z, y = z
τ (x, y, z) = ατ (1, 1, 0) + βτ (0, 1, 0) + γτ (0, 1, 0) = (x + 2y + 3z, 3x + y − 5z, z)

(3,4,0) = 3(1,1,0) + 1(0,1,0) + 0(0,1,1)


(2,1,0) = 2(1,1,0) - (0,1,0) + 0(0,1,1)
(5,-4,1) =5(1,1,0) - 10(0,1,0)
 + (0,1,1)
3 2 5
Matrix : 1 −1 −10
0 0 1
4. Find
 the algebraic
 and geometric multiplicities of the eigen values of the matrix
2 −1 1
A = −1 2 −1
1 −1 2
 
2 −1 1
Ans 4): A = −1 2 −1
1 −1 2 e
ki
|A − λI| = 0 ⇔ (λ − 1)2 (λ − 4) = 0
oo

⇒ λ = 1, 1, 4
pC

λ = 1 ⇒ algebraic multiplicity is 2 [Algebraic multiplicity is the no. of times λ has same value ]
λ = 4 ⇒ algebraic multiplicity is 1
ar

Forλ = 1;   (A− I)λ


  =0
Sh

1 −1 1 x1 0
⇔ 0 0 0 x2  = 0
0 0 0 x3 0
Rank of coefficient matrix = 1
no of Liniearly independent eigen vectors = 3 - 1 = 2
Geometric multiplicity of λ=1 is 2.
Geometric multiplicity of λ=4 is 1.

5. (a)
 Find a matrix
 P for the given matrix A such that P −1 AP is a diagonal matrix.
1 −3 3
A = 3 −5 3

6 −6 4
(b) For a diagonalizable matrix A of the order n, prove that
αA + βIn is also diagonalizable where α and β ∈ R.

 
1 −3 3
Ans 5a): A = 3 −5 3
6 −6 4
⇒ C.E. ⇒ λ3 − 12λ − 16 = 0
eigen values: λ = 4, -2, -2
Eigen vectors:
For λ=4 Ax=4x

52
    
−3 −3 3 x 0
⇒  3 −9 3 y  = 0
6 −6 0 z 0
Reducing
 to row-echelon
    form,
−3 −3 3 x 0
 0 −12 6 y  = 0
0 0 0 z 0
Assume z = α
⇒ 2y = z
⇒x +y − z = 0
x 1
⇒ y  = α 1
z 2
 
1
Thus the eigen vector corresponding to λ = 4 is 1

2
   
1 0
Similarly workout the eigen vectors for λ = −2 which would come out to be scalar multiples of 1 and 1
0 1
∴ thematrix P consisting of the eigen vectors as its column space is
1 0 1
P = 1 1 1
0 1 2
or any matrix by interchange of vectors.
e
ki
5b): A is diagonalizable
⇒ ∃ a non-singular matrix P such that
oo

D = P −1 AP is diagonal
pC

⇒ A = P DP −1
αA + βIn = αP DP −1 + βIn = αP DP −1 + βP P −1
ar

= P D̂P ⇒ D is diagonal.
Sh

αA + βIn is also diagonalizable.

6. Consider the system of linear equations,


25x + 4y − z = 32
2x + 17y + 4z = 35
x + 3y + 10z = 24
(i) To solve by Jacobi’s iteration method, consider the above system in matrix form as
X K+1 = HX K + C. Write the matrices H and C with entries as fraction
(ii) Hence or otherwise perform 2 iterations (up to four decimal places) by Jacobi’s
method taking initial guess x = y = z = 0
(iii) For the same system , perform one iteration (upto four decimal places) by Gauss-Seidel
method with initial guess y = z = 0
 
28 4 −1
Ans 6(i): A =  2 17 4 
1 3 10
A = L + D +
U    
0 0 0 0 4 −1 28 0 0
L = 2 0 0 U = 0 0 4  D =  0 17 0 
1 3 0 0 0 0 0 0 10
 
0 −1/7 1/28
H = D−1 (L + U ) = −2/17 0 −4/17
−1/10 −3/10 0

53
  
1/28 0 0 32
C = D−1 b =  0 1/17 0  35
0 0 1/10 24
   
32/28 8/7
= 35/17 = 35/17
24/10 12/5
   
1.1428 0.9344
(ii): x(1) = 2.0588 x(2) = 1.3596
2.4000 1.6681
1
(iii) Gauss Siedal: x(k+1) = (32 − 4y (k) + z (k) )
28
1
y (k+1) = (35 − 2x(k+1) − 4z (k) )
17
1
z (k+1) = (24 − x(k+1) − 3y (k+1) )
10
x(1) = 1.1428
y (1) = 1.9244
z (1) = 1.7084

7. (a) Find the root of the equation x − cosx = 0 by the Newton Raphson method correct upto
7 decimal places. (take initial guess as 1)
7 (b) Write down the Newton Raphson expression for xn+1 for r times repeated roots.

f (xn )
Ans 7a): xn+1 = xn −
f 0 (xn ) e
ki
xn − cos(xn )
= xn+1 −
oo

1 + sin(xn )
x1 = 0.7503638
pC

x2 = 0.7391128
x3 = 0.7390891
ar

f (xn )
b): xn+1 = xn − r
Sh

f 0 (xn )

54
MATHS II
END SEMESTER EXAMINATION 2018

0 ω ω2 √
 
2 1 3
Ans 1a): A =  ω 0 ω  where ω = − −
2 2
ω2 ω 0
h
e i
ki
Cayley Hamilton theorem states that every matrix satisfies its identity matrix
oo
By
 Cayley Hamilton theorem
−λ ω ω 2

pC

 ω −λ ω 2  = −(1 + λ3 ) = −(1 + A3 ) = 0
ω 2 ω −λ
ar

⇒ A3 = −1
Sh

⇒ A−1 = −A2

and A3 = −1 ⇒ (A3 )33 = A99 = −1


⇒ A100 = −A

Ans 1b):

55
.

2. a) Consider the equation x2 − 4x + 3 = 0. Which of the following equations (i) and (ii) below
should be chosen so that fixed point iteration process converges to a root of the above equation
for any initial guess xo in the range (2,4)? Justify your answer.
x2 + 3 √
(i) x = (ii)x = 4x − 3
4
Ans 2a): For fixed point iteration
|g 0 (x)| < 1
x2 + 3
Using this x = ⇒ g 0 (x) > 1
√ 4
but x = 4x − 3 ⇒ g 0 (x) < 1
So (ii) is the answer.

2b)Applying Newton’s Forward interpolation formula determine the number of students who
obtained less than 45 marks from the data given below. e
ki
M arks : 30 − 40 40 − 50 50 − 60 60 − 70 70 − 80
oo

N o.of students : 31 42 51 35 31
Ans 2b):
pC

M arksbelow N o.of students M f M2 f M3 f M4 f


40 31
ar

42
Sh

50 73 9
51 −25
60 124 −16 37
35 12
70 159 −4
31
80 190
Using Newton’s forward interpolation
(k)(k − 1) 2 (k)(k − 1)(k − 2)(k − 3) 4
f (x) = f (xo ) + k. M f (xo ) + M f + ......... M f
2! 4!
45 − 40
where k = = 0.5
10
⇒ f (45) = 47.867 on solving
So f (45) is 47 or 48.
R4
3. a)Using Simpson’s 1/3rd Rule evaluate the integral (3 decimal places) 1 loge (x)(3x2 + 1) by
dividing [1,4] into 6 equal length sub-intervals.
Ans 3a): Let f (x) = loge (x)(3x2 + 1)
n=6 h=1/2
x 1 1.5 2 2.5 3 3.5 4
f (x) 0 3.14235 9.0109 18.09674 30.7618 47.29180 87.92842
By Simpson’s 1/3 rule
Z 4 
h    
f (x) ≈ f (xo ) + f (x6 ) + 2 f (x2 ) + f (x4 ) + 4 f (x1 ) + f (x3 ) + f (x5 )
1 3

56
 
1
= 421.59618 = 70.266
6
3. b)For a function f(x), given that
x 0 1/4 1/2 3/4 1
f (x) 8 a b 3 −4
where a and b areRreal numbers. Given that for four equal sub-intervals of [0,1]
1
(i) the integral of 0 f (x)dx value equal to 100, by using trapezoid rule.
R1
(ii)the integral of 0 f (x)dx value equal to 101, by using Simpson’s 1/3rd rule.
Then find the values of a and b.
Ans:
Z 1 By trapezoid rule,
hh i
f (x)dx = f (xo ) + 2f (x1 ) + 2f (x2 ) + 2f (x3 ) + 2f (x4 )
0 2
1h i
100 = 8 + 2a + 6 + 2b − 4
8
a + b = 315 . . . . . . . . . . . . . . .(i)
By Simpson’s 1/3rd rule,
Z 1
hh i
f (x)dx = f (xo ) + f (x4 ) + 2f (x2 ) + 4f (x1 ) + 4f (x3 )
0 2
2a + b = 598 . . . . . . . . . . . . . (ii)
⇒ from (i) and (ii)
a = 203
b = 192
Z 2
4. a) For the improper integral
2 + sin(πx)
e
dx,
ki
0 (1 − x)p
oo
(i) find all possible values of p such that the integral converges.
(ii) find all possible values of p such that the Zintegral diverges.
pC


t3
b) Using Beta function evaluate the integral dx
0 (1 + t)7
ar

Ans 4a): If comparison test is used


Sh

1
Take f (x) = and show that,
(1 − x)p
Z 1
1 ∞, p ≥ 1

p
dx = 1
0 (1 − x) 
 , p<1
1−p
2 + sin(πx) 1
0< p
≤ 0≤x<1
(1 − x) (1 − x)p
for p < 1 integral converges
1 2 + sin(πx)
0< ≤ 0≤x<1
(1 − x)p (1 − x)p
for p ≥ 1, integral diverges.
Z ∞ Z ∞
t3 t4−1
Ans 4b): =
0 (1 + t)7 0 (1 + t)4+3 h
= β(4, 3) β(m, n)
1 Γ(m)Γ(n) i
= =
60 Γ(m + n)
5. a) Using double integration find the volume of the solid bounded above the surface z = xy
and bounded below by the region enclosed by y = 4 − x2 , x = 1, x = 2 and the x-axis.(sketch and
shade the region of integration)

57
Z Z
b) Use the transformation x = u − v, y = u + v to evaluate (x + y)dA where R is the region
enclosed by y = x, y = 3x, x + y = 4. Z 2α
 dI 
c) Using Lebnittz Rule, find , where I = αxdx
dα α=1 α

Ans 5a):

Z 2 Z 4−x2
V = xydydx
x=1 y=0
On solving the integral we get, V=9/4.
Ans 5b):

e
ki
oo
pC
ar
Sh

y=x ⇒u+v =u−v ⇒v=0


y = 3x ⇒ u + v = 3(u − v) ⇒ v = u/2
x+y =4 ⇒u+v+u−v =4 ⇒u=2
∂(x, y)
=2
∂(u, v)
Z Z Z 2 Z u/2
16
⇒ (x + y)dA = 4u.du.dv =
0 0 3
Ans 5c): Lebnitz rule states that,
Z b
dI ∂f ∂b ∂a
= dx + f (b, α) − f (a, α)
dαZ a ∂x ∂α ∂α

⇒ x.dx + 2(2α2 ) − α2
α
α2 i2α
= + 3α2
2 α
9
= α2
2
dI 9
|α=1 =
dα 2
58
Sh
ar
pC
oo
ki
e

59
Z Z
7. a) Integrate xz 2 dS in the region bounded by the cylinder y = 2x2 + 1 where x varies from
0 to 2 and z varies from 4 to 8.
b) Find the directional derivative of f (x, y, z) = x2 yz + 4xz 2 at the point (1,-2,-1) in the direction
of the maximum rate of change.

Ans 7a):

e
ki
Z Z
xz 2 dS
oo
Given that
Z Z S Z Z
pC

G(x, y, z)dS = G(x, f (x, z), z)


S pR
= 1 + yx2 + yz2
ar

fx (x, z) = 4x and fz (x, z) = 0


Sh

Z Z Z 2 Z 8 p
2
xz dS = xz 2 1 + (4x)2 dxdz
S x=0Z z=4Z
28  3/2
xz 2 dS =

On solving we get 65 − 1 = 1627.3
S 9
2
Ans 7b): f (x, y, z) = x yz + 4xz 2

for directional derivative 



− ∂ ∂ ∂
∇f = î + ĵ + k̂ .f (x, y, z)
∂x ∂y ∂z
= (2xyz + 4z 2 )î + x2 z ĵ + (x2 y + 8zx)k̂


∇f |(1,−2,−1) = 8î − ĵ − 10k̂


So max change occurs along ∇f


∇f 8î − ĵ − 10k̂
n̂ = → − = √
| ∇f | 165
∴ directional derivative at (1,-2,-1)
8î − ĵ − 10k̂ √
Df |(1,−2,−1) = (8î − ĵ − 10k̂). √ = 165
165
8. a) Using Green’s Theorem evaluate c (x2 − y 2 )dx − (2y − x)dy, where C consists of the boundary
H

of the region in the first quadrant that is bounded by y = x2 , y = x3 .




b) Consider the force field F = (2xy + z 3 )î + x2 ĵ + xz 2 k̂


(i) Verify wether f is a conservative force field or not. If so, then find the potential function of


F.

60


(ii) Find the work done to move an object in this field F from (1,-2,1) to (3,1,4).

Ans 8a):

I
Given , (x2 − y 2 )dx + (2y − x)dy
C
Let P (x, y) = x2 − y 2 e
ki
Q(x, y) = 2y − x
According to Green’s Theorem,
oo

I Z Z 
∂Q ∂P 
P dx + Qdy = − dA
pC

Z Z R ∂x ∂y
⇒ (2y − 1)dA
ar

ZRx=1 Z y=x2
Sh

⇒ (2y − 1)dy.dx
x=0 y=x3
On solving,
11
=−
420 →
− → −
Ans 8b(i):  Force can be conservative
 if ∇ × F = 0 . . .[Necessary condition]
i j k

− → −  ∂ ∂ ∂ 
∇×F =  ∂x

∂y ∂z 
2xy + z 2 x2 3xz 2
= −î(0) − ĵ(3z 2 − 3z 2 ) + k̂(2x − 2x) = 0


Thus F is a conservative force.
Now on comparing, we get,
∂φ ∂φ ∂φ
= 2xy + z 3 , = x2 , = 3xz 2 . . . . . . (1),(2),(3) respectively
∂x ∂y ∂z
On Integrating we get,
φ = x2 y + xz 3 + f (y, z)
Substituting φ in (2) from (4) we get,
∂f
=0
∂y
⇒ f (y, z) = h(z)
Substituting in eq (3) we get,
∂h
=0
∂z
⇒ h(z) = c

61
Thus φ = x2 y + xz 3 + c is the potential function.

Ans 8b(ii) : We know that,


Z P2


work done = F dx
P1
Z P2
= (2xy + z 3 )dx + x2 dy + 3xz 2 dz
P1
= 202 units of work.

e
ki
oo
pC
ar
Sh

62
MATHS II
END-SPRING SEMESTER 2017

e
ki
oo
pC
ar

Ans 1a): T: R5 −→ R2
N(T) = [(x1 , x2 , x3 , x4 , x5 ) ∈ R5 : x1 = 2x2 , x3 = x4 = x5 ]
Sh

Let x1= α and


 x 3 =β  = x4 = x5 , x2 = α/2
x1 1 0
x2  1/2 0
     
∴ x3  = α  0  + β 1
    
x4   0  1
x5 0 1
Thus the dimension of N(T)= 2 as it has 2 basis (1,1/2,0,0,0) and (0,0,1,1,1).
∴ by rank-nulity theorem,
dim N(T) + dim R(T) = 5
⇒ dim R(T) = 3
but for the linear transformation T: R5 −→ R2
dim R(T) ≤ 2
So the linear transformation is not possible.
[remember dim R(T) ≤ 2 and not necessarily equal to 2 from T:R5 −→ R2 ]

Ans 1b): T: P2 (R) −→ P2 (R)


β = {1, x, x2 }
T (f (x)) = f (1) + f 0 (0)x + (f 0 (0) + f 00 (0))x2
when f (x) = 1
T (1) = 1 + 0.x + 0.x2
When f (x) = x,
T (x) = 1 + 1.x + (1 + 0)x2
T (x) = 1 = 1.x + 1.x2

63
when f (x) = x2 ,
T (x2 ) = 1 + 0.x + 2x2
∴ matrix  of linear
 transformation is
1 1 1
[T]ββ = 0 1 0
0 1 2
 
1 0 −1
Ans 1c): A = 1 2 1 
2 2 3
|A − λI| = 0
1 − λ 0 −1

⇒ 1 2−λ 1 = 0
2 2 3 − λ
⇒ (1 − λ)((2 − λ)(3 − λ) − 2) + (−1)(2 − 2(2 − λ)) = 0
⇒ λ3 − 6λ2 + 5λ − 6 = 0
⇒ (λ − 1)(λ − 2)(λ − 3) = 0
⇒ λ = 1, 2, 3
The eigen values are 1 , 2 , 3 .
for λ = 1,    
0 0 −1 x 0
⇒ 1 1 1  y  = 0
2 2 2 z 0
R3 → R3 − 2R2 and then
   R1 ↔ R2
1 1 1 x 0
e
ki
⇒ 0 0 −1 y  = 0
0 0 0 z 0
oo

1 free variable so assuming y = α


pC

⇒ −1.z = 0 1.x + 1.y + 1.z = 0


⇒z =  0   ⇒ x = −z − y ⇒ x = −α
ar

x −1
Sh

∴ y  = α  1 
z 0
Similarly eigen vectors for  λ = 2 and λ = 3 can be found,
−1
Eigen vectors for λ = 1,  1 
0
 
2
Eigen vectors for λ = 2,  −1
−2
 
1
Eigen vectors for λ = 3, −2
−2
 
1 2 1
P = −1 −1 −1
 [all the eigen vectors make up the coloumn space of P]
0 −2 −2
 
1 0 0
D = 0 2 0 [all the eigen values in diagonal]
0 0 3

64
e
ki
Z π/2 √
h
Ans 2a): cos θdθ = [(yo + y6 ) + 4(y1 + y3 + y5 ) + 2(y2 + y4 )]
oo

0 3
where,
pC

x 0 π/12 π/6 π/4 π/3 5π/12 π/2


y 1 0.9828 0.9306 0.8409 0.7071 0.5087 0
ar

yo y1 y2 y3 y4 y5 y6
Sh

π
h=
12
Ans: 1.1873

Ans 2b): 1st iterations:


(1)
x1 = 1.14457831 ' 1.1446
(1)
x2 = 1.8459221501 ' 1.8459
(1)
x3 = 1.8206513055 ' 1.8206

2nd iterations:
(2)
x1 = 0.987680251 ' 0.9877
(2)
x2 = 1.41188022 ' 1.4119
(2)
x3 = 1.9547083 ' 1.9547

Ans 2c): Lagrange’s Interpolation formula


(x − x1 )(x − x2 ) (x − xo )(x − x2 ) (x − xo )(x − x1 )
y(x) = yo + y1 + y2
(xo )(xo − x2 ) (x1 − xo )(x1 − x2 ) (x2 − xo )(x2 − x1 )
x 5 1 21
where,
y 1 3 4
(8 − 14)(8 − 21) (8 − 5)(8 − 21) (8 − 5)(8 − 14) 295
y(8) = + (3) + (4) = ' 1.7559
(5 − 14)(4 − 21) × 1 (14 − 5)(14 − 21) (21 − 5)(21 − 14) 168

65
Ans 3a): For 0 < p < 3 choose q so that p < q < 3.
3
x q ln x ln x
Then, lim p = lim
x→∞ p x3 (1 + x−3 ) x→∞ 3 3
( − )√
x p q p 1 + x−3
If p < 0 or p ≥ 3 , then lim = +∞
If p ∈ (−∞, 0) or p ≥ 3 the integral diverges and 0 < p < 3 it converges.
e
ki
Z π/2
ln(1 + cos α cos x)
Ans 3b): I(α) = dx , where 0 < α < π
oo

0 cos x
Z π/2 Z 
sin α dx
pC

I 0 (α) = − type
0 1 + cos α cos x 1 + b cos X
ar

 π/2 
(−1) × sin α cos α + cos x 
Hence , I 0 (α) = √ cos−1 
Sh

1 + cos α cos x

1 − cos2 α 0
  
cos α + 1
= − cos−1 (cos α) − cos−1
cos α + 1
= −(α − 0)
α2
∴ I 0 (α) = −α ⇒ I(α) = − +c
2
π π 2
α = ⇒ I(α) = 0 , c =
2 8
π 2 α2
∴ I(α) = −
8 2
Ans 3c):
∂f ∂f ∂f

∂u ∂w ∂v

∂(f, g, h) ∂g ∂g ∂g
I= =
∂(u, w, v) ∂u ∂w ∂v
∂h ∂h ∂h

∂u ∂w ∂v


α 0 1

= 2y 1 β = 2uv + αw − αβv − α − γ
γ u w − 1

66
Z π/2
Ans 4a): We have β(m, n) = 2 sin2m−1 θ cos2n−1 θdθ ,
Z π/2 0
2
β(m, n) = 2m−1 sin2m−1 (2θ)dθ
2 0
1
= 2m−1 β(m, 1/2)
2
β(m, 1/2)
⇒ = 22m−1
β(m, m)
∴ α = 2m − 1
e
ki
In Qusetion 4(b) the evaluation of the integral after the transformation is not possible.
oo
Ans 4c):
pC
ar
Sh

Z 1 Z 3−x Z Z Z Z
3−y 3−y
4xe dydx = 4xe 4xe3−y dx.dy
dx.dy +
x=0 y=2x2 D1 D2
Z 3 Z 3−y Z 2 Z √y/2
= 4xe3−y dx.dy + 4xe3−y dx.dy
y=2 x=0 y=0 x=0
= (2e − 4) + (e3 − 3e)
= e3 − e − 4
≈ 13.36725509

67
Ans 5a):

e
ki
oo
pC

We have
Z 2 the Z required area as ,
x+2
ar

A= dy.dx
x=−1 y=x2
Sh

Z 2
= (x + 2 − x2 )dx
−12 2
x x3
= + 2x −
2 3 −1
= (2 + 4 − 8/3) − (1/2 − 2 + 1/3)
10 7 9
= + = = 4.5 sq.units
3 6 2
Ans 5b):

68
As shown in the figure, the surface is a portion of the cylinder x2 + z 2 = 4.
∴ the surface
s area is ,
 2  2

Z Z
∂z ∂y
S= 1+ + dA Here, z = 4 − x2
R s ∂x ∂x
 2
−x −x
Z Z
∂z (−2x)
= 1+ √ + 0dA ∴ = √ =√
4 − x2 ∂x 2 4 − x2 4 − x2
Z 4 RZ 1
2 ∂z
= √ dx.dy =0
y=0 x=0 4−x 2 ∂y
Z 4
=2 [sin−1 (x/2)]1x=0 dy
0
Z 4
π
=2 (π/6)dy = [y]y0
0 3
= 4π/3 = 4.1887 sq.units

Ans 5c):

e
ki
oo
pC
ar
Sh

The solid G and its projection R on the XY-plane are shown in the above fighure. the lower surface of the
solid is the place z = 1Z and
Z Zthe upped Z is the plane x + z = 5 ⇒ z = 5 − x
Z Zsurface 5−x
∴ Volume of G = dv = dz dA
G R 1
For the double integral over√ R we integrate w.r.t. y first
Z 3 Z 9−x2 Z 5−x
∴ Volume of G = √ dz.dy.dx
x=−3 √y=− 9−x2 1
Z 3 Z 9−x2
= √ [z]5−x
z=1 dy.dx
−3 −√ 9−x 2
Z 3 Z 9−x2
= √ (4 − x)dy.dx
Z−3 − 9−x2
3 √
9−x2
= (4 − x)[y]−√ 9−x2
dx
Z−3
3 p
= (8 − 2x) 9 − x2 dx
−3
Z 3p Z 3 p
=8 2
9 − x dx − 2x 9 − x2 dx
−3 −3

69
  Z 3 p
9
=8 π − 2x 9 − x2 dx
2 −3

= 0 since the integrand is an odd function
= 36π cubic units = 113.0973 cubic units


î ĵ k̂

− −
Ans 6a): A × →

r = A1 A2 A3 = (zA2 − yA3 )î + (xA3 − zA1 )ĵ + (yA1 − xA2 )k̂
x y z
ˆ Â × r̂) = ∂ (z) + ∂ (xA3 − zA1 ) + ∂ (yA1 − xA2 )
∇.(
∂x ∂y ∂z
∂A2 ∂A3 ∂A3 ∂A1 ∂A1 ∂A2
=z −y +x −z +y −x
∂x ∂x  ∂y  ∂y ∂z  ∂z 
∂A3 ∂A2 ∂A1 ∂A3 ∂A2 ∂A1
=x − +y − +z e −
ki
∂y ∂z  ∂z  ∂x  ∂x  ∂y   
oo
∂A3 ∂A2 ∂A1 ∂A3 ∂A2 ∂A1
= (xî + y ĵ + z k̂) − î + − ĵ + − k̂
∂y ∂z ∂z ∂x ∂x ∂y
pC


− → − →

= r̂.( ∇ × A ) = → −r.0 =0
ar



Ans 6b): ∇φ = [2ax − (a + 2)]î − bz ĵ − by k̂
Sh



∇ψ = 8xy î + 4x2 ĵ + 3z 2 k̂


( ∇φ)1,−1,2 = [2a − (a + 2)]î − zbĵ + bk̂


( ∇ψ)1,−1,2 = −8î + 4ĵ + 12k̂
∵ φ = 0, ψ = 0 are orthogonal at (1,-1,2).

− →

( ∇φ).( ∇ψ) = 0 at (1,-1,2)
⇒ 2a = b + 4 −→(1)
∵ ax2 − byz = (a + 2)x passes through (1,-1,2) , a + 2b = a + 2 ⇒ b = 1
5
∴ From (1), a =
2

70
Ans 7a): ∇f = 2xy 3 z 4 î + 3x2 y 2 z 4 ĵ + 4x2 y 3 z 3 k̂
= 4î + 12ĵ + 16k̂ at (2,1,1)
î + 2ĵ + 2k̂ 60
Directional derivative is ∇f.b̂ = (4î + 12ĵ + 16k̂). √ = = 20
9 3
Ans 7b): curl F = 0
now, F = ∇φ
∂φ ∂φ ∂φ
2xyz 2 î + [x2 z 2 + z cos(yz)]ĵ + [2x2 yz + y cos(yz)]k̂ = ( î + ĵ + k̂)
∂x ∂y ∂z
∂φ
= 2xyz 2 φ = x2 yz 2 + f1 (y, z)
∂x
∂φ ∂φ ∂f1
= x2 z 2 + z cos(yz) = x2 z 2 +
∂y ∂y ∂y
∂φ e
ki
2
= 2x yz + y cos(yz)
∂z
oo

∂f1
∴ = z cos(yz)
∂y
pC

∴ f1 = sin(yz) + f2 (z)
∴ φ = x2 yz 2 + sin(yz) + f2 (z)
ar

∂φ ∂f2
∴ = 2x2 yz + y cos(yz) +
Sh

∂z ∂z
∂f2
∴ =0
∂z
∴ f2 = c
φ = x2 yz 2 + sin(yz) + c
(1,π/4,2)
dφ = [x2 yz 2 + sin(yz)](0,0,1) = π + 1
R R R
c F.dr = c ∇φ.dr =

Ans
Z Z7c):
 By Green’s
 theorem , given line integral
∂f2 ∂f1
= − dx.dy
R ∂x ∂y

Z Z
= (−6y + 16y)dx.dy
ZR1 Z 1−x 
= 10 ydy dx
0 0

71
Z 1  2 1−x Z 1
y 5
= 10 dx = 5 (1 − x)2 dx =
0 2 0 0 3
Along
Z OA,
Z 1 y = 0, dy = 0, x varies from 0 to 1.
= 3x2 dx = 1
OA 0
Z 0 y = 1 − x, dy = −dx, Zx 0varies from 1 to 0.
Along AB,
Z
= [3x2 − 8(1 − x)]2 dx + [4(1 − x) − 6x(1 − x)(−1)]dx
AB Z1 0 1
8
= (−11x2 + 26x − 12)dx =
1 3
Along
Z BO,
Z 0 x = 0, dx = 0, y varies from 1 to 0
= 4ydy = −2
BO 1
8 5
∴ φc (f1 dx + f2 dy) = 1 + −2=
3 3

e
ki
oo
pC
ar
Sh

72
MATHS II
END SEMESTER EXAMINATION 2016
1) (a) Let T : R5 −→ R3 be a linear mapping defined by T (x, y, z) = (x + z, −x + 2y + z, y + z). Taking the
usual basis of R3 i.e. (1,0,0) , (0,1,0) , (0,0,1) find the basis and dimension of the Range space and Null space.
(b) For −1
 the given
 matrix A, find its characteristic equation and findA using Caley-Hamiltaon’s theorem.
1 0 2
A = 0 2 1

2 0 3

Ans 1a): T : R3 → R3 be a linear map


T (e1 ) = (−1, −1, 0)
T (e2 ) = (0, 2, 1)
T (e3 ) = (1, 1, 1)
R3 = span(e1 , e2 , e3 )
R(T ) = span(T (e1 ), T (e2 ), T (e3 ))
= span((1, −1, 0), (0, 2, 1), (1, 1, 1))
= span((1, −1, 0), (0, 2, 1))
since(1, 1, 1) = (1, −1, 0) + (0, 2, 1)
hence, R(T ) has a dimention 2 since it has 2 vectors as basis.
N (T ) = (x, y, z)/T (x, y, z) = (0, 0, 0) 
e
= (x, y, z) | (x + z,−x + 2y + z, y + z) = (0, 0, 0)
ki
= (x, x, −x)|x ∈ R
oo

= span (1, 1, −1)


1, 1, −1 is the basis of N (T ) hence dimension is 1.
pC

 
1 0 2
1b): A = 0 2 1
ar

2 0 3
Sh

|A − λI| = 0⇔ λ3 − 6λ 2
 + 7λ + 2 = 0
5 0 8
Now, A2 = 2 4 5 
8 0 13
 
21 0 34
A3 = 12 8 23
34 0 55
Hence, A3 − 6A2 + 7A + 2I = 0
A3 − 6A2 + 7A = −2I
Multiplying A−1 on both sides
⇒ A2 − 6A +h7I = −2A−1 i
1
⇒ A−1 = − A2 − 6A + 7I
2 
−3 0 2
= −1 1/2 1/2
2 0 −1
2. (a) From the table given below construct Newton Forward difference table and estimate the
value of f (x) at x=0.15
x 0.1 0.2 0.3 0.4 0.5
f (x) 1.4 1.56 1.76 2.00 2.28
Z b
(b) In the interval [a,b] apply Simpson’s 1/3 rd rule to estimate the integral f (t)dt for two
a
intervals ∈ [a,b] where

73
Z b   
14000
f (t) = 200 ln − 9.8t
a 14000 − 2100t
(c) Using the expression for estimating the integral in 2(b). Find the valuie of the integral
for a=30 , b=8.
Ans 2a): (i) Forward Difference method
x f (x) 4f 42 f 43 f 44 f
0.1 1.4
0.16
0.2 1.56 0.04
0.20 0
0.3 1.76 0.04 0
0.24 0
0.4 2.00 0.04
0.28
0.5 2.28
s(s − 1) 2
(ii) f = fo + s 4 f + 4 f
2
x = 0.15 , h = 0.1
x − xo 0.15 − 0.1
since, s = = = 0.5
h 0.1
(0.5)(0.5 − 1)
f = 1.4 + 0.5 × 0.16 + × 0.04
2
= 1.4795

b − ah a+b i
2b):
Rb
a f (t).δt = f (a) + 4f ( ) + f (b) e
ki
6 2
oo

2c): f (8) = 177.27, f (19) = 424.75, f (30) = 901.67


22 h i
pC

A= f (8) + 4f (19) + f (30) = 11065.78


6
R 30  h 140000 i 
where A = 8 2000ln − 9.8t δt
ar

140000 − 2100t
Sh

3. (a) Show whether:


Z 2
ln x
(i) √ dx converges or diverges.
Z0 ∞ x
dx
(ii) √ converges or diverges.
2 x2 − 1
(b) Using Lebnitz rule of differentiation under integral evaluate
Z π/2
dx
0 a2 cos2 x+ b2 sin2 x

ln(x) ln(x)
Ans 3a): lim (x − 0)3/4 1/2
= lim −1/4 = 0
x→0+ x x→0 x
+
R 2 ln(x)
as µ = 3/4 < 1, 0
√ converges . . . . . . . [reffer to convergence via µ method]
Z ∞ x
δx
(ii) √
2 x2 − 1
1 x
lim x × √ = lim p = 1 6= 0
x→∞ x2−1 x→∞ x 1 − 1/x2
Z ∞
dx
i.e. √ diverges.
2 x2 − 1
Z π/2
R π/2 dx sec2 (x).dx
3b): I1 = 0 =
a2 cos2 (x) + b2 sin2 (x) 0 a2 + b2 tan2 (x)
74
Let tan(x) = t; x=0⇒t=0
Z ∞ x= Z π/2 ⇒ t = ∞
dt 1 ∞ dt π
I= 2 2 2
= 2 2 2 2
=
0 a +b t b 0 t + a /b 2ab
Z π/2 Z π/2
∂ dx −2acos2 (x).dx
=
∂a 0 a2 cos2 (x) + b2 sin2 (x) 0 (a2 cos2 (x) + b2 sin2 (x))2
Z π/2 2
cos (x) π
i.e. 2 2 2 2 2
= 3
0 (a cos (x) + b sin (x)) 4a b
Z π/2 2
sin (x)
Similarly, 2 cos2 (x) + b2 sin2 (x))2
(a
Z 0π/2
sin2 ( π2 − x)
0 (a2 sin2 (x) + b2 cos2 (x))2
π
= 3
Z π/24ab
dx π 1 1
Hence, 2 2 2 2 2
= ( 2 + 2)
0 (a cos (x) + b sin (x)) 4ab a b
π(a2 + b2 )
=
4a3 b3

1
(1 − x4 )3/4
Z
4. (a) Find the value of the integral dx
0 (1 + x4 )2
β(m + 1, n)
(b) Find the value of
Z 1 Z √2−x2
β(m, n) e
ki
x
(c) Evaluate dy.dx by changing the order of integration.
oo
p
x=0 y=x x + y2
2
Z 1
(1 − x4 )3/4
pC

Ans 4a): Finding the value of the integral dx in terms of β function.


0 (1 + x4 )2
ar

1 − x4
Let z =
Sh

4
Z 1 1 + x 4 3/4 Z 1
(1 − x ) 1
then 4 )2
dx = 9/4 z 3/4 (1 − z)−3/4 dz
0 (1 + x 2 0
Z 1
Clearly of the form; xm−1 (1 − x)n−1
0
1 1 7
Hence I = 9/4 β ,
2 4 4
m
Ans 4b):
m+n
Ans 4c):
On changing √ order of Integration,
R1 : 1 ≤ y ≤ p2
0 ≤ x ≤ 2 − y2
R2 : 0 ≤ y ≤ 1
0≤x≤y
Z Z Z √2 Z √2−y2
x x
I1 = p .dxdy = p .dxdy
R1 2
x +y 2 y=1 x=0 x + y2
2

Z 2 √
= ( 2 − y).dy
y=1 √
3−2 2
=
Z Z Z 1 2Z y
x x
I2 = p .dxdy = p .dxdy
R2 2
x +y 2 y=0 x=0 x + y2
2

75

2−1
=
Z Z 2
x
p .dxdy = I1 + I2
R x + y2
2

2−1
=
2
5 (a) Use double integral to find the area of the region bounded by the parabola y = 4x2 and the line y = 6x−2
e
ki
(b) Calculate the volume of the solid which is the part of the right circular cylinder x2 + y 2 = 25 lying in the
oo
first octant between z = 0 and z = 3 using triple integral in cartesian co-ordinates. Also find the same volume
using cylindrical co-ordinates.
pC

(c) Using double integral, find the surface area of the part of the plane 2x + 3y + z = 6 that lies in the first
octant.
ar
Sh

Ans 5a): y = x2 , y = 6x − 2
Points of intersection
⇒ 4x2 − 6x + 2 = 0
⇒ 2x2 − 3x + 1 = 0
x = 1/2 or x = 1
(1/2, 1) , (1,4) are the points of intersection.

Z Z
Area = dx.dy
R
Z 1 Z 6x−2
= dx.dy
1/2 4x2

76
1
=
12

Z 5Z 25−x2 Z 3
5b): V = dzdydx
0 0 0
5 √
Z
= 3 25 − xrdx
0

75π
=
4
5c):

Z Z s  ∂z 2  ∂z 2
Area = 1+ .dxdy
∂x ∂y

Z 3
6 − 2x e
ki
= 14 dydx
√ 0 3
oo

= 3 14
pC

6. (a) for the scalar function f = x2 + y 2 find the maximum change in the rate ie. the maximum value of
ar

directional derivative at the point (2,1,-1).




Sh

(b) Given that F = (x + 2y + αz)î + (βx − 3y − z)ĵ + (4x + γy + 2z)k̂ is irrational, find the values of α , β
and γ.

− →
− →
− → −
(c) Given that F and G are two irrational vector functions prove that F × G is solenoidal.

1 →
. −

Ans 6a): Directional derivative = → − a .∇f
|a|
where â is the direction
f = x2 yz 3


∇ f = 2xyz 3 î + x2 z 3 ĵ + 3x2 yz 2 k̂
→−
(∇ f )p = −4î − 4ĵ + 12k̂
Maximum in direction −4î − 4ĵ + 12k̂


6b): curl F = (γ + 1)î + (α − 4)ĵ + (β − 2)k̂

− → − →
− → −
6c): ∇ × F = 0, ∇×G =0

− →− → →
− →
− →
− →
− →
−
∇ F ×− y =→ −
 
y ∇×F −F ∇×G =0

− →− −
∴ ∇ F ×→ y =0
Hence solenoidal.

7. (a) Verification of Green’s theorem for φ(x2 + y 2 )dx + (x2 − y 2 )dy.




(b) F (x, y, z) = (6x + 2y)î + 2xĵ + k̂

77

− →
− →

(i) To check whether F is conservative. If it is then find f : F = ∇f


(ii) Find work done by F in moving a particle from (1,0,0) to (0,0,2).

Ans 7):

Line integral = φC1 . . . . . + φC2 . . . .


φC1 = 8/3 φC2 = −32/5
So φC = φC1 + φC2 = −56/15
∂f ∂
= 2x + f (y, z)
∂y ∂y
e
ki
∂f
= 6x + 2y
∂x
oo


f1 (y, z) = 0
pC

∂y
So f (x, y, z) = 3x2 + 2xy + f2 (z)
ar

∂f ∂fz
Sh

= =1
∂z ∂z
⇒ fz (z) = z + c
f (x, y, z) = 3x2 + 2xy + z + c
Z (0,0,2)
work done = F.dx
(1,0,0)
Z (0,0,2)
= df
(1,0,0)
= −1

78
PHYSICS
MID-SPRING SEMESTER EXAMINATION 2017-2018
1 1
1 a. A thin circular disc of mass kg and radius 2 m suspended by an elastic
π π
thread performs small torsional oscillations in a liquid. The moment of
12.5
elastic force emerging in the thread is τ = 3 θ N-m, where θ is the angle
π
of rotation from the equilibrium position. The resistance force acting on
a unit area of the disc is equal to P = 3π 3 ν N/m2 where the velocity, ν,
in m/s, of a given element of the disc relative to the liquid. Find the
frequency of the small oscillations.
Ans:

e
ki
oo

Torque due to resistive force = (3π 3 ν × 2πrδr) × r ....... [the term force×area×radius]
pC

=3π 3 rθ × 2πr2 δr = 6π 4 r3 δr = τres


125
ar

Moment due to oscilating force (the thread) = 3 θ


π
Sh

r4
τres = 6π 4 θ × 2 = 3π 4 θR4
4
....... [the 2 factor is multiplied as forces act on both upper and lower surfaces]

125
τossc = θ
π3
Equation of Motion:
mR2 12.5
θ̈ + 3 θ + 3π 4 R4 θ̇ = 0
2 π
1 1
Now R = 2
and m =
π π
1 12.5 3π 4
θ̈ + θ + θ̇ = 0
2π 3 π3 π8

θ̈ + 25π 2 θ + 6π θ̇ = 0
Comparing with: ẍ + 2β ẋ + ωo2 x = 0
we get, ωpo = 5π, β = 3π
⇒ ω 0 = ωo2 − β 2
T = 2π/ω 0 = (1/2)s. Thus ν = 2Hz

79
1 b). Show the regions of undamped, underdamped, critically damped and overdamped oscilla-
tions in the k-r plane, where the restoring force, Fs = kr and the resistive force , Fd = −rẋ.

Ans: Equation of motion: mẍ + rẋ + kx = 0 . . . . . . ωo = k/m


0
p p
⇒ ω = ωo − β = k/m − r2 /4
2 2

Now for overdamped, ω 0 is complex . . . .[Please refer theory for complete explanation]
k
⇒ r2 > 4
m
Now for underdamped, ω 0 isd real
k
⇒ r2 < 4
m
k
Now for critically damped, r2 = 4
m

e
ki
oo
pC
ar

2. A mass, m, attached to the middle point of a stretched string of length, 2b, and tension,
Sh

T, rests on a frictionless horizontal plane. One end of the string is fixed while the other end has
a periodic motion of small amplitude, a, at right angles to the string in the horizontal plane such
that the displacement, y, at time, t, is given by y(t) = asin(ωt) (sce Fig below). The displacement of
the mass at time, t, is c(t). Assume T to be constant during the motion neglecting the effects of
x2 and a2 . Find the initial displacement x(0) and the velocity ẋ(O) such that the resulting motion
x(t) of the mass is purely forced (that is there is no component of free oscillations)

Ans. Analyzing the Free Body Diagram

T cosθ = T cosφ . . .and . . . mẍ = T sinθ − T sinφ


⇒ mo ẍ = T (θ − φ) . . . . . . . Taking θ ∼ φ ∼ 0

80
T x y−x
⇒ mo ẍ = (y − 2x) . . . . . . . . As φ = andθ =
b b b
T 2T Ta
⇒ mo ẍ = (asin(ωt) − 2x) ⇒ mo ẍ + ( )x = sin(ωt)
b b b
2T Ta
⇒ ẍ + ( )x = sin(ωt)
mo b mo b
fo
ON solving the differential equation we get x(t) = 2 sin(ωt) + Csin(ωo t) + Dcos(ωo t)
ωo − ω 2
Ta 2T
where fo = and ωo =
mo b bm
fo
Neglecting the transient part ⇒ x(t) = 2 sin(ωt) . . . . . i.e C = D = 0
ωo − ω 2
ωaT ωaT
⇒ x(0) = 0 ; ẋ(0) = 2 =
(ωo − ω 2 )mb 2T − ω 2 mb

Q 3 a) A coupled LC circuit is shown in Fig. below. The inductors have the same self inductance
Ll = L2 = 2.5µF and the mutual inductance between them is M 1.5mH. The capacitances C1 = C2 =
2.5µF . Q1 and Q2 arc thc instantaneous charges on the capacitors Cl and C2 respectively at any
instant. Set up the equations for Q1 and Q2 . Calculate the normal mode frequencies for this circuit.
What would be the directions of the currents in the circuit in these two modes?

e
ki
oo

Ans. Writing the electrical equations, L1 q¨1 + M q¨2 + q1 /C1 = 0 . . . . . . (1)


pC

where, L1 q¨1 = L1 I1 (self-inductance) q2 M = M I2 (mutual-inductance)


Similarly, L2 q¨2 + M q¨2 + q2 /C2 = 0 . . . . . . (2)
ar

Now let qo0 = (q1 + q2 )/2 and q10 = (q1 − q2 )/2


Sh

Adding and subtracting (1) and (2)


(L + M )q¨o0 + qo /C = 0 and (L − M )q¨10 + q10 /C = 0
1 1
⇒ ωo2 = ⇒ ω12 =
C(L − M ) C(L + M )
⇒ ωo2 = 104 rad/s ⇒ ω12 = 2 × 104 rad/s

*Direction of winding is necessary to give exact answer.

3. b) Now suppose in the above problem the mutual inductance is varied from M = 0 to
M = 2.0mH. Plot the variation of normal mode frequencies as a function of M roughly. [3]
1 1
Ans: ωo2 = ω12 =
(L + M )C (L − M )C
Now ωo must decrease with increasing M, and conversely ω1 must increase with increasing M.
Also for M=0, ωo = ω1

81
4. a) A ’wave packet’ in a certain medium is represented by the following
ψ(x, t) = 4cos(5x204t)cos(2x72t)cos(x36t). Find the group velocity and phase velocity for this packet.

Ans: ψ(x, t) = 4cos(5x − 204t)cos(2x − 72t)cos(x − 36t)


= 2[cos(7x − 276t) + cos(3x − 132t)]cos(x − 36t)
e
= cos(8x − 312t) + cos(6x − 240t) + cos(4x − 168t) + cos(2x − 96t)
ki
ωavg = 204 rad/s kavg = 5 m−1
oo

⇒ vp = ωavg /kavg = 40.8 ms−1


312 − 96 216
= 36 ms−1
pC

⇒ vg = ∆ω/∆k = =
8−2 6
ar

4 b) Plot the phase velocity in the medium as a function of wave number k, near k = 5. Also
suggest a plausible dispersion relation near k = 5.
Sh

Ans: A possible dispersion relation:


ω 24
ω = 24 + 36k ⇒ = + 36 = vp
k k

Both side has asymtotic relations.

82
PHYSICS
MID - AUTUMN SEMESTER EXAMINATION2017-2018
1. For forced oscillation with a forceF = F0 cos wt of an oscillator of mass m and damping constant β ,
the amplitude of forced oscillation is given by .
F0 1
A= q
m (w2 − w2 )2 + 4w2 β 2
0 0

a. If damping constantβ of a driven system is 5s−1 and time period of natural oscillation T0 is 0.3sec find the
ratio of maximum amplitude and the amplitude at very low driving frequency(w << w0 )
b. if a system undergoing force oscillator has equal displacement amplitude at frequenciesw1 = 400rad/sandw2 =
600rad/s, find its resonance frequency.

ans.

F0 1 2π
1. a. A = q , Now T0 = 0.3secandβ = 5Hzandw0 = T0
m (w2 − w2 )2 + 4w2 β 2
0 0
Amax means condition of resonance , maximum energy implying maximum amplitude.
e
ki
q
F0
w02 − 2β 2 and ⇒ Amax = √
oo

⇒ w = wres = [ try to take these as formula]


w02 −β 2 (2mβ)
F0
At low frequency w << wres , the amplitude becomesAlow =
pC

k
F0 q 1
A = mw 2
w2 w β2
0 (1− )2 +4( w )2
ar

w2 0 w2
0 0

√k/m
Sh

⇒ Amax
Alow = √F0 2 × k
F0 = 2
≈ 2.157
2mβ w0 −β 2 2β w0 −β 2

F0
b. Given atw1 andw2 , the amplitude is same m
√ 1
= Fm0 √ 1
(w02 −w12 )2 +4w12 β 2 (w02 −w22 )2 +4w22 β 2

or (w02 − w12 )2 + 4w12 β 2 = (w02 − w22 )2 + 4w22 β 2 solving we get


w12 +w22
wres = w02 − 2β 2 = 2 ≈ 509.9rad/s

2. Consider a one dimensional system depicted in the figure where two blocks of equal mass m are with
springs whose opposite ends are fixed with respective walls. Two blocks are also inte- connected with a system
of spring
√ and velocity- damper. Each spring has spring constant k and the damping constant of the damper is
γ = km

a)Express the equations of motion for both the blocks.

b) Determine normal coordinates in terms of the displacements of the blocks , and find nature of the nor-
mal modes of oscillations.

c) Find the displacements of both the blocks as a function of time t=0 , one of the blocks is at rest in its

83
mean position and the other blockhas velocity v0 at the position x0 from its mean.

ans
a. Equation of motion mx¨1 = −kx1 − k(x1 − kx2 ) − r(x˙1 − x˙2 )
mx¨2 = −kx2 − k(x2 − kx1 ) − r(x˙2 − x˙1 )
T akingq0 = x1 + x2 andq1 = x1 − x2
we get q¨0 = −w12 q1 andq¨1 = −w12 q1 −2β q˙1 where q0 = x1 +x2 and q1 = x1 −x2 and w0 = k/m and w2 = 3k/m
p p

and β = r/m
where
√ q0 is in phase
p oscillation and q1 is out of phase oscillation , q1 is an underdamped oscillation
β = km/m = k/m ⇒ w1 > beta(underdamped)
q
⇒ x1 (t) = Acos(w0 t + φ1 ) + Be−βt cos( w22 − β 2 (t) + φ2 )
q
x2 (t) = Acos(w0 t + φ1 ) − Be−βt cos( w22 − β 2 (t) + φ2 )
Imposing conditions x1 (0) = x0 , x2 (0) =q0, x˙1 (0) = v0 , x˙2 (0) = 0
q0 = Acos(w1 t + φ1 )andq1 = Be−βt cos( w22 − β 2 (t) + φ2 ) e
ki
q0 (0) = x0 q1 (0) = 0 q˙0 (0) = v0 q˙1 (0) = v0
oo

use this conditions to find q andx1 and x2


pC
ar

q
v0 + βx0 q
x1 (t) = 1/2[x0 cos(w1 t)] + e−βt /2[x0 cos(t w02 − β 2 + q sin(t w22 − β 2
Sh

w02 − β 2

−βt
q
v0 + βx0 q
x2 (t) = 1/2[x0 cos(w1 t)] + e /2[x0 cos(t w02 − β2 +q sin(t w22 − β 2
w02 − β 2

84
ANSWER:


− −
→ →
−→ −
[ Compare with E = E0 ei(wt− k x ) where→

x = xi+yj +zk] c. For transverse waves, the direction of propagation

is perpendicular to the direction of vibration e


ki

− →
− →

⇒ show → −
n.k = → −n . B = 0 where n̂ is the direction of electric field B is itself the magnetic field
oo

4. a) Find equation of motion corresponding to the Lagarangian


pC

1
L = ẋ2 − 10x2 + x[cos4t]
2
ar

d δL δL

Sh

using Euler Lagarangian equation dt ( δ ẋ ) δx =0

b)Show that the steady state solution can be written as x = 14 ei4t

ANS:
L = 21 ẋ2 − 10x2 + x[cos4t]
⇒ δL δL
δ ẋ = ẋ δx = −20x + cos4t and
d δL
dt ( δ ẋ ) = ẍ
⇒ using Euler Lagarangian form
d δL δL
( )− =0
dt δ ẋ δx

we get ẍ + 20x = cos4t which is the solution of a forced harmonic oscillation .


comparing with 4mẍ + w02 x = F0 coswt
we get m = 1, w02 = 20, F0 = 1, w = 4

Solving differential equation , we get x(t) = Asinw0 + Bcosw0 t + wF20−w /m iwt


2e Note thet the particular solu-
0
tion contains a complex part but x(t) must be real function of t. But we can take the complex part as the
solution.
Neglecting transient parts x(t) = wF20−w
/m iwt
2e = 14 ei4t [substitute requiured values]
0

85
PHYSICS
MID SEMESTER SPRING EXAMINATION 2016-17

Ans 1): m1 x¨1 + k(x1 − x2 ) = 0


mx¨2 + k(x2 − x1 ) = 0

e
ki
oo
pC
ar
Sh

NOTE: The question


r is unclear. Please consult your professor.
k(m1 + m2 )
One of the ω =
m1 m2

Ans 2): ∴ mx¨1 + kx1 + k(x1 − x2 ) = 0


ans mx¨2 + k(x2 − x1 ) = 0
Put x1 = A1 sin ωt and x2 = A2 sin ωt
∴ −ω 2 mx1 + 2kx1 − kx2 = 0
∴ −ω 2 mx 2 + kx22 − kx1 = 0
2k − ω m −k
Solving =0
−k k − ω 2 m

86

(± 5 − 3)
∴ω= k
2

e
ki
ω 1800
Ans 3(i): vp = = ≈ 340ms−1
k 5.3
oo

ε = A cos(ωt − kx)
Ans 3(ii): Displacement amplitude = A = 60µm
pC


λ= ≈ 1.186m
3.3
ar

∴ ratio = 5.059×10−5
Sh

ε1 − ε2
Ans 4): Given = 0.01 [1%]
ε1
or ε1 − ε2 = 0.01ε1
or 0.99ε1 = ε2
ε2
or = 0.99
ε1
Now let there be one point O, then there must be another point where this condition satisfies.
∴ ε1 = ae−γ(O) [ All are
∴ ε2 = ae−γ(x) continious functions]
And required values = φ = k(x − O) = kx[(ωt − k(O)) − (ωt − k(x))]
φ γ
−γ( ) −( )φ
∴ 0.99 = e R cos φ = e R

87
PHYSICS
END-AUTUMN SEMESTER EXAMINATION 2017-2018
1. In a double-slits experimental set-up for observing Fraunhoffer diffraction and interference
patterns, the width of each slit is b and the separation between the mid-points of two slits is a.
Consider normal incidence of monochromatic light of wavelength from a distant source on the slits.
The first order diffraction minima occur at = /6, where is the angle of observation making with an
axis perpendicular to the slits and passing through the mid-point of separation between two slits.
(a) Find if b = 1600 nm.
(b) If a/b = 5, determine the value of sin for all the constructive interference patterns within
the principal maxima.
πbsinθ
Ans a) : The first minimum occurs at β = ±π where β =
λ
πbsinθ
⇒ ±π = ⇒ bsinθ = ±λ
λ
1
Now for θ = ±π/6 , ⇒ b × = λ = 800 nm = 0.6µm
2
πbsinθ πdsinθ
Ans b) : β = and γ =
λ λ
where b and d are slit-width and period respectively.
Now d/b = 5 means that the 5th maxima disappears.
Thus 4 maximas are present.
Now for maxima γ = mπ ⇒ dsinθ = mλ
λ e
λ
ki
⇒ sinθ = m( ) = m( )
d 5b
oo

800 m
⇒ sinθ = m( )=( )
5 × 1000 10
pC

1 2 3 4
⇒ sinθ = 0, ± , ± , ± , ±
10 10 10 10
ar
Sh

2. In an infinite square well potential given by


(
0 f or 0 ≤ x ≤ L
f (n) =
∞ f or x > L and x < 0

the quantum state of a particle of mass m is described by an initial wave function



3 1
Ψ(x, 0) = φ1 (x) + φ2 (x)
2 2
where φn (x) (n ≥ 1) are normalized eigenstates of Hamiltonian corresponding to this potential with
En .
(a) What is the expectation value of Hamiltonian in the state Ψ(x, 0)
(b) Determine Ψ(x, t) at time t.
R ∞ ∗ Calculate expectation value of momentum in the above state Ψ(x, t) at time t, i.e. hpi =
(c)
−∞ Ψ (x, t)pΨ(x, t)δx
Ans a) : We know Hψ = Eψ
c2 E1 + c22 E2 3 1
hEi = 1 2 2 = E1 + E2
c1 + c2 4 4
3 4 7 h2 7h2 n2 h2
= ( + )E1 = × = [where En = ]
4 4 4 8mL2 32mL2 8mL2
√ −iE1 −iE2
3 z 1 z
Ans b) : Ψ(x, t) = φ1 (x)e ~ + φ2 (x) ~
2 2

88
r
2 nπx n 2 h2
where φn = sin( ) and En =
L L 8mL2

Ans c) : p̂ = −i~
∂x
RL ∂
⇒ hpi = 0 δxΨ(x, t)(−i~ )φ(x, t)
∂x
√ i(E2 − E1 ) √ −i(E2 − E1 )
RL 2π 2πx πx 3 t 2πx πx 2 3 t
= 0 δx(−i~) 2 [sin( )cos( ) e ~ + cos( )sin( ) e ~ ]
L L L 2 L L 4
~ E2 − E1
= √ sin( t)
L 3 ~

e
ki
oo

Ans 3a) : From the knowledge of JEE


pC

yd
(µ − 1)t =
D
ar

tD
⇒ y = (µ − 1)
Sh

d

Ans 3b) : For 2nd dark fringe, path difference =
2
yd 3λ 3λD
⇒ = ⇒y=
D 2 2d
3λD tD
⇒ = (µ − 1)
2d d
2t 2
⇒ λ = (µ − 1) = × 1.5 × 10−6 × 0.5 = 0.5 µm
3 3

89

Ans 4a) : We know Eψ = −i~ ψ and Hψ = Eψ
∂t
∂ψ Hψ
⇒ =
∂t −i~
∂ψ i~ ∂ 2 ψ i
⇒ ψ∗ =( 2
− vψ) × ψ ∗ . . . . . . . . . . (1)
∂t 2m ∂x ~
∂ψ ∗ i~ ∂ 2 ψ ∗ i
and ψ =( 2
− vψ ∗ ) × ψ . . . . . . . . [Take compliment of (1)]
∂t 2m ∂x ~ " #
∂ ∗ ∂ ∂ ∂ψ ∂ψ ∗
Adding (ψ ψ) . . . . . . . . . . . . . . . |ψ|2 = (ψ ∗ ψ) = ψ ∗ +ψ
∂t ∂t ∂t ∂t ∂t
i~ h 2
∂ ψ 2
∂ ψ ∗ i
= ψ∗ 2 − ψ
2m ∂x ∂x2
i~ h ∂ ψ ∂ψ ∗ ∂ψ   ∂ 2 ψ ∗ ∂ψ ∗ ∂ψ i
2
= ψ∗ 2 + − ψ +
2m ∂x ∂x ∂x ∂x2 ∂x ∂x

=
i~ ∂ h
ψ∗
∂ψ
−ψ
∂ψ i
e
ki
2m ∂x ∂x ∂x
oo
Converting to 3-D co-ordinates
∂ 2 →
− → −
|ψ | = (− ∇ · J )
pC

∂x

−  → − →
−  i~
⇒ J = ψ( ∇ · ψ ∗ ) − ψ ∗ ( ∇.ψ)
ar

2m
i~
Sh

ψ(∇ · ψ ∗ ) − ψ ∗ (∇.ψ)
 
Ans 4b) : J =
2m
Now ψ = e (i/~)(Et − px x − py y − pz z)
and ψ ∗ = e(−i/~)(Et − px x − py y − pz z)
∂ i ipx
⇒ ψ ψ ∗ = + px ψψ ∗ =
∂x ~ ~
∗ i→

Similarly ψ(∇ψ ) = p .(i + j + k) . . . . . . . . . . . (1)
~
∗ −i →−  
and also ψ (∇ψ) = p .(i + j + k) Taking compliment of (1)
~
i~  i  −1
⇒J = (px + py + pz ) × 2 = (px + py + pz ) = −(vx + vy + vz ) = −v
2m ~ m
⇒ Jˆ = −v̂
Z ∞
1 2 ∗ 2
Ans 4c) : hxi = 2 e−αx xe−α x δx = 0 [Odd function]
Z ∞ N −∞
1
hxi = 2 ψ ∗ (x, t)xψ(x, t)δx = 0
N −∞
N = Normalization Constant [You can also perform the integration to get the same results]

90
Ans 5a): [L1 , L3 ] = [ypz − zpy , xpy − ypx ]
= [ypz , xpy ] − [ypz , ypx ] − [zpy , xpy ] + [zpy , ypx ]
= y[pz , xpy ] + [y, xpy ]pz
= y[pz , x]py + y[pz , py ] + x[y, py ]pz + [y, x]xpz
= 0 + 0 + i~xpz
Similarly [ypz , ypx ] = y[pz , ypx ] + [y, ypx ]pz
= y[pz , y]px + y 2 [pz , px ] + y[y, px ]pz + [y, y]px pz
=0
[zpy , ypx ] = −i~zpx
Similarly [zpy , xpy ] = 0 and
e
ki
⇒ [L1 , L3 ] = i~[xpz − zpx ] = −i~L2
Remembering the cyclic form Lx = ypz − zpy
oo

Ly = zpx − xpz
pC

Lz = xpy − ypx
⇒ [lx , Lz ] = −i~Ly
ar

~2 0 1 1 0 ~2 0 −1
Sh

    
Ans 5b) : [σ1 , σ2 ] = = = i~σ2
4 1 0 0 −1 4 1 0
To prove othonormal
  eigen  have to show σ1 |β1 i = |β1 i and σ2 |β2 i = |β2 i
 states, we
~ 0 1 1 ~ 1
σ1 |β1 i = √ = √ = |β1 i
2 2 1 0 1  2 2 1 
~ 0 1 1 ~ 1
σ2 |β2 i = √ = √ *There is probably some problem regarding the sign.
2 2 1 0 −1 −1
 2 2  
1 ~ 1 0 1
hβ2 |σ3 |β2 i = 1 −1
2 2  0 −1 −1

~  1 ~
= 1 −1 = [0] = 0
4 1 4 
   
1~  0 1 1 ~  1 ~
hβ2 |σ3 |β2 i = 1 1 = 1 1 =
22 1 0 1 4 1 2

91
Ans 6a) : Resistive force = cẋ
∴ Equation of motion ⇒ mẍ + cẋ + kx = 0 ⇒ ẍ + 2β ẋ + ωo x = 0. . .where ωo2 = k/m and β = c/2m
Now x = Ae iωt

∴ −Aω 2 eiωt + 2β(iω)Aeiωt + ωo2 Aeiωt = 0


or ω 2 − (2βi)ωp− ωo2 = 0 p
∴ ω = (2βi ± −4β 2 + p 4ω 2 )/2 = iβ ±
o ωo2 − β 2
2 2
∴ x = eiωt = e(−β ± i ωo − β )t
∴ ωo > β underdamped
⇒ ωo >> β Lightly damped
∴ ωo < β overdamped
∴ ωo = β critically damped

Ans 6b) : Differential equation: mẍ + cẋ + kx = Fo cos(ωf t)


or ẍ + 2β ẋ + ωo2 x = fo cos(ωf t)
∴ 2β = c/m and ωo2 = k/m
fo
The complex amplitude is zo = 2
ωo − ω 2 + 2iβω
e
fo ((ωo2 − ω 2 ) − 2iβω)
ki
=
(ωo2 − ω 2 )2 + 4β 2 ω 2
oo
pC
ar
Sh

 −2βω 
∴ φ = tan−1
(ωo2 − ω 2 )
For ω → 0, Re(zo ) > 0 and Im(zo ) < 0
∴ the phasor approaches 0 ∴φ=0
For ω → ∞, Re(zo ) < 0 and Im(zo ) > 0
∴ the phasor approaches −π ∴ φ = −π
fo
|zo | = p
(ωo2 − ω 2 )2 + 4β 2 ω 2
fo
For ω → 0 |zo | = 2
ωo
fo fo ωo β
For ω → ∞ |zo | = r = 2 . . . . as and both tend to zero
ω 2
o β 2 ω ω ω
ω 2 ( 2 − 1) + 4
ω ω

92
Ans 7a) : ∆E = h∆v or ∆v = ∆E/h
Also from uncertainty principal ∆E∆t = ~/2 [for minimum uncertainty]
~ 1 1
∴ ∆v = × = ⇒ ∆t = 10ms
2∆t h 4π∆t
E2 − E1 6 × 1.6 × 10 −19
∴v= = = 1.45 × 1015 Hz
h 6.634 × 10−34
And ∆v = 8Hz
Here ∆E is uncertain due to the uncertainty or specifically instability in the E2 state. The lifetime t2 cause a
variation ∆v
and E = hv
on partial differentiation we get ∆E = h∆v

Ans 7b) : ∆x = 2∆r = 8 × 10−15 m


∆x∆p = ~/2 [minimum deviation]
~
∴ ∆p =
2∆x
(∆p)2 ~2 ~
∴E= = 2
= = 81Kev
2m m(∆x) 32mr2

e
ki
oo
pC
ar
Sh

Ans 8a) : The reflected light is completely polarized at Brewster’s Angle


∴ θb = tan−1 (1.5) = 56.309o
Ans 8b(i)) : vo = c/µo = 1.754 × 108 ms
ve = c/µe = 1.724 × 108 ms−1
Ans 8b(ii)) : D = d(µe − µo ) where d = thickness and D = λ/2
λ
∴d= = 10µm
2(µe − µo )

Multiple Choice Questions:

Ans: (B)) In x-direction and z-direction, respectively


Propagation along +ve x (ωt − kx) type

93

− →
− → −
k = c( E × B ) . . . . . . where c is a real constant


∴ B is along +ve z direction.

Ans: (D) Kinetic Energy is minimum at ω = 0


ẍ + ωo x = fo cos(ωt)
fo
For Forced underdamped oscillations x(t) = 2 cos(ωt) + Bcos(ωo t) + Csin(ωo t)
ωo − ω 2
fo
Neglecting transients x(t) = 2 cos(ωt). [Steady state solution]
ωo − ω 2
Complimentary function: Bcos(ωo t) + Csin(ωo t)
It is simply oscillating.
e
ki
Particular integral is oscillating with ω and not ωo .
v=0 is not true for all cases.It is only when t=0.
oo
pC
ar
Sh

Ans: (D)stable at x = -2;3. Unstable at x = 2


v(x) = x4 − 4x3 − 8x2 + 48x
v 0 (x) = 4x3 − 12x2 − 16x + 48 = 4(x3 − 3x2 − 4x + 12) = 4(x + 2)(x − 2)(x − 3)
Maxima at x=2 and mimina at x=-2,3
∴ stable at minima and unstable at maxima. Hence (D).

94
Ans: (A) E = (n + 1/2)~ω
1
For n = 0, E = ~ω
2

Ans: (D)Right-elliptically polarized



→ Eo
E2 = −ĵ √ sin(ωt − kz)
3

→ Eo
E1 = î √ sin(ωt − kz)
2

e
ki
oo
pC

Elliptically polarised as magnitudes of E1 and E2 are different.


Also it rotates clockwise.
ar
Sh

95
PHYSICS
END - SPRING SEMESTER EXAMINATION 2017-2018
Q 1.a) A cadmium red spectral line has maximum intensity , Imax , at wavelengh 643.847 nm. The inten-
sity falls to the value Imax /2 at wavelength 643.84752 nm and 643.84648 nm. Estimate the coherence length
of the electron , e= 1.6 × 10−19 C , 1eV= 1.6 × 10−19 J.[3]

Q 1.b) A beam of white light is normally incident on a transparent plate of refractive index 1.575 and thickness
0.5 × 10−6 m. What wavelengths lying within the limits of visible spectrum (400-700 nm) will be attenuated
most in the transmitted beam? [4]

Q 1.c) An ideal young’s double slit is illuminated with a source having two wavelengths, λ1 = 418.6nm and
λ2 = 421.4nm. In the source the intensity at λ1 is double of that at λ2 . Determine the visibility of fringes near
order m=0 and near order m=75 on the screen.[ Visibility =(Imax − Imin )/Imax + Imin ) ] [5]

Q 1.d) A Fabry - Perot intereferometer is used to obtain circular fringes using monochromatic source. The
separation between the mirrors is d= 6 nm. By moving one of the mirrors by 0.03 mm it has been found that
100 fringes cross the field of viw. If the coefficient of amplitude reflection, r, of the mirrors is 0.734 find the
minimum wavelength separation( near the source wavelength ) which could be resolved by the above interfer-
ometer. [6]

e
ki
C
1.a) lc = c(∆T ) = |∆V | ∆T = time of wave packet
oo

Vλ=c ∆V = bandwidth of source


or (∆V )λ + V (∆λ) = 0
pC

(∆λ)c
⇒ ∆V = ∆ λ λV = λ2
2 −9 2
= (643.847×10 )
ar

c λ
∴ lc = ∆V = (∆λ) 1.04×10−12
m = 0.4m
Sh

b) Minimum condition for the transmitted beam in thin files


2µcosθ = (m + 1/2)λ [ Refer interference of transmitted beam in thin film interference ]
Now cosθ = 1 [ normally incident]
2µt
∴ λ = (m+1/2)
−6
= 2×1.575×0.5×10
(m+1/2) m, m = 1, 2, 3, 4
m = 2 gives λ = 0.63µm
m = 3 gives λ = 0.45µm

1.c) For concordance to occur


(p + 1)λ1 = pλ2 = path difference
or (p + 1)418.6 × 10−9 = p × 421.4 × 10−9
p = 149.5 ≈ 150
∴ For m=0 ,150,300,.....The maxima of λ1 will lie on max of λ2
For discordance to occur
pλ1 = (p + 1/2)λ2 = Path difference
p = 75,(150+75),(300+75),..... The minima of λ1 falls on maxima of λ2

For m =0 For m = 75
Imax = 3, Imin = 0 Imax = 2, Imin = 1
V =1 V = 1/3

1.d) 2d = m λ
or 2(∆d) = (∆m)λ

96
→ λ = 2(∆d)
∆m = 0.6µm
∴ m = 2d
λ = 2 × 10
4

∴ CRP = mπf mπ 2r mπr


2 = 2 × 1−r2 = 1−r2
λ mπr
∴ ∆λ = CRP = 1−r 2 [ chromatic resolving power of Fabry - Perot interferometer]
λ(1−r2 )
→ ∆λ = mπr = 6pm

Q 2.a) a double s;lit has individual slit width b= 5.74µm. The separation between centres of the slits is
d= 22.96µm. A λ = 0.6µm and intensity 1 unit falls normally on this double slit. Drraw a neat Fraunhhofer
intensity pattern as function of diffraction angle θ , showing the qualitative and quantitative details, i.e. the
peaks inside the primary and first secondary lobes, their positions and intensities.( NO graph paper required) [5]

Q 2.b) A plane monochromatic light wave of intensity I0 and wavelentgh 0.64µm along theb +Z axis is normally
incident on an aperture placed at Z=0 plane with its centre, P, √ at origin . The
√ aperture , as shown in the Fig
e
1, has three concentric zones withe radii r1 = 0.4mm , r2 = 2r1 and r3 = 3r1 . The inner opaque part is
ki
made of two semicircles of radius r1 , r2 with their centres coinciding at P.only the white part captured between
oo

the shaded regions is open.Find the intensity on the z axis at a = 0.5m from the origin.
pC
ar
Sh

2 c) Calculate the parameter β = πb sin θ/λ for the first and the second maxima in a single slit
Fraunhofer diffraction and hence find the ratio of their intensities (see Fig. 2)

2.a) First d/b = 4


∴ The missing orders are 4,8,12,......
2β 2N r
I = I0 sin
β2
× sin
sin2 r
Where β = πbsinθ
λ and r = πdsinθ
λ
∴ The outer cover is of I0 sin2 β/β 2 and inside is the wave of sin2 N r/sin2 r

Maxima at dsinθ = nλ, n 6= 4, 8, 12, .....

97
For central maxima ,I0 = 1 = I0
For 1st maxima , dsinθ = λ1 → sinθ = λ/d → θ ≈ 1.5◦
For 2nd maxima , dsinθ = 2λ → sinθ = 2λ/d → θ ≈ 4.5◦
For 3rd maxima , dsinθ = 3λ → θ ≈ 3◦
For 4th maxima (absent), dsinθ = 4λ → θ ≈ 6◦ e
ki
For 5th maxima,θ ≈ 7.5◦
oo

For 6th maxima,θ ≈ 9◦


For 7th maxima,θ ≈ 10.5◦
pC

For 8th maxima,θ ≈ 12◦


ar

Also I0 = 1, I1 = 0.81, I2 = 0.40, I3 = 0.09, I4 = 0, I5 = 0.032, I6 = 0.045, I7 = 0.016, I8 = 0


2β 2N r
For maxima I = I0 sin as si →1
Sh

β2 sin2 r
β = πbsinθ
λ (d/b = 4, dsinθ = nλ)
= πλ × d4 sinθ = nλ × πd
4λ = 4

The 1st row is off the 1st lobe and 2nd row is of the 2nd lobe.

2.b) rn = nbλ
2 r12
n = rn
bλ = n 1 = bλ = 2
1

∴ n2 = 2n1 = 1 n3 = 3n1 = 3/2


[ Read the concept of vibration spiral]
∴ 1/2 fresnel zone is covered next 1/2 is half open and 3rd 1/2 is full open.
√ √
∴ Inet = ( 2A 0 2 2 5
2 ) + ( 2A0 ) = 2 I0
The 2nd 1/2 is halved as the zone is half open. The 3rd 1/2 is full is at full open .
2.c) For maximum tanβ = β
A maxima will occur near β = (2m + 1)π/2, m = 1, 2, 3 as maxima occurs at .
Let first maxima occur at β = 3π
2 + δ1 , δ1 is small
∴ tan(3π/2 + δ1 ) = 3π2 + δ 1
→ −cotδ1 = 3π 2 + δ 1
1
→ − tanδ 1
= 3π
2 + δ1
→ δ11 + δ1 = − 3π
2
Solving we get ,δ1 = −0.07π
∴ β = 1.430π

98
Similarly for the 2nd one , β = 5π
2 +2 , δ2 is small
∴ β = 2.459π and δ2 ≈ −0.041π
∴ II21 ≈ 0.0164
0.047
≈3

e
ki
oo
pC
ar
Sh

3.a) n0 > ne [-ve crystal ]


(please read the theory of E - ray and O - ray in +ve or -ve crystals)
∴ ϑ0 (ordinay ray) ¡ ϑe (extraordinay ray)
Also after the 2nd surface , the E-ray becomes the O-ray.

99
e
ki
oo
pC

3.b) Let the light incident has intensity I0


ar

Ax = A0 sinωt or Ax = A0 cosωt
Sh

∴ After the light passing through the quarter wavw plate , a phase difference of π/2 is introduced
∴ Ay = −A0 sinωt or Ax = A0 cosωt any one of them occurs .
∴ The light becomes linearly or plane polarised at π/4 or 3π/4

circularly polarised
∴ Anet = A0 always
∴ Intensity = I0
Then after passing through polaroid, it becomes I0 cos2 π4 or I0 cos2 3π4 = I0 /2
In first case, the optics axis itself acts as a polaroid ,As there is unpolarised light, the light entering as O-ray
has its intensity halved while getting polarised and similarly the case of E- ray .
Thus A0 → A0 /2

100
..........

h
4.a) λc = m0 c [Crompton] λ = hp [Debroglie]

equating we get p = m0 c
also p = mϑ = √ m0 ϑ2 2
e
ki
1−(ϑ /c )
oo
p
∴ KE = p 2 c2 + m2 c4 − m c2
p 0 0√
= m20 c4 + m20 c4 − m0 c2 = ( 2 − 1)m0 c2 = 0.21M eV
pC
ar

2 2 2E
4.d) (i) hEi = |c1 | |cE11|+|c2 | E2 +|c3 | 3
Where c1 = 21 , c2 = √i , c3 = − 12
Sh

2 +|c |2 +|c |2
2 3 2
= 4.5 E1 = 1, E2 = 4, E3 = 9
|c1 |2 1/2
(ii)P2 = |c1 |2 +|c2 |2 +|c3 |2
= 1/4+1/2+1/4 = 1/2
4.b) (∆γ)max ∼ x (∆P )min ∼ ~
x = 1.7 × 10−25 kgm/s
2
Pmin
Pmin ∼ (∆P )min → Emin = 2me = 0.16 × 10−19 ≈ 0.1eV

4.c) We know P is hamiltonian operator


∴ P = PT
d d T d T
→ i~ dx = (i~ dx ) = i~( dx )
d d T
→ (− dx ) = ( dx )
d T d d
→ (− dx ) = ( dx ) 6= −( dx )
∴ It is not an Homiltonian operator

4.e)[xy, Lz ]
= x[y, Lz ] + [x, Lz ]y
= x[y, xPy − yPx ] + [x, xPy − yPx ]y
= x[y, xPy ] − x[y, yPx ] + [x, xPy ]y − [x, yPx ]y
Now [y, yPx ] = [y, y]Px + y[y, Px ] = 0
Similarly [x, xPy ] = 0
∴ [xy , Lz ] = x[y, xPy ] − [x, yPx ]y
= x2 [y, Py ] + x[y, x]Py − y[x, Px ]y − [x, y]yPx

101
= x2 [y, Py ] + 0 − y 2 [x, Px ] − 0
= (x2 − y 2 )i~
Use [A,BC] = [A ,B]C + B[A,C]
and the results given also.
Also [x, Py ] = [y, Px ] = 0

e
ki
oo
pC
ar
Sh

102
PHYSICS
END - SPRING SEMESTER EXAMINATION2016-2017

e
ki
oo

√ √
1.a) Let y = 2 sin(Wt) and t = 3 sin(Wt + π/3)
pC

By theory it is ellipse ⇒ Polarization is elliptical


ar

y2 z2 2xy
⇒ 2 + 3 − √ cos(π/3)
6
= sin2 (π/3)
Sh

(b). (1)∆λ = mh0 c (1 − cosφ)


(∆λ)max = m2h0 c = 4.8pm ⇒ Energy transferred to the electron = hc
λ - hc
λ+∆λ = hc
λ(λ+∆λ) (∆λ) (Approximately)
hc(∆λ)
= λ2
6×10−34 ×3×108 ×4.8×10−12
⇒ ∆E = 4000×4000×10−20
= 3.1725 × 10−5 eV

(2)Work function = 3.09eV

⇒ It cannot eject electrons from metal by Compton Collision .

(3)Please consult your professor


q √
k 9
(c)w1 = = 1 =3
q m
w2 = 9+0.2
1 Approximately = 3.32
⇒ q0 = A1 cosw1 t and q2 = A2 cosw2 t
Where q0 = x1 +x
2
2
and q1 = x1 −x
2
2
[ Please refer other papers for proper explanation]

q1 +q0
x1 = 2 = A1 cosw1 t + A2 cosw2 t

x1 = q0 −q
2
1
= A1 cosw1 t − A2 cosw2 t
⇒ x1 (0) = 5 ⇒ A1 + A2 = 5

103
and x2 (0) = 0 andA1 − A2 = 0

⇒ A2 = A1 = 2.5
⇒ x1 = 2.5(cosw1 t + cosw2 t)
x2 = 2.5(cosw1 t − cosw2 t)

Tbeat = w12π
−w2 Approximate = 190s
This is the full span between which the particle oscillates [ plot the function to have better visualization] ⇒

Required time period= 95s


Average time period = w12π
+w2 = 2.08s
2
⇒ Approx 45 to 46 oscillations .
e
ki
oo
pC
ar
Sh

2.a) β = λD
d
−9 ×D
⇒ 10−3 = 600×10
0.5×10−3
or D = 0.5/0.6 = 0.833m

b) (µ − 1)t = dy
D
0.5×100×10−6 10−3 ×0.5×10−3
or y = (µ−1)tD
d = 0.5×10−3
× 600×10−9
= 0.5/6 = 0.083m

c) Intensity αcos2 (θ/2) Where φ is the phase difference.


At φ = 0 , maxima and at φ/2 = π4 , it is half intensity .
⇒ φ = π/2
⇒ φ = 2πdsinθ
λ = π/2
λ 600×10−9
or sinθ = 4d = 4×0.5×10 −3

104
y
D
0.6
= sinθ = θ = 4×0.5 × 10−3
or y = 0.3 × 10−3 × 0.833 = 0.25mm

d) For single slit maxima ,β = mπ, mN


Where β = πbsinθ
λ ⇒ mπ = πbsinθ
λ [Refer notes for better explanation ]
or mπ = bsinθ ⇒ 2λ1 = 3λ
or λ1 = 1.5 × λ = 1.5 × 600nm = 900nm

e
ki
oo
pC

Probably not in syllabus.


ar
Sh

ANSWER:p
∆x = hx2 i −Rhxi2
∞ 2
Now hxi = A2 −∞ xe−2ax dx
⇒ It is anR odd function , it is zero
∞ 2
hxi = A2 −∞ xe−2ax dx = I(let) [ hzi = Ψ∗ ẑΨdx Whereẑ is the operator of z
R

Let x2 =R∞
z ⇒ dt = 2xdx
I = A −∞ ze−2az 2dz
2 √
z
R ∞ √ −2az 2 2√
2
= A −∞ ze dz = A2 r(1/2) = A 2 π

105
p Aπ 1/4
⇒ ∆x = hx2 i = √
2
2 2
For P , P = −ih̄ dΨ
dx = −ih̄(−2ax)Ae−ax
= A2aih̄xe−ax
⇒ Actually for gaussian wave function , the uncertainty is always h̄/2 [minimum uncertainty]
⇒ ∆x∆p
√ = h̄/2
or A/ 2π 1/4 (∆p) = h̄/2
⇒ (∆p) = A(π)h̄1/4 √2
(1/2)2
(b) hOio1 = (1/2)2 +(1/3)2
= 0.693
(1/3)2
hOio2 = (1/2)2 +(1/3)2
= 0.308

h2 (n+1)2 h2
(c) For 1 dimensional infinite potential well ,En = (n + 1)2 E0 where E0 = 8mL2
, En = 8mL2
⇒ hEi =
c21 E0 +c22 E1 +c23 E2
c21 +c22 +c23
Where c1 = 3/13, c2 = 4/13, c3 = 12/13

See Ψ1 is the wave fuction of the ground state of the particle .


2 2 2 2 ×32 )
8.1h̄2
hEi = (3 +4 ×213+12
2 E0 = 8.1E0 = 8mL 2 =
1.0125h2
mL2
−iE1 t −iE2 t −iE3 t
3 4 12
Ψ(x, t) = 13 Ψ1 (x)e h̄ + 13 Ψ2 (x)e h̄ + 13 Ψ3 (x)e

WeRknow that all φ are orthogonal



⇒ −∞ φm (x)φn (x)dx = 0 for m 6= n
−iEt
Ignoring the time independent part [e h̄
e
When we do Ψ1 (x, t)∗ Ψ2 (x, t) we get terms like Ψ1 (x)∗ Ψ2 (x)e−i(E1 −E2 )/h̄ But averaging over space performing
ki
the integral , we get zero .So the average energy is time - independent.
oo
pC
ar
Sh

ANSWER: 5. a) E = hv
P P −10
⇒n= E = hv = 6.6×1010
−34 ×96×106 = 1.5783 × 10
15

b)λ = Ph
⇒ P = hv c =
E
c

Or dP dE
dt = (1/c) dt
10−10
or force =(1/c) × P ower = 3×108
= 3.3 × 10−17 N

c) Q = 1 × 4.2 × 10

106
E = hv = hc/λ

4.2×10×520×10−9
⇒h= 6.6×10−34 ×3×108
= 1.1 × 1020

d) For diffraction , λ should be in the order of the Slit width , lets take it equal.
⇒ λ = Ph = mv h
, λ = 0.76
6.6×10−34
⇒ v = mλ = 65×0.76 = 1.34 × 10−35
h

6.6×10−34 ×3×108
e) ∆E = h(∆v) = hc( λ12 − 1
λ1 ) = 10−9
1
( 97.23 − 1
486.2 ) = 10eV

⇒ The energy of the hydrogen atom decreased by 10eV

e
ki
oo
pC
ar
Sh

107
ELECTRICAL TECHNOLOGY
MID - AUTUMN SEMESTER EXAMINATION2017-2018

Q.1. a) Find the value of the resistance R as shown in the figure below such that minimum
power is transferred to the 4Ω resistor and also calculate the amount of power dissipated in the
4 Ω resistor under this power transfer condition.

e
ki
oo
pC
ar
Sh

ANSWER:

108
e
ki
oo
pC
ar

..............
Sh

Taking currents I1 in IDC and I2 in IEF and passing other currents in other branches using KVL .
Using KVL in loop EFDC
+10 + 2(6 − I1 ) − 2I1 = 0
⇒ 10 + 12 = 4I1
⇒ I1 = 5.5A
∴ I(4Ω) = 4 − I1 + I2
= 10−6R
R+4
Minimum power is 0 watt
∴ I = 0 for R= 5/3
∴ I 2 R = 0watt
But I =0 For R = 5/3 . Thus minimum Power is 0.
Using KVL loop in FGE (4Ω)
+ 10 − R(10 − I1 + I2 ) − 4(4 − I1 + I2 ) = 0
⇒ 10 − 10R − R(I2 − I1 ) − 16 − 4(I2 − I1 ) = 0
⇒ −6 − 10R = (I2 − 5.5)(R + 4)
⇒ I2 = 5.5 − 6+10R R+4

Q. 1.b)Calculate the following AC network . Obtain the thevenin voltage and Thevenin im-
pedence at the terminals a-b of the network.

109
..................

ANSWER:

1.b)For RT h

e
ki
oo
pC
ar
Sh

Znet = 8(−6j) 4(12j)


8−6j + 4+12j = 6.48 − 2.64j ≈ 7∠ − 22.17

∴ ZT h = 6.48 − 2.64jΩ
For VT h
120∠75◦ ×8
Va = 10∠−36.87 ◦

Va = 96∠111.87◦ V

×12∠90 ◦
Vb = 120∠75
12.65∠71.57◦
Vb = 113.834∠99.43◦ V

VT h = Vb − Va = 28.83∠53.6◦ V

3.Q. Consider a housing load connected to a single - phase 50 Hz supply as follows .


(i) In the absence of the capacitor , the voltage across the terminals a-b is 200 V . Calculate
the source current Is and the overall power factor as seen by the source.
(ii) Calculate the value of the capacitance to be connected at the terminals a-b as shown . So
that the magnitude of the source current Is in the case?

110
ANSWER:

3. For Refrigerator .
Power = 1000 + 750j e
ki
For Heating and lightning
oo

Power = 1000
pC

P
For Refrigerator I = V cosφ 1000
= 200×0.8 = 6.25∠ − 36.87◦
P
= 1000 ◦
ar

For Heater I = V cosφ 200 = 5∠0


∴ Isource = 10.68∠ − 20.56 ◦
Sh

Overall power factor = cos ( 20.56) = 0.936


Zref rigerator = 32∠36.87◦
Zheater = 40∠0◦

Let Vab = 230∠0◦


and Vsource = 230∠δ
P
I1 = V cosφ 1000
= 230×0.8 = 5.435∠ − 36.87◦
P
I2 = V cosφ 1000
= 230×1 = 4.348∠0◦
∴ I1 + I2 = 9.287∠ − 20.56◦
∴ I1 + I2 + Ic ∠90◦

111
∴ 230∠δ = 230∠0◦ + (9.287∠ √ − 20.56◦ + Ic ∠90◦ )(1.414∠45◦ )
= 230∠0 + 13.134∠24.44 + 2Ic ∠135◦
◦ ◦

= (241.948Ic ) + (5.43 + Ic )j
∴ (230)2 = (241.948Ic )2 + (5.43 + Ic )2
52900 = 58538.83 − 483.896Ic + Ic2 + 29.48 + Ic2 + 10.86Ic
0 = 5668.31 − 473.036Ic + 2Ic2
Ic = 12.661, 223.86
We know V ωC = Ic
∴ (230)(2π(50))C = 12.661, 223.86
C = 175.2µF, 3.098mF

Q.4. A fixed and finite frequency sinusoidal voltage source of 5 V rms value is connected to a
series RLC branch. This branch has fixed valued R and L and variable valued C as shown in the
following figure . When C = 15 farad the rms value of current flowing through the series RLC
branch is 1A and the active power supplied by the voltage is 3W. When the value of capacitor
1
is changed to 45 farad the rms value of current flowing through the RLC branch is again 1A.
(i) Calculate the values of R and L.[2 + 6]
(ii)For what value of C, maximum power will be transferred to the RLC branch?[4]

e
ki
oo
pC
ar
Sh

ANSWER:
Vrms = 5V
Case I:
C = 1/5F Irms = 1A
2
∴ Power Irms R[Irms 2 |Z|cosφ = I 2 R]
rms
3W = 1 × R
R = 3Ω
since in both cases magnitude of current is same , thus impedence magnitude is same
as R is fixed
1 1
∴ |ωL − ωC 1
| = |ωL − ωC 2
|
1 solution is C1 = C2
1 1
ωL − ωC 1
= ωC 2
− ωL
1 1
⇒ 2ωL = ω ( C1 + C12 )
⇒ 2 × ω × L = ω1 (50) − − − − − − − − − − − − − − − −(1)
I = 1A
|V | = |I||Z| as R = 1Ω
For capacitance VR = 3V in both cases
∴ Voltage across land C = 4V in both cases
1
4 = 1 × |ωL − ωC 1
|

112
C1 > C2
∴ X1 < X2 [ X is capacitive reactance]
∴ Circuit is inductive in 1st case and opp in 2nd case.
1
4 = 1 × |ωL − ωC 1
|
5 25
⇒ 4 + ω = ωL = ω [from (i) ]
4 = 20
ω
⇒ ω = 5Hz
∴ 4 + 55 = 5L
⇒L=1
Maximum power transfer means resonance condition
1
∴ ωL = ωC
⇒ C = ω21L = 25×11
= 0.04F

Q.5. Each phase impedence of the star connected three phase load of resistance of 20 ohm
and inductance of 0.2H . Three capacitors each of C = 90µF , are connected in delta as shown
in the figure .The arrangement is connected to a three phase 400 V , 50 Hz balanced voltage
source . The pahse sequence RYB . Considering phase - R voltage as reference , obtain phasors
(i) I1 , I2 and I3 and (ii) IR , IY and IB . Draw a phasor diagram showinh the line voltages and line
currents at the source end

e
ki
oo
pC
ar
Sh

Answer: Zm = 20 + j(100πtimes0.2) C = 90µF VL = 400V


VRY = 400∠ + 30◦ Zph = 20 + j(62.832) = 65.94∠72.34◦
VY B = 400∠ − 90◦
VBR = 400∠ − 210circ
VR = (Voltage across Z1 in Rn branch )
√ ∠0◦
= 400
3

113
Also VY = 400
√ ∠
3
− 120◦
VB = 400
√ ∠
3
− 240◦

∴ I1 = VZR = ( 400
√ ∠0◦ )/65.94∠72.34circ ≈ 3.5∠ − 72.34◦
3
∴ I2 = 3.5∠ − 192.34◦
I3 = 3.5∠ − 312.34◦
Converting ∆ network to Y
0 −j −j×106
Where Z = ωC = 100π×90
−j×103
= 9π ≈ −35.368j

e
ki
oo
pC
ar
Sh

0
∴ Z /3 ≈ −11.789j = 11.789∠ − 90◦
0
√ ∠0◦ )/11.789∠ − 90◦ = 19.589∠90◦ A
∴ I1 = ( 400
3
0
∴ I2 = 19.589∠ − 30◦ A
0
I3 = 19.589∠ − 150◦ A
0
∴ IR = I1 + I1 = 19.589∠90◦ + 3.5∠ − 72.34◦ = 16.289∠86.26◦ A
0
IY = I2 + I2
= 19.589∠ − 30◦ + 3.5∠ − 192.34◦
= 16.289∠ − 33.74◦ A
∴ IB = 16.289∠ − 153.74◦ A

114
e
ki
oo
pC

Q. 6. Assume that the circuit shown in the following figure is at steady state with switch ” S ”
ar

open . This switch closed at t =0 .The inductor current is i(t).


Sh

di +
(i) calculate i(0+ ) and dt (0 )
(ii) Calculate the steady state inductor current with switch ” S ” closed .
(iii) Obtain the expression for the inductor current i(t) for t>0.
Answer:

When switch is open


Rnet = 9×180
27 + 4 = 10Ω
120
∴ Current (in net circuit)= 10 A = 12A
18
∴ iL = 27 × 12 = 8A

115
∴ i(0+ ) = i(0− ) = 8A (properly of inductor)
When switch is closed

e
ki
Using Thevenin theorem
oo
pC
ar
Sh

∴ RT h = 9 + 18×2
20 = 10.8Ω
18
VT h = 20 times120V = 108V
∴ Equivalent Thevenin circuit

di
∴ L dt + iR = V
di
2 dt + i( 108
10 ) = 108
di 54
dt + 10 i = 54
di
or 10 dt = 54(10 − i)

116
R di R
or 10 10−i = 54dt
(Intergrating both sides and putting proper limits )
10−i
10 × ln| 10−8 | = −54t
10 − i = 2e−5.4t
10 − 2e−5.4t = i
∴ current through the inductor for t¿0 is i = (10 − 2e−5.4t )A
∴ steady state current is at t = ∞
∴ i = 10A
di −5.4t
dt = 10.8e
di +
∴ dt (0 ) = 10.8

e
ki
oo
pC
ar
Sh

117
ELECTRICAL TECHNOLOGY
MID-SPRING SEMESTER EXAMINATION 2017

Ans 1a): Peak power factor = peak value/RMS value


1/2
Z T
(sin ωt + a sin 5ωt)2 e 2π
ki
RMS value = where, T =
0 T ω
oo

1 √
= √ 1 + a2
√2
pC

Peak value = 1√+ a2


∴ peak factor = 2
ar
Sh

Ans 1b): For the circuit to be resistive


XC = 8.66Ω Parallel circuits have same
1
or, = 8.66 voltage across them.
ωC
1
or, C = As resistances are same, the shift in phasor up due to the XL must
8.66 × 2π(50) 
=36.76 mF be equal to the shift in phasor down due to the XC
IL + IC = 10∠0o A
∴ Impedance of inductive branch ≈ 10∠60o Ω
∴ Impedance of capacitive branch ≈ 10∠ − 60o Ω
XL I (10∠60o )(10∠0o )
IC = =
XL + XC 5
√ = 20∠60o A
∴ IL = 10 3∠ − 90o A
V = IC XC = (20∠60o )(10∠ − 60o )
= 200∠0 o
√ V
VR1 = 5 × IL = 50 3∠ − 90o V
VR2 = 5 × IC = 100∠60o V
VXL = 200∠0o − VR1 ≈ 218∠23.4o V
VXC = 200∠0o − VR2 ≈ 173.2∠ − 30o V
I = 10∠0o A
∴ Phasors can be drawn accordingly.

118
e
ki
Ans 2a): XL = j(ω)(140) × 10−6
oo

−j
XC =
pC

ω(870 × 10−12 )
R = 2000
1 1 1 1
ar

∴ = + +
Znet XL  XC R
Sh


1 1 1
= (7142.857) − ω(870 × 10−12 ) +

 j  2000
1 7142.857 1
= − 870ω × 10−12 +
j ω  2000
7142.857 1
= j 870ω × 10−12 − +
ω 2000
2000
∴ Z(jω) =  
7142.857
1 + 2000 × (j) 870ω × 10−12 −
ω
2000
=
1.43 × 107
 
1 + (j) 1.74ω × 10 − −6
ω
Now, IC is maximum when VC is maximum.
As parallel circuit, V is same
V = I × Xnet
∴ |Xnet | is maximum.
The denominator is 1
1.43 × 107
∴ 1.74 × 10−6 ω =
ω
2 1.43 13
or, ω = × 10
1.74
or, ω = 2.87 × 107

119
Ans 2b): For current sources opened and voltage sources shorted

1(−j)
Xnorton = + j(2) = (0.5 + 1.5j)Ω
1−j

e
ki
oo
pC


 
0.5 − 0.5j
∴ IN = [10 − 3j]
(0.5 − 0.5j) + 2j √
ar

= [−0.2 − 0.4j][10 − 3j]


Sh

= 4.539∠ − 126.4o
≈ 4.54∠ − 126.4o

XT h = (0.5 + 1.5j)Ω
I = 4.54∠ − 126.4o A
For maximum power drawn,
R =p [RT2 h + XT2 h ]1/2
= (0.5)2 + (1.5) 2
 ≈ 1.58Ω
ZT h
Iab = I
ZT h + R 
ZT h R
∴ Vab = I
ZT h + R
=(0.974∠35.77o )(4.54∠ − 126.4o ) V
=4.422∠ − 90.63o
=4.422 cos(10t − 90.63o ) V
=4.422 cos(10t − 1.582) V
Ans 3a): Let both the networks be replaced by Thevenin equivalents. [See the terminals]

120
e
ki
oo
pC
ar
Sh

VA VB
∴ + = 16.5 A From 1
RA RB
VA VB
and, − = 7.5A From 2
RA RB
VA
∴2 = 24
RA
or, VA = 12RA . . . . . . . . . . . (i)
VA − VB
and, =1 From 3 We need,
RA + RB
VA + VB
or, VA − VB = RA + RB I4 =
RA + RB
or, 12RA − VB = RA + RB
or, 11RA = VB + RB
RA VB
or, 11 = +1
RB RB
RA VA
or, = − 7.5 + 1 From 2
RB RA
RA
or, = 12 + 1 − 7.5 From (i)
RB
or, RA = 2RB
∴ From 3

121
VA − VB = RA + RB
or, 12RA − VB = RA + 0.5RA
or, VB = 10.5RA
12RA + 10.5RA
∴ I4 = = 15 A
RA + 0.5RA
Ans 3b(i):
 P = V I∗ 
(100 − 100∠φ)
P1 = Re 100 ×
1 + 0.5j

(1 − 1∠φ)

e
ki
= Re 100 × 100 ×
1 + 0.5j
oo
 4  
10 (1 − 1∠φ)
= Re
1.12 1∠ − 26.565o
pC

π φ
 
104 φ o
j( − +26.565 )
ar

= Re  × 2 × sin × e 2 2 
1.12 2
Sh

φ
 
104 φ j(116.565o − )
= Re  × 2 × sin ×e 2 
1.12 2
φ φ
∴ we have to maximize sin cos(116.565o − )
2 2
1 o o
= [sin(116.565 ) + sin(φ − 116.565 )]
2
∴ φ = 116.565o + 90o
∴ φ = 206.565o
Ans 3b(ii):

122
P = V I∗ 
(−100∠φ)(100 − 100∠φ)∗

∴ P2 = Re ∗
 (1 + 0.5j)
100 − 100∠φ
I=
1 +0.5j 
(100∠φ)(100∠(−φ) − 100)
∴ P2 = Re
 4 1 − 0.5j
10 − 104 ∠φ
= Re
 1 4− 0.5j4 
10 − 10 ∠φ
= Re
 1.12∠ − 26.565
104

1 − 1∠φ
= Re
 1.12  1∠ − 26.565
104

φ j φπ o
= Re 2 sin e ( + 26.565 )
1.12 2 12

e
ki
φ φ
oo

∴we have to maximize sin cos( − 63.435o )


2 2
1
pC

= [sin(63.435o ) + sin(φ − 63.435o )]


2
∴ φ = 63.435o = 90o
ar

or, φ = 153.435o
Sh

123
Ans 4a):

∴q =VC
dθ(t) dθ
or, i = C =2
dt dt
Voltage across capacitor,
3(2 − i) = θ(t)
dV
or, 6 − 3 × 2 =V
dt
V dV
or, 1 − =
Z t 6 Z dt V
dV
or, dt =
0 0 1− V
6V

V e
ki

or, t = 6 ln 1 −
6
oo

 V 0
V
= −6 ln 1 −
pC

6 0
V
or, e−t/6 = 1 −
ar

6
or, V (t) = 6(1 − e−t/6 ) volts
Sh

dV 1
∴ i(t) = 2 = 2 × 6(e−t/6 ) = 2e−t/6 Amperes
dt 6
Steady state voltage φ(∞) = 6 V

Ans 4b):

dv −i
= where i is the current
dt 2
dv
or, i = −2 As direction is opposite it is negative.
 dt
t
∴ 18 cos − 4i − v = 0
8   
t dv dv
or, 18 cos = 4i + v = v + 4 −2 =v−8
8 dt dt

124
 
dv t
or, 8 = v − 18 cos
dt   8
t dv
or, 18 cos =v−8
8 dt
t
On solving we get, v(t) = 9(1 − sin )V
 8 
dv t 1 9 t
∴i=2 = −18 cos × = − cos amperes
dt 8 8 4 8

e
ki
oo
pC
ar
Sh

Ans 5a): Va = −20.92 − j(239.0.9)


≈ 240∠ − 95o
Vb = −196.6 + j(137.66) ≈ 240∠145o
Sequence is cab
Va = 240∠ − 95o
Vb = 240∠ − 215o
Vc = 240∠25o

125
Power at wye load = 3kW at unity power factor.
Power at source = 4.8 + j(6.4)
∴ Power at delta = 1.8 + j(6.4)
∴ Apparent power = 6.65kVA Vp h = 240 V
V2
∴ = 6.65 × 103
|Z|
or, |Z| = 0.115  
−1 6.4
Also power factor = cos(tan ) = 0.271
1.8
∴ Z = 0.115∠74.3 o

Ans 5b): Working with rms values only


iRY = 90∠0o VR = 240∠0o
iY B = 90∠ − 120o VY = 240∠ − 120o
iBR = 90∠ − 240 o VB = 240∠ − 240o
We can replace the capacitor and current source with a capacitor and voltage source.
e
∴ those voltages (PR Y = 90∠0o )(84∠ − 90o ) = 7560∠ − 90o
ki
PY B = 7560∠ − 210o
oo

PBR = 7560∠ − 330o


pC
ar
Sh

Only 1 phase of the line


Both sides are grounded as it is balanced
I = 94.5 + 3j
∴ VM = 240 + (2j)(94.5 + 3j)
= 240 + 189 − 6j
= 429 √− 6j = 429∠ − 0.8o
∴ VM = 429 2 sin(100ωt − 0.8o )
∴ line voltages

VRY = 429 √6 sin(100π + 29.2o )
VY B = 429√ 6 sin(100π + 149.2o )
VBR = 429 6 sin(100π + 269.2o )

126
Ans 6a):

e
ki
oo
pC

ZS = 4 + 8j
ZL = 300 + 210j Ω √
ar

Phase voltage = 400/ 3


Sh

Converting to star,

ZL0 = 100 +70j0 


−1 XL + XS
φ = tan 0 +R ≈ 37o
RL S
Power = VRY IR ∗

127
 2
Vph 0
∴ the real part = × RL
|ZS + ZL0 |
400 × 400
= × 100 = 315.582 W
3 × 130 × 130
∴ Total power = 946.746 W
Let VRY = 400∠0o VY B = 400∠ − 120o
400 400
VR = √ ∠ − 30o VB = √ ∠90o
3 3
400 o 400
∴ IR = √ ∠ − 67 IB = √ ∠53o
3 × 130 3 × 130
∴ VR0 = VR − IR × ZS
400 300
= √ ∠(−30o ) − √ × 8.9∠ − 3.57
3 3 × 130
= 217∠ − 31.86 o

∴ VY 0 = 217∠ − 151.86o
∴ VB 0 = 217∠ − 271.86o
∴ VR0 Y 0 = VR0 − VY 0 = 375.86∠ − 1.86o
∴ VB 0 Y 0 = VB 0 − VY 0 = 375.86∠58.14 o
(400)2
  
∴ For 1st ammeter Preal = Re √ o o
∠67 (375.86∠ − 1.86 ) = 280.77 W
3 × 130
For 2nd ammeter,
  
o 400 o
Preal = Re (375∠58.14 ) √ ∠ − 53 = 665.03 W
130 3
Note: the connection of the voltage coils of the wattmeters are not clear. Please verify from your professors.
Ans 6b):
e
ki
oo
pC
ar
Sh

XL = 400 × 2π × 5 × 10−3 = 12.57Ω


RL = 4000Ω
∴ power factor = cos(tan−1 (12.57/4000))
Note : The datas are extremely unmatchable. A load of (4000 + 12.57j) should not draw 4.5 A of current from
a source pf 240 volts. please consult your professor regarding this.

128
ELECTRICAL TECHNOLOGY
MID-AUTUMN SEMESTER EXAMINATION 2015

e
ki
RA RB RB RC RC RA
Ans 1a): Rab = P Rbc = P Rca = P
oo

RA RA RA
pC
ar
Sh

* Refer from books, such questions do not come these days.


Ans 1b):

15 15
∴I= =
R + 50 + 100 R + 150
−3 15
∴ 50 × 10 < < 100 × 10−3
R + 150
15
or 50 < < 100
R + 150
15 × 100 15 × 100
or R + 150 < > R + 150
5 10
129
or R + 150 < 300 R>0
or R < 150 R>0
∴ R varies from 0 to 150 Ω

Ans 2a): lnet (iron − core) = 0.12 × 2 + 0.16 + (0.16 − 0.002) = 0.558 = lc

e
ki
oo
pC
ar
Sh

lnet (air) = 0.002m = la  


lc la 1 lc
Equivalent magnetic resistance = + = + la
 µ µ A
r o  o µ A µ oA µr
1 0.558
= + 0.002
4π × 10−7 × 16 × 10−4 5000
16 × 10−4
 
0.558
∴ N I = 1.2 × + 0.002
4π × 10−7 × 16 × 10−4 5000
Flux density = 1.2
Total flux = Flux density × area
=16 × 10−4 × 1.2
∴ I = 4.033 A
Ans 2b): We know, φ = LI
or, 0.06 × 10−3 = LA × 10
or, 6 × 10−6 = LA = 6µH
di
Also V = −L
dt
φB = 0.8φA
dφB dφA
∴ = 0.8
dt dt
20
or, VB = 0.8VA = 0.8 × 6 × 10−6 × = 4.8 mV
0.02

130
∵ they are identical
∴ LA = LB = 6µH
By definition of mutual inductance
φB = MAB IA
or, 10−3 × 0.8 × 0.06 = MAB × 10
or, MAB = 4.8µH

e
ki
oo
pC

Ans 3): Let V2 be 100∠0o


V2 has same magnitude as V1 (Vs ) = 100V
ar

∴ IL = 10∠ − 37o
Sh

and let V1 be 100∠φ


∴ 100∠φ = (IL + IC )(2 + 6j) + 100∠0o

100∠φ = (8 − 6j + (100ωc)j)(2 + 6j) + 100∠0o


=(8 + (100ωc − 6)j)(2 + 6j) + 100∠0o
=(16 + 36 − 600ωc) + j(48 + 200ωc − 12) + 100
=(152 − 600ωc) + j(36 + 200ωc)

Taking magnitude on both sides,


(152 − 600ωc)2 + (36 + 200ωc)2 = (100)2

131
or, (1.52 − 6ωc)2 + (0.36 + 2ωc)2 = 1
or, (1.52)2 + (0.36)2 + (2ωc)2 + (6ωc)2 + 2(0.36 × 2 − 1.52 × 6)ωc = 1
or, 1.44 + 40(ωc)2 − 17.28ωc = 0
⇒ ωc = 0.31923 or 0.11277
or, 2π(50)C = 0.31923 or 0.11277
or, C = 1.016mF or 0.359mF
∴ Csh = 0.359mF
∴ IL = 10∠4 − 37o A
∴ IC = 11.277∠90o A [IC = 100ωc]

e
ki
oo

Ans 4): For t < 0


pC
ar
Sh

100
At steady state, I = = 0.8 mA
125
∴ Voltage across capacitor = 25 × 103 × 0.8 × 10−3 = 20 V
Vc (0+ ) = 20 V
After, t = 0

132
[where τ = RT h C]
We know, VC (t) = V∞ + (V0 − V∞ )e−t/τ
V∞ = 283V [It can be easily found as at t = ∞, no current flows through the capacitors and simply
voltage across capacitor is V∞ ]
Vo = 20V
100 × 25
∴ RT h = = 20Ω
125
∴ τ = 20 × 103 × 5 × 10−6
=100 × 103 × 10−6 = 0.1Hz
∴ V (t) = 28 + (20 − 28)e−10t e
ki
= 28 − 8e−10t
oo

∴ V (0.08) = 28 − 8e−0.8 = 24.41 V


pC

q =VC
=(28 − 8e−10t )(5 × 10−6 )
ar

∴ I = (8 × 10e−10t )(5 × 10−6 )


=(400 × 10−6 )e−10t
Sh

=0.4e−10t mA

133
Ans 5): VRY = 100∠0o Converting 4 to Y load,

VY B = 100∠ − 120o it gives, RY = 10/ 3Ω
VBR = 100∠ − 240o
100
∴ IR = 30∠0o VR = √ ∠ − 30o
3
100 e
ki
∴ IY = 30∠ − 120o VY = √ ∠ − 150o
3
oo

o 100
∴ IB = 30∠0 VB = √ ∠ − 270o
3
pC

[All the currents marked are after the capacitor. So they are not affected by the capacitors as they are in
parallel.]
ar

Also VY B = VY − VB
Sh

∴ Voltage at the junction of two 50µF capacitors is


VY − VB −j
V = VB + ×
−2j/ωC ωC
VY VB VY + VB
=VB + − =
2 2 2
100
= √ [1∠ − 270 + 1∠ − 150o ]
o
3
100 100
= √ × 1∠150o = √ ∠150o
3 3
∴ Reading  of wattmeter= V IR ∗

100 o o
Preal = Re √ ∠150 (30∠0 )
3
= -1500 W

134
Ans 6a): Changing the ammeter to equivalent voltmeter,

e
ki
oo
pC
ar
Sh

VA − 20 VA − (−5) VA
∴ At node A, + + =0
4 10 4
VA VA 1 VA
or, −5+ + + =0
4  10 2 4
1 1 1 9
or, VA + =5− =
 2 10
 2 2
5+1 9
or, VA =
10 2
15
or, VA = = 7.5 V
2
VA − (−5) 7.5 + 5
∴I= = = 1.25 A
10 10
∴ 1.25A current flows from A to B.

135
Ans 6b):

By Nodal Law analysis,


V −4 V V
+ + =0
10  10 40
2 1

4 e
ki
or, V + =
10 40 10
oo

or, V (8 + 1) = 16
16
pC

or, V = V
9
16
ar

∴ I40Ω = A = 0.044A
9 × 40
Sh

*****

136
ELECTRICAL TECHNOLOGY
END - AUTUMN SEMESTER EXAMINATION2017-2018

Q. 1.a) The magnetic circuit as shown in the figure has rectangular cross section . The cen-
tral limb has a cross section 1 cm × 2cm and all other limbs have cross section of 2cm × 2cm . Coil
1 has 500 turns(N1 ) and coil 2 has (N2 ) . Bothe the coils carry 1 A of current .In the figure , mean
flux path lengths L1 , L2 and L0 are 20 cm ,16 cm and 8 cm respectively.Relative permeability of
the material is 1600. Calculate
(i) the reluctance of the three sections having mean path lengths L1 , L2 and L0 [3]
(ii)the flux in the central limb.
(iii) self and mutual inductance of coil 1 and coil 2

e
ki
oo
pC
ar
Sh

1.a)(i)

L1 = 20cm
L2 = 16cm
L0 = 8cm
10−2 ×8
RL0 = µ0 µl r A = 1600×4π×10−7 ×2×10−4
= 1.989 × 105 AT /W b
−2
10 ×20
RL1 = 1600×4π×10−7 ×4×10−4
= 2.487 × 105 AT /W b

137
10−2 ×16
RL2 = 1600×4π×10−7 ×4×10−4
= 1.989 × 105 AT /W b
(ii)

V −500 V V +1000
R L1 + RL0 +R L2 =0
⇒V ( R1L + R1L + R1L ) = R500 L1
− 1000
R L2
1 0 0
1 2 500 1000
⇒V ( 2.487 + 1.989 ) = 2.487 − 1.9889
⇒ V (1.4076) ≈ −301.72
⇒ V ≈ −214.351AT /m
214.351
∴ φcentrallimb = RVL = 1.989×10 5 ≈ 1.078mW b
0

N2 R ×R
(iii)L1 = Rnet Rnet (f orL1 ) = RL1 + RLL1 +RLL0 = 3.482 × 103 AT /W b
2 0
∴ L1 = 0.72H
2 R ×R
L2 = RNnet Rnet (f orL2 ) = RL2 + RLL1 +RLL0 = 3.094 × 103 AT /W be
ki
1 0
Flux due 500At on other coil
R L0
oo

= RL +R L
× Rnet500
(f orL1 ) = 0.718H
1 0
pC

Q.1.b) A ring of magnetic material with relative permeability 3000 has rectangular cross section
of 5cm2 .The inner and outer diameters of the ring 20 cm and 25 cm respectively, gap of 1 mm
ar

length is cut across the ring . The ringis wound with 500 turns and a sinusoidal alternating
Sh

current of 50Hz.
(i) What will be the value of the current when the magnitude of the induced voltage in the coil
is 60V?
(ii) Find the maximum energy stored in the magnetic material and the air gap for a coil current
of 2A at 50Hz.

1.b)

µr = 3000
A = 5cm2
N = 500
Mean diameter = 22.5cm
Mean length = πd = 70.686cm
−2 ×70.596 10−3
Reluctance µr µl 0 A = 3000×4π10times5×10 −7 ×10−4 + 4π times5×10−4 ×10−7
= 19.66 × 105 At/W b

138
2
L = NR = 0.1272H
let the current be I0 sinωt and induced voltage be E0 sin(ωt + φ)
di
∴ L dt =E
Both averaged over time , we can write
∴ ωLI = E
⇒ I = 60/(π2 × 50 × 0.1272) = 1.5A

(ii)Total energy = 21 LI 2 = 12 × 0.1272 × 8 = 0.515J



Imax = 2 2A
LI = N φ √
⇒ φmax = 0.1272×
500
8
= 0.72mW b
Bm 2 2 −3
10 ×(7.2×10 ) −4 2
Air gap energy = 2µ 0
× Ax = xφ
2µ0 A = 2×4π times5×10−7 ×10−4 = 0.4133W
max

∴ Energy in magnetic material =( 0.51 - 0.413 )J = 0.097J

Q.2.a) Consider a conventional two winding transformer of 240V/120V, 12kVA with full load
efficiency % for a unity power factor load .It is reconnected as an auto transformer to provide
power to a load at 360V from a 120 V supply.
(i) Show the winding connection diagram of the auto transformer .[2]
(ii) Calculate the kVA rating of the auto transformer[4]
Assuming negligible core loss , calculate the full load efficiency of the auto transformer for a
0.8(lagging) power factor load.[4]

ANSWER: e
ki
2.a) (i) η = 96.2%
oo

Papp = 12kV A
pC
ar
Sh

IH = 12×1000
240 = 50A
IG = 12×1000
120 = 100A
∴ kV A rating = (150 ×120)kV A = 18000kV A
nScosθ
Also η = nScosθ+x 2P
cu
Put x=1 , cosθ = 1, S = 12kW
∴ Pcu = 474W
18×0.8
∴ For Auto-transformer ,η = 18×0.8+0.474 = 96.8%

139
2.b) OC test (LV side )
Rc = 333.33Ω = Voc2 /Poc
Rc = Vocq/Rc = 0.6A
∴ IL = Inet 2 − I 2 = 3.955A
R e
ki
∴ XL = Voc /IL = 50.572Ω
oo

SC test (HV Test)


2 × R = 300
Isc
pC

c
⇒ Rc = 3Ω
|z| = p
Vsc /Isc
ar


X = |z|2 − Rc2 = 3 3Ω
Sh

SC parameters referred to LV side(a=10), R = 0.003Ω, X = 0.052Ω


∴ Equivalent circuit

Q.3.a) The inductors L2 and L3 are coupled with mutual inductance of Mas shown in the
circuit. Calculate the voltage across L2 for a supply of 10 V, 50 Hz [10]

140
3.a)

e
ki
oo
pC
ar
Sh

XL1 = 5.03Ω XH = 3.14Ω


1
XL2 = 4.08Ω Xc = 100π×1.6×10 −3 = 2Ω
XL3 = 5.97Ω
Mesh 1 :
10 = (4 + 5.03j)I1 + (I1 + I2 )(3 + 4.03j + 3.14j)
or 10 = I1 (7 + 12.25j) + I2 (3 + 7.22j)
Mesh 2:
(I1 + I2 )(3 + 7.22j) + I2 (5.97j + 3.14j − 2j + 6) = 0
or I1 (9 + 14.33j) + I2 (6 + 7.11j) = 0
∴ I1 = 2.44 + 1.08j
I2 = −4.12 − 2.56j
∴ VL2 = (I1 + I2 )(XL2 + XH )
= (−1.68 − 1.48j)(7.22j)
= 10.69 − 12.13j ≈ 16.17∠ − 48.62◦
i = √8100
2 +42
= 11.2
∴ Real power = i2 R = 1KW
reactive power = i2 X = (11.2)2 (4) = 502W
Current through R = I/a= 22.4A

Q.3.b) For the circuit shown below the AC source has an internal impedance of Z= 3+ 4j Ω
. If the transformer is ideal and has N = 10 turns .calculate.

141
(i) N for which maximum power transferred to the resistive load R [4]
(ii)the real and reactive power delivered by the source for the condition as in (i) above [4]
(iii)current through load R for the condition as in (i) above[2]

3.b)

e
ki
oo
pC
ar
Sh

N1 = 10
(104 )(20a2 )
i = [(3+20a100
2 )2 +42 ]1/2 Power across R= (3+20a2 )2 +16
104 ×20
PR = ( a3 +20a)2 + 162
a
PR is maximum when a =0.5

∴ a = N1 /N2
⇒ N2 = 20

142
Q.4.a) Consider a 15kVA , 2300 V/230 v transformer having parameteras the high voltage side as core loss
component R = 100 kΩmagnetizing reactance X = 10kΩ ,equivalent leakage reactance Xeq = 6Ω and equivalent
resistance Req = 5Ω. The load is connected to the low voltage side
(i) Calculate the voltage regulation at full load at 0.25 lagging power factor [4]
(ii)For a unity power factor load,calculate the load current at which the efficiency of the transformer is maxi-
mum.Calculate the maximum efficiency [4+2].

4.a)

e
ki
oo
pC
ar
Sh

I = 15×1000
2300 = 6.522A
cosθ = 0.85 sinθ = 0.5265
∴ Voltage regulation = I(Rcosθ + Xsinθ)
= 6.522(5 × 0.85 + 6 × 0.5265)
= 48.33V

48.33V
∴ Voltage regulation % = 2300 × 100% ≈ 2.1%
V 2 (2300)2
(ii) Core loss =R = 100×103 = 52.9W
Efficiency is maximum when Pi = Pc
IL2 × (5) = 52.9W
or IL = 3.253A
∴ Efficiency is maximum at IL = 3.253A

143
4.b) (i) XL = 2000 R = 2000Ω ω = 100
1
Xc = 100×4×10 −6 = 2500
Steady state
√ RMS current e
ki
2 2
= √ = 0.883mH
oo
2 2
(2000) +(2500)
∴ rms Voltage = Irms Xc = 2.21V
pC

RC = 2 × 103 × 4 × 10−6 = 8 × 10−3 s


ar

dq q
(ii) dT + RC = VR0 sinωt
Sh

∴ (D + RC 1
)q = VR0 sinωt
V0 sinωt
⇒ q = R D+ 1
RC
1
V0 (D− RC )
= R (D2 − 21 2 ) sinωt [ You must know the way of solving differential equation ]
R C
1
V0 sinωt−ωcosωt
= R
RC
ω 2 + 21 2
+ ke−t/RC
R C
q(0) = 0
and V= q/C
1
V0 sinωt−ωcosωt V0 ωe−t/RC
∴ ϑ(t) = R
RC
ω 2 + 21 2
+ RC(ω 2 + 21 2 )
R C R C

50000 −t/0.008
= 500[ 125sin100t−100cos100t
25625 ] + 25625 e

= 2.44sin100t − 1.95cos100t + 1.95e−125t

i(t) = eϑ̇(t) = 97.56cos100t + 78sin100t − 97.5e3−125t

∴ Energy = 12 e(V (0.01))2 = 4.85µJ

Q.5. In the circuit diagram below arrangement is made for three phase power measurement
by 2 watt-meter method
(I) Derive and show that the sum of the two watt-meter readings is equal to three phase power
delivered to the load ,consider phase A voltage as reference and ABC as phase sequence .

144
(II) with Z∠θ = 20∠45◦ Ω balanced voltage supply of 100 V ,50 Hz and ABC as phase sequence
, obtain the 2 watt-meter readings for the connection as in the diagram and the total reactive
power drawn form the source.

e
ki
5. Power measured by wattmeter
oo

∗) + V
= (VAC )(I√A ∗
BC IB √
pC

VA = V0 / 2∠0 Vc = V0 / 2∠ − 240◦

√ √
ar

VB = V√ − 120◦ V0 / 2 = 100V
0 / 2∠√
VAB = 3V0 / 2∠30◦
Sh

√ √
VBC = 3V0 / 2∠ − 90◦
√ √ √
− √3V0
VCA = 3V0 / 2∠ − 210◦ VAC = 2

converting to equivalent star impedence Zeq = Z/3∠θ


IA = √3V3Z
0
∠ − θ − 120◦

3V0
Ic =√
3Z
∠ − θ − 240◦
IB = √3V3Z
0
∠ − θ − 120◦

∗ ) = Re(−100∠ − 210◦ )( √
ω1 = Re(VAC IA 3×100
∠45◦ )
3×20

= 836.5W
∗ ) = Re(−100∠ − 90◦ )( √
Re(VBC IB 3×100
∠120◦ + 45◦ )
3×20

= 224.14W
2 |X|
Also total reactive power =3Ireac
3×100 20
= 3( √3×20 )2 × 3√2 = 1060.66V AR

Q. 6.a)

145
6.a)

( S1 − 1) = 11.025

120×60 1800
ηs = 6 = 1800rpm ω = 60 × 2π = 60π
e
ki
s = 0.02
oo

(59.4j)(1.1+0.103j)
RT h = (1.1+59.503j) = 1.096 + 0.123j
pC

460 59.4j
VT h = √ × 1.1+59.503j ≈ 265.076V
ar

3
3VT2h
Sh

r2
∴ PG = r2 2
(RT h + s ) +(XT h +x2 )2
× s

3×(265.076)2 ×(0.225/0.02)
= (1.096+11.25)2 +(0.123+1.13)2
≈ 15.4KW
(1−s)PG PG
∴ Torque = ωr = ωs = 15399.68 = 81.7N m

Rotor loss = 3I 2 R = sPa ≈ 308W

Mechanical Power = PG − Rotor loss


= 15399.68 − 308)W ≈ 15.1KW
(59.4j)(11.25+1.13j)
Znet = 1.1 + 0.103j + (11.25+60.53)

= 11.572 + 3.158j = 11.995∠15.265◦


3VP2h (460)2
∴ Pinput = Rnet =3× 3×11.572 ≈ 18.286KW

∴ %ef f iciency ≈ (15.1/18.286) ≈ 82.6%

Q.6.b)A three phase squirrel cage induction machine operating at rated voltage and frequency has a starting
torque of 135% and a maximum torque of 220% both with respect to its rated load torque . Neglecting the
effects of stator resistance , core losses and rotational losses and assuming a fixed rotor resistance , determine

146
the slip at maximum torque,with 0 ≤ Smax ≤ 1 [8]

2Tmax
6.b) T = S S
ST max
+ T max
S
2×2.2×Tr
∴ 1.35Tr = 1 Sm
ST max
+ 1
or ST max = 2.9, 0.034
0 ≤ ST max < 1
∴ ST max = 0.34 = 34%

e
ki
oo
pC
ar
Sh

147
ELECTRICAL TECHNOLOGY
END - SPRING SEMESTER EXAMINATION2015-2016

e
ki
ANSWER:
oo

1.a)For RT h
pC
ar
Sh

∴ Rth = 6×3 4×2


9 + 6
4 10
= 2 + 3 = 3 = 3.33Ω

For VT h

148
Rnet = 3 + 6×6
12 = 6
∴ I = 4A
∴ V1 = 24 − 12 = 12
V2 = V1 − 2 × 2 = 12 − 4 = 8
∴ VT h = 24 − V2 = 24 − 8 = 16V e
ki
100−(150+130j) −50−130j
= 28.92∠ − 106.27◦
oo

b)I = 6(−12j) = 4.8−2.4j+2j


2j+ 6−12j
Voltage by wattmeter
pC

= 100 − (28.92∠ − 106.27◦ )(2∠90◦ ) ≈ 47.34∠20.02◦


∴ Wattmeter readings = Re((47.34∠20.02◦ )(28.92∠ − 106.27)
ar

= −810W
Sh

149
2.a)

A = 4 × 10−4 m2
φ = 2 × 10−3
−2
∴ R1 = (5+5+15)×10
−2
µr µA
R2 = µ15r µA , R3 = (5+5+10)×10
−2 e
ki
µr µA
∴ VCD = R3 × φ
oo

−2
= 20×10 −3
µr µA × 2 × 10
pC

∴ φR2 R2 = φR3 R3
−2 20×10−2
Or 15×10
µr µA × φR2 = µr µA × 2
ar

⇒ φR2 = 20 15 × 2 = 8/3
Sh

∴ φR1 = φR2 + φR3 = (2 + 83 ) × 10−3 = 14 3 × 10


−3

∴ VAB = φR1 × R1 + R3 × φR3


= 14 −3 × 20×10−2 + 2 × 10−3 × 20×10−2
3 × 10 µr µA µr µA
= ( 400 −5 1
3 × 10 ) × 4×10−4 ×4π×10−7 ×600
1 4×10−5
I= 500 ( 288π×10−11 = 8.842A

b) Power = (17.55 + 12.25j)


Other Power = 10 -6.19j
∴√Total Power = 27.55 -6.058j
∴ √3VL IL cosφ = Total power
or 3 × 440 × IL × cos(tan−1 ( 6.053
27.55 ) = 27.55 × 10
3

or 3 × 440 × IL × 0.9767 = 24.55 × 103
or IL = 38A

150
5000
ANSWER: 3.a) I1 = 100 = 50

e
ki
oo
pC
ar
Sh

∴ (i) Total Input current= (50 - 12.5) = 37.5A Output current = 12.5A
(ii)kV A rating = 37.5 × 100 = 3750V = 3.75kV A
∴ kVA transformed = ( 5 -3.75 )kVA = 1.25kVA
Current in primary coil = 50 A
current in the 2nd coil = 12.5A
(iii) Shown above. 3.b) Please consult your Professor.

4.a) η = 0.88
load = 60kW
Ptotal = 68.182kW PG = air gap power
stator loss + iron loss =( Ptotal − PG )
rotor loss = sPG
iron loss = sPG
no load loss = core loss (iron loss) +stator loss

151
∴ mechanical loss = 14 (Ptotal − PG ) = Pmech
∴ Pload + Pmech + Protor = PG
⇒ 60kW + 14 (68.182 − PG ) + sPG = PG
Also iron loss = sPG = stator loss
∴ Ptotal − PG = 2sPG

⇒ PG (2s + 1) = Ptotal
PG
∴ 60kW + 68.182
4 kW − 4 = (1 − s)PG
⇒ 77.0455 = PG (1 − s + (1/4))
77.0455 = 68.182 5
2s+1 ( 4 − s)
⇒ 1.13(2s + 1) = 1.25 − s
⇒ 2.26s + 1.13 = 1.25 − s
⇒ 3.26s = 0.12
∴ s ≈ 0.0368 = 3.68percent
4. b)

e
ki
oo
pC
ar
Sh

load =0.12( 1s − 1) ≈ 3.31


s = 0.035
P = 2000HP (output) = 1470998W
∴ Net Impedence =( 0.02 = 0.32j) + 451.2 k50jk(3.43 +0.32j) = 3.367 + 0.859j = 3.475∠14.32◦
2300
∴ current = √3×3.475∠14.32 √ ∠ − 14.32◦ = 382.2∠ − 14.32◦
662
◦ ≈ 3

∴ Input
√ power = 3 × V L × I L × cosφ
= 3 × 2300 × 382.2 × cos(14.32◦ ) ≈ 1475292W

VAB = 2.3/ 3∠00 − (382.2∠ − 14.32◦ )(0.342∠69.44◦ ) = 1257.74∠ − 4.891◦
∴ I1 = VAB /(3.43 + 0.32j) = 365.10∠ − 10.221◦
∴ Poutput = 3 × (365.1)2 × 3.31 = 1240320
∴ efficiency = 1240320
1470998 ≈ 84.3percent

8AM to 1PM :65kW,45kVAR,79.1kVA ,η = 0.791


1PM to 6PM :80kW,50kVAR,94.34kVA ,η = 0.9434
6PM to 1AM :30kW,80kVAR,42.43kVA ,η = 0.4243
1AM to 8AM : No Load,η = 0

65×5+80×5+30×7
ef f iciency = 65×5+80×5+30×7+(0.7912 ×5+0.9342 ×5+0.42432 ×7)×1.2+0.37
935
= 935+10.5+0.37 ≈ 98.85percent

120×f 120×50 3000


b) (i) ηs = P = 2k = k

152
Now P = 2k, where kN
Nearest no. to 725 rpm ≈ 4
∴ ηs = 750
∴ s = 750−725
750 ≈ 3.33%
∴ No. of poles P = 8
frequency of rotor current = relative frequency = s(50Hz)
25
= 750 × 50 = 1.67Hz
(ii) ηs = 120×f
P = 120×50
24 = 250rpm
∵ There is no stator resistance and core reactance, e
ki
PG = Psupply
oo

s = 250−247
250 = 0.012
∴ For STmax , S = r2 /x2 = 0.016
0.265 = 0.0604
pC

∴ Tc = s2Tcmax+ smax
smax s
ar

Tc 2
∴ Tmax = 0.012
+ 0.0604
Sh

0.0604 0.012
or Tmax
Tc = 5.232
2 = 2.616

NOT IN SYLLABUS

153
ELECTRICAL TECHNOLOGY
END - AUTUMN SEMESTER EXAMINATION2015-2016

e
ki
oo
pC

ANSWER:
ar

1.a) ω = 2π(50) = 100π


Sh

1
(1) At resonance ,ωL = ωC
6
⇒ L = ω21C = (100π)102 ×15.91 = 0.63684H
I = VR = 10A∠0◦
∴ VL = (10∠0◦ )(ωL)∠90◦
= (10∠0◦ )(0.63684 × 100π)∠90◦
= 2000.7∠90◦ ≈ 2kV ∠90◦
(ii) At resonance ,maximum current flows.
∴ Voltage VL is maximum
V0
Vc = q 1
1
× ωC VL = q V0 ωL 1 2
R2 +(ωL− ωC )2 R2 +(ωL− ωC )
V0 ω
For VL = q
1 2
R2 +(ωL− ωC )
L −→ α
V0 ω
∴ Vα = √ 2 = V0
ω

154
e
ki
oo
pC
ar

b) I (1000) 60
Sh

60
Or I = 1000 = 0.006A = 60mA I = −60mA ∴ At point A
∴ Power absorbed = 60 × 110 × 10−3 W = 6.6W

155
e
ki
ANSWER:
oo

For OC test a) I 2 R = W
pC

or R = 710/(9.6)2
= 7.704Ω
ar

Z = V /I ≈ 23Ω
Sh

X = 21.67Ω
∴ Referred to HV side
R= 7.704 × 4
= 30.82Ω
and X = 21.67 × 4
= 86.68Ω
from HV side :

For SC test
1030 = (56.8)2 × R
or R = 0.32Ω

156
Z = 42/56.8 ≈ 0.74Ω
∴ X = 0.67Ω
∴ Referred to LV
R = 0.32/4 = 0.08Ω
X = 0.67/4 = 0.1675Ω
For LV side

b)

e
ki
oo
pC
ar
Sh

3
I = 25×10
440 ≈ 56.82
φ = cos−1 (0.8) ≈ 37◦
∴ I = 56.82∠ − 37◦

VLV = 440∠0◦ − (56.82∠ − 37◦ )(0.32 + 0.67j)


≈ 403.04∠ − 2.77◦
∴ VLV = 403.04/4 ≈ 100.76V

157
c) [Every sum is referred to the HV side]

VLV = 440∠0◦ + (56.82∠ − 37◦ )(0.32 + 0.67j)


≈ 447.83∠2.33◦
∴ VHV = 477.83
xScosθ e
ki
d) η = xScosθ+x 2 P +P
c i
maximum at cosθ =p1
oo

p
and x = Pi /Pc = 710/1030 = 0.83
pC

0.83×25
∴ η = 0.83×25+2×0.71 ≈ 93.6%
kV A = 0.83 × 25kV A = 20.75kV A
ar
Sh

4.a) (i) ηs = 120×f


P = 120×50
4 = 1500
1500−1450 50 1
∴ s = 1500 = 1500 = 30 = 3.33%
1
(ii) frequency of the rotor current = 30×1500 = 50
1500
(iii) velocity w.r.t stator = 60 × 2π ≈ 157rad/s[SWs ]
(iv) velocity w.r.t rotor = 5.236 rad/s
velocity w.r.t stator = 157 rad/s
Both stator field and rotor field is the rotating magnetic field .

b) ns = 120×f
p = 1500rpm
r2
STmax = x2 = 0.025
0.12 = 0.208

158
∴ nr = (1 − s)ns = 1187.5rpm
Neglecting stator resistance , and at start , s=1
Tmax (0.75) = Tstart
∴ Tc = S2T maxSmax
Smax
+ S
2Tmax
⇒ Tstart = Smax + S 1
max
1 2
or Smax + Smax = 0.75= 83
2
or 2Smax − 3Smax + 3 = 0
Smax = 2.22/0.45
∵S<1
∴ STmax = 0.45 or r2 = 0.45 × 0.12 = 0.054Ω
∴ additional rotor resistance =0.029Ω

e
ki
oo
pC
ar
Sh

please consult your respective professor

(i) Given SWs


∴ SWs = 3.1416 = π
or Ws = 25π

159
∴ rpm
60 × 2π = 25π
or rpm = 25 × 30 = 750
120×f
P = 750
⇒ P = 120×50
750 = 8
∴ 8poles (ii) S=005
Tmax = 2.5Toperate
T = S 2Tmax ST
max
ST
+ 0.05
max
or + S0.05
0.05
Smax
max
=5
2 2
or (0.05) + Smax = 2 × 0.05 × Smax
2
or Smax − 0.25Smax + 0.0025 = 0
or Smax ≈ 0.2396/0.0104

e
ki
oo
pC

Operating zone is on the right of STmax


∴ STmax = 0.0104
ar

∴ Tstart = 12×T+0.0104
max
[s = 1]
Sh

0.0104
Tstart
Or Tmax = 0.02079 ≈ 2.08%

160
6.a) liron = lsteel = π(20)
2 − 0.02 = 31.396cm
liron lsteel lair
∴ Rtotal = µiron µ0 A + µiron µ0 A + µ0 A
1 liron lsteel
∴ NI = µ0 A ( µiron + µsteel + lair )φ
8×10 −4 1 1 0.2×10−3 ×2
= 4π×10−7 ×10×10−4
(0.31396( 800 + 165 ) + 1 )
= 1715.842

b) Output power = 37kW


∴Input
√ Power =41.11 kW
∴ VL IL cosφ = 370
9 × 1000
⇒ IL = 67.43A
IL = 67.43A∠ − 37◦

e
ki
oo
pC
ar

√ ωc∠90◦
Ic = 4403
Sh

IL + Ic ∠90◦ = Inet ∠25.842◦


⇒ 67.43∠ − 37◦ + Ic ∠90◦ = Inet ∠25.842◦
⇒ 53.852 − 40.58j + j(Ic ) = Inet (0.8999 + j(0.43589))
⇒ 53.852 + (Ic − 40.58) = Inet (0.8999 + j(0.43589))
−40.58
∴ Ic53.852 = 0.4843
⇒ Ic = 66.662
∴ 66.662 = 440
√ × 2 × π × 50 × c
3
⇒ c = 0.8353mF

161
Mid-sem 2017-2018
Spring Semester
MID-SEMESTER EXAMINATION
CS10001: Programming and Data Structure (Solutions)
Question 1: [2.5+2.5+2=7]

(a) Interpret 11011011 as a two's complement binary number, and give its
decimal equivalent.
Answer:

First, note that the number is negative, since it starts with a 1.

Change the sign to get the magnitude of the number.

1 1 0 1 1 0 1 1
¬ 0 0 1 0 0 1 0 0
+ 1
0 0 1 0 0 1 0 1
Convert the magnitude to decimal: 001001012 = 2516 = 2×16 + 5 = 3710.

Since the original number was negative, the final result is -37.

(b) Show how a computer would perform 10 + -3 using eight bit two’s
complement representation. Is there a carry? Is there an overflow?
ie
Answer:
ok

10 + -3 = 7:
o

1 1 1 1 1
pC

0 0 0 0 1 0 1 0
+ 1 1 1 1 1 1 0 1
ar

0 0 0 0 0 1 1 1
Sh

Carryout without overflow. Sum is correct.

(c) Convert the 8­bit signed two’s complement hex number 0x3F to
decimal.

Answer: Easiest is to convert to binary: 0011 1111. Then convert to


decimal: 32 + 16 + 8 + 4 + 2 + 1 = 63, which is the answer

Question 2: [4+4=8]

(a) Convert -35.75 to IEEE 754 floating point format. What is the hex
representation of the resultant bit pattern?

Answer: First, we need to write down the binary representation of


35.75: 100011.11
Next, we need to put normalize it: 1.0001111 * 25
Now we can get the 3 pieces of the number we need:
S: 1 (negative)
E: 132 (127 + 5) = 1000 0100
M: 000 1111 0000 0000 0000 0000
Now we just put all the bits together: 1100 0010 0000 1111 0000 0000
0000 0000 and convert each group of 4 to a hex digit:
Answer: 0xC20F0000

162
(b) Convert the IEEE format floating point number 0x40200000 to decimal.
Answer: First, convert the hex to binary: 0100 0000 0010 0000 0000
0000 0000 0000
Then pull out each of the 3 pieces:
S: 0 (positive)
E: 1000 0000 = 128. Taking 128 - 127 = 1
M: 010 0000 0000 0000 0000 0000 So we have 1.01 * 21 = 10.1
Then convert that to decimal to get the answer: 2.5

Question 3:

What is the output of the following C Programs? [3+3+3 = 9]

(a) #include <stdio.h>

int main()

int a = 10, b = 20;

int *p = &a, *q = &b;


ie
printf("%d ", (*p)++);
ok

printf("%d ", --(*q));


o
pC

printf("%d\n", a+b);

return 0;
ar

Underlying Idea:( checking whether you


understand postfix prefix):
Sh

} 1.*p is first printed and then incremented


Answer: 10 19 30 by 1 because the operator is postfix.
2. 2. q is first decremented then printed
because the operator is prefix.
3. the updated values of a and b i.e. *p
and *q are added which are 11 and 19
respectively.

163
(b) #include<stdio.h>
Note: * in the fuction signature implies the fuction will return a
int *fun(int p, int *q)
pointer.]
{

p = 4;

*q = 2; The function is returning 2 to variable z and also the value


return (q); of y becomes 2 in the main function because q is a pointer
to variable y
}

int main()

int x = 6, y = 9, z = 3;

z = *fun(x, &y); Key Concept: value of x is passed while


address of y is passed so any changes
made in the respective formal
parameters(p and
*q) will be reflected at y but not at x.]
ie
o ok
pC
ar
Sh

164
printf("%d %d %d\n", x, y, z);

return 0;

}
Thus x remains unchanged i.e. 6 while z and y both become 2 as
Answer: 6 2 2
returned by the *fun function.

(c) #include<stdio.h>

int main()

float arr[5] = {1.5, 2.5, 3.5, 4.5, 5.5};

ptr1 is a pointer to the 2nd element in the array i.e. *ptr1 will have the value 2.5
float *ptr1 = &arr[1];

float *ptr2 = ptr1 + 3; ptr2 is a pointer to the 5th element of the array i.e. *ptr2 will have the value 5.5

float *ptr3 = ptr2 + 1;


ptr3=ptr2+1*sizeof(float). Go through pointer operation to learn more.
printf("%f ", *ptr2); ptr3 do not correspond to any element in the array as the
sizeof(arr)=5.
ie
*ptr2 will thus display 5.500000. (the extra 5 zeros are because float data
printf("%ld ", ptr2 - ptr1);
ok

type which in C displays 6 numbers after decimal point.

printf("%f\n", arr[ptr3 - ptr1]);


o
pC

ptr3 containsaddress of ptr1+4*sizeof(float) therefore ptr3-ptr1will result in


return 0; Ptr2-ptr1 is
3*sizeof(float) 4*sizeof(float) which will be scaled down to 4. Read pointer operations to know
ar

which will be
} scaled down to 3. more.
Sh

Answer: 5.500000 3 5.500000

Question 4:

A lazy programmer wants to write a program that will perform two


activities together -- (a) reverse the elements of an integer array, and
(b) sum the elements of the array. (S)he thought of writing an
additional function to exchange the values of two variables. However,
(s)he feels that it can be done without introducing any temporary
variable within that function, exchange(int *p, int *q), which takes two
pointers p and q containing the addresses/locations of two integer
elements of an array and then it exchanges the values in those two
locations. With this thought, (s)he started writing the program, but did
not complete it. Help the lazy programmer to complete the program
correctly. The part of the code is provided below. [1.5 x 4 = 6]

#include<stdio.h>

void exchange(int *p, int This is a swapping function and since the addresses of
*q) the values to be swapped has been passed so the
swapped values will reflect on the actual parameters
{ as well.

*p = *p + *q;

*q = *p - *q ;

165
*p = *p - *q ; Simple swapping without third variable logic has been used.

int main()

// defining an array

int a[] = {7,4,8,2,9,0,1,3,6,5};

// marking the initial index of the array

int i=0;

// marking the final index of the array

int f = sizeof(a)/sizeof(int) - 1;
f is storing the last index of the given array
// initializing the sum
i.e. 9
int sum = 0;

while ( i < f ) // converge from both ends of the array index


{ Since the traversal is happening from both end the i<f marks that all the elements have been swapped.
ie
// updating the sum (adding two elements of the array at a time)
ok

sum = sum + a[i] + a[f] ; adding elements from the initial end and the back end
o

exchange( &a[i] , &a[f] ); // call to the exchange function


pC

Swapping
i = i + 1 ; // incrementing initial index
ar

f = f - 1 ; // decrementing final index By incrementing i and decrementing f the


Sh

array is being converged from both


} ends.

return 0;

Question 5:

The following program finds an approximate value of a definite integral.


Let l be the left and r the right boundary for the integral. Also let h
be the step size. The idea is to break the interval [l, r] into sub-
intervals [l, l + h], [l + h, l + 2h], [l + 2h, l + 3h], . . . , [r − h,
r]. Assume that r − l is an integral multiple of h. The program
evaluates the function to be integrated at the center of each interval,
multiplies these values by the width h and computes the sum of these
products as the approximate integral. Suppose you are integrating the
function x2. Complete the following program. Each blank can have at most
one statement. [ 1 + 2 = 3 ]

#include <stdio.h>

int main ()

166
{

double l, r, h, x, s;

printf("Enter left boundary : "); scanf("%lf", &l);

printf("Enter right boundary : "); scanf("%lf", &r);

printf("Enter step size : "); scanf("%lf", &h);

s = 0;
for f(x)=x*x the functional value at the mid of the small
for (x = l; x < r ; x += h){
regions is (x+h/2)*(x+h/2) and that multiplied by the width h
s += h*(x + h/2)*(x + h/2) ; and is totaled and stored in s. s+=h*(x+h/2)*(x+h/2) is
equivalent to s=s+h*(x+h/2)*(x+h/2)
}

printf("Integral = %lf\n", s);

return 0;

Question 6:
ie
The following program attempts to find the roots of the polynomial x3 −
ok

4x2 − 4x + 16. It uses the fact that the product of the roots is the
negative of the constant term of the polynomial. For instance, x3 − 4x2 −
o

4x + 16 has roots, 2, −2, 4, and the product of the roots: 2 × −2 × 4 =


pC

−16 (negative of the constant term 16). To find the roots, the program
checks for each integer r if it is a factor of the constant term 16, and
ar

if so, whether replacing x by the integer r in the polynomial evaluates


Sh

to zero. For instance, since 4 is a root, 43 − 4 × 42 − 4 × 4 + 16 = 0.

Complete the following program. Each blank can have at most one
statement. [1 + 2 + 1 = 4]

#include <stdio.h>

int main ()

int r, nroot = 0;
nroot<3 is the limiting condition because there
for (r=1; nroot<3; r++) { are 3 roots of the given equation.
if (16 % r == 0) {

if (r*r*r-4r*r-4*r+16==0) {

printf("Root found : %d\n",r);

nroot++ ; if a root is found then the nroot is incremented by 1 .

if ( -r*r*r - 4r*r + 4*r + 16 == 0 ) {


Replacing –r in the function
we must get 0 if –r is a root
of the function

167
printf("Root found : %d\n",-r);

nroot++ ;
if a root is found then the nroot is
} incremented by 1 .

return 0;

Question 7:

The following program randomly generates a sequence of integers between


-8 and +91 and outputs the maximum and minimum values generated so far.
It exits if a negative integer is generated. The program does not store
the sequence generated (say, in an array), but updates the maximum and
minimum values on the fly, as soon as a new entry is generated. Complete
the following program. Each blank can have at most one statement.[1×4=4]

Hint: To generate a random number between 0 to n, you can use


ie
rand()%(n+1)
ok

So, for example, you can get a random number between 5 to 90 by --


o

rand()%86 + 5
pC

#include <stdio.h>
ar

#include <stdlib.h>
Sh

#include <time.h>

int main ()

int a, i, max, min;

i = 1;
use the logic in the example on how to generate random
numbers between 5 and 90 given above. rand()%100 will
while (1) generate random numbers between 0 and 99 and thus
{ substracting 8 to it will generate random numbers between
a = rand()%100 – 8 ; -8 and +91.

//a stores a random number between -8 to +91

printf("Iteration %d: new entry = %d, ", i, a);

if ( a < 0 ) { if the generated value is a negative number(a<0) then exiting.

printf("...exiting...\n\n\n"); break;

168
for the first iteration the value generated is assigned to
both max and min for furthur comparisions.
if (i == 1) max = min = a ;

else { if the new generated value is more than the current max then
if (a > max) max = a; max is the new generated value i.e. max=a.

if (a < min) min = a ; if the new generated value is less than the current min then
min is the new generated value i.e. min=a.
}

printf("max = %d, min = %d\n", max, min);

++i;

return 0;

Question 8:

What will be the outputs of the following programs? [2 + 2 = 4]

(a) #include<stdio.h>
ie
int main()
ok

{
o
pC

int x = 0, y = 10, z = 20;


ar

while (1) {
Sh

x++; iteration 1: x=1;y=14 and z=22


iteration 2: x=2;y=22 and z=26
if (y > z) break;
iteration 3: x=3;y=34 and z=32
y += 4*x; z += 2*x; iteration 4: x=4 and since y>z so .breaks
}

printf("x = %d, y = %d, z = %d", x, y, z);

return 0;

Answer: x = 4, y = 34, z = 32

(b) #include<stdio.h>

int main()

int s = 0;

while (s++ < 10) { first s will be compared with the logical statement s<10 and the s will
be incremented. The highest number satisfying <10 is 9 but when s=9
after comparing with <10 s will be incremented and become 10.

169
when (s<4) which implies s<9 as well the
loop will interate without proceeding
if ((s < 4) && (s < 9)) continue; furthur.
printf("%d ", s);

} therefore value of s>=4 upto the loop terminates will be


printed. That is 4 5 6 7 8 9 10
return 0;

Answer: 4 5 6 7 8 9 10

Question 9:

What will be the output of the following C programs? Only one of the 4
choices are correct. Circle only the correct choice. [5x2 = 10]

(a) int fun(int *a,int n) The function fun returns *a and not a so its return
type is int not int *. (don’t get confused)
{

n++; Value of n is incremented and is now 12

*a = *a + n;
*a=11+12=23
return *a;
}
ie
this n in the formal parameter is a local variable to
this function Therefore changes made to this n won't be
int main()
ok

reflected to n in main function and for that matter to


*a.
{
o
pC

int n=10;
n is incremented by 1 and address of n and value of n is passed to
ar

n = fun(&n,++n); function fun


Sh

printf("%d",n);
n receives the value of *a and is therefore 23
return 0;

A. 20 B. 21 C. 22 D. 23

here r is assigned
21.
B. #include <stdio.h>
int n = 17, r=35; here n is 17( the globally declared
value) since there is no local
void my_disp_func( int r ) { printf("%d,%d
variable with the name n.
\n",n,r); } int main ()

int n = 21, r=14; GENERAL NOTE: If there is a global and a


local variable with the same name then the
value of the local variable will be considered
while inside the function

170
my_disp_func(n);

A. 17,21 B. 21,14 C. 17,35 D. 21,35

(c) void h( int A[], int n ) Any changes made to A[] is reflected to A[] in main
{ function as well because array is always called by
reference type.
int i;

for (i = 1; i < n; ++i) A[1]=A[1]*A[0]=2*1=2

A[i] *= A[i-1]; A[2]=A[2]*A[1]=3*2=6

A[3]=A[3]*A[2]=4*6=24
}

int main ()
the loop runs for
{ i=1,2,3 since n=4.

int A[5] = {1,2,3,4,48};

h(A,4);

printf("%d", A[4]/A[3]);
ie
A[4] is unchanged and is equal to 48 and the modified value of
A[3]=24 as shown above so A[4]/A[3]=48/24=2
ok

return 0;
o
pC

A. 16 B. 48 C. 2 D. 4
ar
Sh

(d) #include <stdio.h>

int f(int i)

return i%2; Will return 1 if i is odd else returns 0 if i is even.

int main()

int i=27;
As soon as i becomes a even number the function retuns 0 and the loop
while(f(i)) exits.
{
iteration 1: i=27
printf("%d", i);
iteration 2: i=13
} i = i/2; iteration 3: i=6 so I becomes even and
therefore the function returns 0 and the loop
exits.

171
notice no space is
there after %d so
2713 will be printed
and not 27 13
return 0;

A. 27 B. 27 13 C. 2713 D. 1

(e) #include <stdio.h>

void jumble(char A[], int size)


ideally to reverse
{ the word completely
i<size/2 but since
int i; i<size is given
A[0]=A[9], A[1]=A[8],
for(i=0;i<size;i++) A[2]=A[7],A[3]=A[6]
and A[4]=A[5] happens
A[i] = A[size - i -1]; twice and so the
} second half of array is
int main() mirrored of the 1st
half.
{

char A[] = "REMARKABLE";

jumble(A, 10);
ie
ok

printf("%s", A); The first and the last element of the array will be
swaped and since i doesnot stop at size/2 so after
o

return 0; i=4 the array element will be mirrored.


pC

}
ar

A. REMARKABLE B. ELBAKRAMER C. ELBAKKABLE D. None of the above


Sh

Question 10:

A natural number N is called a 'perfect number' if the divisors of N


that are less than N add to N. Example: 6 is a perfect number because
the divisors of 6 that are less than 6 (, namely 1,2, and 3) add to 6.
Given below is a program that prints all perfect numbers in a given
range using functions. Fill in the blanks. [1 x 5 = 5]

/**

* C program to print all perfect numbers in given range using function

*/

#include <stdio.h>

/* Function declarations */

int isPerfect(int num);

172
void printPerfect(int start, int end);

int main()

int start, end;

/* Input lower and upper limit to print perfect numbers */

printf("Enter lower limit to print perfect numbers: ");

scanf("%d", &start);

printf("Enter upper limit to print perfect numbers: ");

scanf("%d", &end);

printf("All perfect numbers between %d to %d are: \n", start, end);

printPerfect(start, end);
ie
ok

return 0;
o
pC

}
ar
Sh

/**

* Check whether the given number is perfect or not.

* Returns 1 if the number is perfect and 0 otherwise.

*/

int isPerfect(int num)

int i, sum;

/* Finds sum of all proper divisors */

sum = 0;

for(i=1; i<num; i++)

if( num%i == 0 )
checking whether i is a factor of num

173
sum = sum + i ; if i is a factor then adding it to the sum
}

}
if( sum == num ) checking whether the sum of factors of num equals num . if they are
found to be equal the then num is perfect and returns 1.
return 1;

else

return 0;

/**

* Print all perfect numbers between given range start and end.

*/

void printPerfect(int start, int end)

/* Iterates from start to end */


ie
ok

while(start <= end)


o

{
Checking whether start is perfect or not. if perfect then the
pC

if( isPerfect(start) ) function returns 1 and the if statement is executed i.e. the value of
start is printed.
ar

{
Sh

printf("%d, ", start);

start++ ;
incrementing start.
}

174
MID-SEM 2017-2018
AUTUMN SEMESTER
Indian Institute of Technology Kharagpur
Programming and Data Structures (CS10001)
Autumn 2017-18: Mid-Semester Examination
Time: 2 Hours Full Marks: 60

INSTRUCTIONS
1. Answer ALL questions
2. Please write the answers either within the boxes provided or on the blank lines to be filled up. Any
answer written elsewhere will not be evaluated.
3. You may use the last two blank pages for your rough works.

Q.1. Answer the following questions as directed.

a) What will get displayed when the following program is executed? [1]

#include <stdio.h>
int main() {
e result=5
int x = 2, y = 17, result = 5;
ki
Equivalent to result = result-(x/5  13  y/3 
result -= x/5  13  y/3  x;
oo

x). x=2 is an integer so when divided by 5 will


printf(“result=%d\n”, result); result in 0.(integer data type truncates the
pC

return 0; decimal part). So the whole product becomes


} 0 and thus result=result-0. Thus result=5
ar
Sh

b) What will get displayed when the following program is executed? [1]

#include <stdio.h>
int main() {
int x = -5, y = 10;
if (x > y) x = 1; x=-10
else if (y < 0) x = x  (-1);
else x = 2  x;
printf(“x=%d\n”, x);
return 0;
}

c) What is the 8-bit two’s complement representation of the decimal number 37? [2]

11011011 First write the 8bit binary equivalent of 37=00100101. Then


compliment the entire binary equivalent which will then look like
11011010. Add 1 to this complimented number to get 2’s
compliment of -37=11011011.

175
d) What will get displayed when the following program is executed? [1]

#include <stdio.h> a=’a’. so although a<=’c’ is true but


a>’a’ is false. And since && operator is
int main() {
used so (a>’a’)&&(a<=’c’) results in
char a = 'a'; false and the loop is thus not executed.
while ((a > 'a') && (a <= 'c')) a++;
a
So just the a is printed as it was i.e. a.
printf(“%c\n”, a); Note: the a++ is a part of the loop. If a
curly braces is not given then after a
return 0; loop,only the first statement is considered a
} part of the loop by default.

e) What will get displayed when the following program is executed? [2]

#include <stdio.h>
int main() {
int sum = 1, index = 9;
sum=2, index=8
do {
index = index - 1; It’s do while loop so at first 1 iteration is executed
and then the condition is checked. Iteration1:
sum = 2  sum;
index=8 and sum=2. Now index>9 is false so exits.
} while (index > 9);
printf(“sum=%d, index=%d\n”, sum, index);
return 0;
e
ki
}
oo
pC

f) What value will the following function return when called as recur(3)? [2]
ar

int recur(int data) { 0


Recur(3)
Sh

if (data > 2)
Recur(2) Recur(1)
return (recur(data - 1) - recur(data - 2));
else return 1;
} Returns 1 Returns 1

Thus 1-1=0
g) What value will the following function return when called as g(1024)? [1]

int g(int n) { 1 g(1024) g(512) ………


if (n < 2) return n; g(2) g(1)
return g(n/2);
} Rreturns 1

h) What is the binary number corresponding to the hexadecimal number C5.75? [2]

Write 4 bit binary equivalent of each.


11000101.01110101 C(12)-1100;
5-0101;
7-0111; Write them in order as they are,
replacing them by the 4 bit binary numbers
respectively
11000101.01110101

176
i) Consider the program segment given below to read a letter from a..z and A..Z from the keyboard
and convert it to uppercase if not already so. It is assumed that the user will only input a character
from a..z and A..Z. Fill up the missing line with a single C expression so that the variable ch will
contain the character in uppercase. Do not use any library functions. [2]

char ch; basic problem of ternary operator.


ch = getchar();

ch = (ch>=’A’)&&(ch<=’Z’)?ch:’A’+ch-‘a’;

j) The following program segment is supposed to check whether the values stored by three integer
variables a, b, and c are in ascending order. However, it contains an error. Encircle the part of the
program that contains the error and write only that part corrected. [1]

if (a < b < c)
printf(“Numbers in ascending order \n”);
((a<b) && (b<c))
else printf(“Not in ascending order\n”);

e
ki
oo

Q.2. Answer the following questions as directed.


pC

a) What will get displayed when the following program is executed? [2]
ar

#include <stdio.h>
-1
Sh

In the loop i is first initialized by 1 and then


int main() { the control moves to the condition check area
int i; where instead of a condition i was assigned -1
for (i = 1; i = -1; i++) .Then control enters the loop and the
if (i < 5) break; condition is checked and is satisfied i.e. -1 <5
so the loop breaks and then prints the current
printf(“%d\n”, i); value of i i.e. -1.
return 0;
}

b) What will get displayed when the following program is executed? [2]

#include <stdio.h>
i=11, j=0
TRICKY QUESTION!!
void increment(int i) { The variable i in the function increment is just given
to confuse you. It’s a local variable with scope just in
i++; the function increment. It will play no role to alter i
} in the main function. So when i will be equal to 10 it
int main() { the loop will break but i will also get incremented by
1 as a postfix operator is used. So I will have value
int i = 0, j = 0; 10+1=11 and j remains same throughout the program
while (i++ < 10) increment(j); i.e. 0.
i=11, j=0
printf(“i=%d, j=%d\n”, i, j);
return 0;
}

177
c) What will get displayed when the following program is executed? [2]

#include <stdio.h>
int main() { n=6, j=64.000000
float j = 1.0, i = 2.0;
int n = 0;
When (i/j<0.05) then the loop will terminate.
while (i/j > 0.05) { n
Therefore, 2/2 <0.05. This is true only for
j = j + j;
n=6(minimum value of n). Thus n=6 and
n++;
j=26=64.000000. The 6 zeros are a must for
} you to write or else marks won’t be awarded.
printf(“n=%d, j=%f\n”, n, j); The six zeros after the decimal is because the
return 0; data type of j is float.
}

d) What will get displayed when the following function is called as f(2, 8)? [2]

int f(int x, int y) { 2 : 1 : 6 : 13 :


int sum = 0;
y--; e
ki
y decremented
if (x == 0) return 0;
oo
f(2,8) by 1. y=7. f(1,7)
sum=7+f(1,7)
else { y decremented sum=6+f(0,6)
pC

printf(“%d : “, x); Returns sum=6+0=6 by 1. y=6.

sum = y + f(x – 1, y);


ar

f(0,6) RETURNS 0
printf (“%d : “, sum);
Sh

} Therefore sum=6+0=6 in the function f(1,7) and then it


returns 6 to f(2,8) where sum =7+6=13..
return sum; At each step value of x is printed and at the end value of sum
} is printed all values are succeded by a : as given in the printf.

e) What will get displayed when the following program is executed? [2]

#include <stdio.h> sum=1683, i=102


int main() {
int sum = 0, i = 3;
I will be a multiple of 3 and greatest value of i
while (i < 100) { that will satisfy i<100 is 99. after that i=i+3=102
sum = sum + i; and the condition becomes false and loop exits.
i = i + 3; thus the value of i is 102.The sum will be a A.P.
} 3+6+9+…..99
=3*(1+2+….+33)
printf(“sum=%d, i=%d\n”, sum, i); =3*((33*34)/2)
return 0; =1683
}

178
Q.3.
a) A number is said to be perfect if it is the sum of all its factors (except itself). For example, 6 has
factors 1, 2, 3 and 1+2+3 = 6, hence it is perfect. Also, 28 = 1+2+4+7+14 is perfect. In the
following function checkPerfect fill up the missing lines so that it returns 1 if n is a perfect
number and 0 if n is not a perfect number. [2 + 2]

int checkPerfect(int n) {
int i, sum = 0;
for (i = 1; i < n; i++) { Checking whether i is a factor of n and if found true
adding it to find the sum of the factors.
if (n % i == 0 )
Checking if sum==n then the number is perfect and will
sum += i; result in 1 so the function will return 1 as asked else will
} return 0. Alternateive: n/sum is also a correct answer as sum
of factors cannot exceed the number and will be equal to the
return (sum == n ); number if it is perfect. so will return 1 if n is perfect else will
return 0 as sum<n so n/sum will yield 0.

b) The following function strEqual takes two strings S1 and S2 as parameters. Fill up the missing
lines in the function so that it returns 1 if the two strings are the same, 0 otherwise.
e
ki
[1 + 2 + 2]
oo

We have been asked to take two Strings


pC

int strEqual(char S1[], char S2[]) as parameters. In C strings are array of


{ characters.
ar

int i = 0;
Sh

/ Go on until the end of either of the strings /


Scanning through the strings letter by
while (s1[i] != '\0' && s2[i] != ’\0’) letter and checking whether they are
same or not. If not immediately the
{ strings are not same so returns 0.
if (S1[i] != S2[i]) return 0;
i++; If all the letters are same that does not
} mean the two strings are same. example
S1[]=“POST” and S2[]=“POSTAL”, in
if (s1[i] == '\0' && s2[i] =='\0') the loop both will be checked till i=3 in
return 1; and the ‘\0’ will be encountered in S1
else and loop will quit. But S1[4]=’\0’ but
return 0; S2[4]!=’\0’ so the strings are not same.
so checking after the loop breaks if both
} have null character at the same i it
implies they have same length and same
letters so the two strings are same and
returns 1 else 0.

179
Q.4.
a) The following recursive function find_power should return xn when called as find_power(x,n),
n being a non-negative integer. Fill up the missing lines in the function so that it returns xn.
[1 + 1 + 2]

float find_power(float x, int n) {

if (n == 0) Straightforward simple problem of power


calculation via recursion. continue till the
return 1; base case of n==0 is reached and then
return 1.
else
return x*find_power(x,n-1);
}

b) Fill up the missing lines in the following program so that it will display the sum of the elements
of the array A when executed. [2 + 2]

#include <stdio.h>
int main() {
e
ki
int i, n, k = 0, A[10], lim;
oo

printf("Enter number of elements ");


pC

scanf("%d", &n);
printf("Enter the elements ");
ar

for (i = 0; i < n; i++)


Sh

scanf("%d", &A[i]);
for (i = 0, lim = n/2; i < lim; i++) {

/ Accumulate Sum / The first and the last elements are added and
then the second and the second last and so on…
k = k + A[i]+A[n-i-1]; This way we can do the sum in n/2 iterations. If
n is odd then middle element is left.
}
if (lim <(n-lim)) / if middle element left out / If n is odd middle element is left
k = k + A[i]; out . if n is odd lim<(n-lim) or
printf("%d\n", k); you can also do n%2!=0 to check
whether n is odd or not. Both are
return 0; correct solutions.
}

180
Q.5.
a) The following program is supposed to insert a new integer value x into an already sorted (in
ascending order) array A containing n distinct integers. You can assume that x does not already
exist in A, and there is space available to insert x in A. For example, assume that n is 10, and A
has the elements 10, 20, 30, 40, 60, 70, 80, 90, 100, 110, and x is 56. After insertion of x, the
array would become 10, 20, 30, 40, 56, 60, 70, 80, 90, 100, 110, and n would be 11. Fill up the
missing lines in the program so that the program inserts x in the sorted array A.
[2 + 2 + 2 + 2]

#include <stdio.h>
int main(){
int x, i = 0, j, n, A[100];
scanf("%d%d", &n, &x);
for (j = 0; j < n; j++) scanf("%d", &A[j]);

while (x > A[i] && i < n) i++; / find position after which to insert /

for (j=n; j>= i+1; j--) / make space for inserting x /

A[j] = A[j-1]; Straightforward Insertion sort logic. The


e
ki
comments are self explanatory and is
n++; sufficient to understand and fill in the gaps.
oo

A[i] = x; / insert the element at the required place /


pC

for (i = 0; i < n; i++) printf("%d ", A[i]);


ar

return 0;
Sh

b) The following recursive function reverse takes as parameters an integer array A and two other
integers leftIndex and rightIndex which are indices of A. After the function returns, only the
part of the array from leftIndex to rightIndex (including both) should be reversed if leftIndex <
rightIndex. For example, if the elements of the array A are 1, 2, 3, 4, 5, 6, 7, and the function
call reverse(A,2,6) is made, then on return, the array A will contain 1, 2, 7, 6, 5, 4, 3 (i.e., A[0]
and A[1] remain unchanged, and A[2] to A[6] get reversed). Fill up the missing lines in the
function so that it reverses the part of the array A between leftIndex and rightIndex.
[1 + 3]
void reverse(int A[], int leftIndex, int rightIndex) {
int temp; leftIndex<=rightIndex will also be accepted as a
if (leftIndex < rightIndex) { solution. just in that case if the number of elements
to be swapped is odd then the middle element will
temp = A[leftIndex]; be swapped by itself which is not required. so the
A[leftIndex] = A[rightIndex]; given highlighted solution is better.
A[rightIndex] = temp;
reverse(A, leftIndex+1, rightIndex-1);
After swapping the leftmost
element with the right most
} element increasing the index of
} the left and decreasing that of
7 of 8
the right to get a converging
swap of the corresponding
elements of the array.
181
Q.6.
The function closest given below takes as parameters an integer array A, the number of elements n
in A, and an integer val. Assume that all integers in the array A are distinct and A is already sorted
in ascending order. The function returns the index of the element in A with minimum absolute
difference with val (i.e., it returns the index i such that |A[i] – val| is minimum). If more than one
element has the same minimum absolute difference with val, then it returns the smallest index. For
example, if A contains the elements 10, 13, 15, 19, 110 and val is 18, the function returns 3, which
is the index of 19 (as |19  18| is the minimum). However, if val is 14, it returns 1 (as both |15  14|
and |13 – 14| are the minimum, 13 occurs at index 1 and 15 at index 2, and 1 is the smaller index).
Fill up the missing lines in the function so that it does the above. [2 + 2 + 2]

int closest(int A[], int n, int val) {


int index, i;
if (val < A[0]) / smaller than the smallest element /
index = 0;
else if (val > A[n-1]) / larger than the largest element /
index = n - 1; e
ki
else { / find the elements closest to val /
oo

Notice the ; given after the loop. That is there is no body of


for (i = 0; A[i]<val ; i++) ; the loop. It is given to find the first i for which A[i] becomes
pC

>val. then the element closest to val will be i or i-1.

if ((A[i]-val)<(val-A[i-1]))
ar

A[i-1]<val and A[i]>val. so checking the absolute


Sh

differences to see which one is closer to val.


/ if current element closest /
index = i;
else index= i-1 ; / set index to the closest element /
if (A[i]-val)>(val-A[i-1] ) then i-1th index has element
closer to val also if both are equal the answer is i-1
} because it’s the smallest index.
return index;
}

182
MID SEMESTER 2016-2017
(Spring Semester)
1. (9 marks) Write C statements ( program segments only) of a program that reads the lengths of the sides
of a triangle to find the nature of the triangle. You are required to only write program segments for the
following tasks only and not the complete program. Marks: 1 + 1 + 2 + 2 + 3
(a) Declare variables a, b and c of type float.

float a,b,c;

(b) Read a, b, c.

scanf("%f %f %f",&a,&b,&c);

(c) Check if a contains the largest value (larger than b and c). If not, print an error message.

if(a<b) printf("Error: \
a not largest n");
else if(a<c) printf("Error: \ n");
a not largest

(d) Write a program fragment to check and print whether a, b, c form the sides of a valid triangle.
Assume that a has a value larger than b and c.

if(b+c < a) printf("Error: not a valid triangle\n");


ie
o ok

(e) Print ”acute”, ”right-angled” or ”obtuse”, depending on the type of triangle formed by the sides a,
pC

b, c. Assume a is the largest side.


ar
Sh

if(b*b + c*c < a*a) printf("Obtuse triangle\n"); else


if(b*b + c*c > a*a) printf("Acute triangle\n"); else
printf("Right-angled triangle\n");

183
2. (7 marks) Complete the following C program so that it computes the sum of the following series upto
n terms. Marks: 2 + 2 + 2 + 1

x2 x4
x6
1
− + − + ···
2! 4! 6!
/* Compute the sum of the series [ 1-Xˆ2/2!+Xˆ4/4!- ...] */
#include < stdio.h >
int main()
\{
float x, sum, term;
int i, n;

printf("Enter the value of x and the number of terms to sum\n");


scanf("%f%d", &x, &n);

// Initialize values

sum=1;

term=1;

for (i = 1; i < n; i++)


\{
ie
ok

term = term * ((-1*x*x)/((2*i)*(2*i-1)));


o
pC

sum = sum + term;


ar

printf("\n Sum = %f\n");


Sh

return 1;
\}

184
3. (6 marks) Complete the following C program which given an input string prints whether it is a
palindrome or not. Marks: 1 + 2 + 1 + 1 + 1

#include <stdio.h >


int main() {
char a[100];
int i, j, length_a;

printf("Enter the String(max length 100): ");


// Read the string
Standard format to scan a
scanf("%s",a); string i.e. an array of
character.

// Compute the length of string a using a loop and store it in


// length_a. Do not use any library function.

Checking for the position at


for(i=0;i<100;i++) which null character appears.
{ Null i.e. ‘\0’ appears at the
if(a[i]==’\0’) end of the string. So the index
at which null character is
break; present is the length of the
} string. So as soon as ‘\0’ is
encountered breaking the loop
length_a=i; and assigning i to length.
ie
ok

for (i = 0; i < length_a/2; i++) {


Checking whether the
o
pC

if (a[i] == a[length_a-i-1]) characters from the front


continue; end matches their backend
ar

else { counterpart.
Sh

printf("Not a palindrome\n");
return 0; Even if one pair of
} corresponding characters are
not same then they are not
} palindrome. So displaying the
message that they are not
printf("String is a palindrome\n"); palindrome.
return 0;
} else if all the
corresponding elements
match then displaying the
message that the string is
palindrome.

185
4. (5 marks) Write C program statements in the blanks such that the following function returns the
minimum element in the array a[ ] between indices start and end (both inclusive): Marks:
1+1+3

int minv_arr (int a[], int start, int end) {


int temp;

if(start >= end) // base condition return

a[start];

else {
// Make the recursive call and return the minimum element.
// You are not allowed to use any loop the function is being
called each time with the
end index decremented by
one. This is done till end
temp=minv_arr(a,start,end-1); and start will point to the
same element of the array
i.e. the element start have
been pointing to. Then it
if(temp<a[end]) returns the element at
index start and that is
compared with the adjacent
element which is pointed to
return temp; by that particular called
functions end index. The
minimum among the two is
ie
returned to the calling
else return a[end];
ok

function and again that


} returned element is
o

} compared with the next


pC

element in the sequence and


the minimum of them is
for example if start was 2 and end was 5 returned and so on until
ar

then first the functions will be all the elements have been
recursively called until end is decremented
Sh

compared and a absolute


and equals start that is 2(base condition). minimum between start and
At this point arr[2]is returned to the end has been found. This
calling function(itself)and assigned to minimum value is then
temp and it will be compared to arr[3] and returned to whatsoever
will return the minimum between them to its other function has called
calling function(itself). Suppose arr[3] is the minv_arr function.
minimum then temp will now be arr[3] and it
will get compared with arr[4] and the
minimum between them will be returned. Say
arr[3] is the less than arr[4] so again
temp will be assigned arr[3] and now arr[3]
i.e. temp and arr[5] are compared and the
minimum between the two is sent to some
other function which initially called
minv_arr function. Try and draw recursion
tree(boxes) and keep track of which
variable has what value in which box. That
way you can solve problems based on
recursion easily as this approach gives you
a vivid pictorial view.

186
5. (10 marks) Write a program that takes as input n, followed by n integer numbers and store them in
an array A. It then calls a function which copies the distinct elements of array A to an integer array B
so that array B contains all elements of A but does not repeat any element. For example, if A stores
{ 2, 17, 19, 5, 2, 9, 9, 8, 2 }, array B will contain {17, 2, 19, 5, 9, 8} after the function call. The program
17,
comprises of a main( ), the function makeset( ) and the function check( ) which is called by
makeset( ). Marks: 3 + 5 + 2
The function printarray( ) is given which takes as input an array of integers A and its length n
and prints the array.

void printarray (int A[], int n) {


int i;
for (i = 0; i < n; i++) printf ("%d ", A[i]) ;
printf("\n") ;
}
(a) Write the function check( ) which takes as input an integer x, an array A and its size n. It
should return 1 if x is occurs in array A and 0 otherwise

int check(int x, int A[], int n) Scanning through the


{ array and checking
int i; whether the element
for(i=0;i<n;i++) is present and if
{ found then returning
if(a[i]==x) 1 and the function is
return 1; terminated. Remember
} as soon as a return
ie
return 0; statement is
ok

} encountered the
function exits.
o
pC
ar
Sh

187
(b) Write the function makeset( ) which takes as input an array of integers A, its size n1, and an
array of integers B, The function must copy the unique elements of A into the array B and return
the number of elements in B, by making use of calls to the function check( ) defined above.

int makeset(int A[], int n1, int B[]) Inside the loop we are
{ calling check function to
int i,j=0; see if the element A[i] is
for(i=0;i<n1;i++) already present in array B
{ or not. If it is not
if(check(A[i],B,j)==0) present we are adding it
B[j++]=A[i]; to array B and
} incrementing its index via
return j; j. If the element already
} exists in array B then
check function returns 1
and the element is not
added to B.
(c) Complete the function main( )
int main ( ) {
int A[100], int B[100] ;
int i, nA, nB;
scanf (‘‘%d’’, &nA) ;
for (i=0; i<nA; i++)
scanf (‘‘%d’’, &A[i]) ;
// Call makeset

nB=makeset(A,nA,B);
ie
ok

printarray (A, nA) ; printarray (B, nB) ;


return 0;
o

}
pC
ar
Sh

188
6. (11 marks) What will be printed when the following programs/ program segments execute? Write only
the output that will be printed if the program is executed within the box.
Marks: 3 + 4 + 4
(a) #include <stdio.h>
int main()
{
int i = 12, j, last;
while (i > 1) {
j = 1;
printf("%d: ", i);

while (j < i) {

if ((i % j) == 0) {
printf("%d ", j);
last = j;
}
j++;
}
i = last;
printf("\n");
}
return 0;
} We need to figure out what the snippet is
trying to do. Its first printing all the
ie
SOLUTION: factors of twelve (excluding 12) using the
ok

12: 1 2 3 4 6 inner while loop. A variable named last


6: 1 2 3 stores the highest factor of 12 i.e. 6 and
o

3: 1 outside the inner while loop i is assigned


pC

last that is i is assigned 6. Again the


inner while loop is iterated and all
ar

factors of 6 is displayed. last stores the


Sh

highest factor of 6 i.e. 3 and i is thus


assigned 3. Factors of 3 are displayed and
since the highest factor of 3 is 1 so the
outer loop breaks once the inner loop has
completed printing the factors of 3.

189
(b) #include <stdio.h>
int main ()
{
int a[] = { 6, 3, 2, 8 };
int i, j;

for (i = 0; i < 4; i++) {


printf ("%d: ", a[i]);

for (j = 0; j < 4; j++) {

if ((a[i] % a[j]) == 0) {
printf ("%d ", a[j]);
continue;
}

if ((a[j] % a[i]) == 0) {
printf ("%d ", a[j]);
break;
}
}
printf ("\n");
}
return 0;
}
What this snippet is trying to
ie
do is checking for the factors
ok

(present in the array) of a


SOLUTION:
number in the array and also
o

6: 6 3 2
pC

displaying the first multiple


3: 6 of a number if it’s present in
2: 6
ar

the array. so the factors of 6


8: 2 8 in the array are 6,3,2 and for
Sh

8 is 2 and 8. And the first


multiples of 2 and 3 are 6 so
6 is displayed against them.

190
(c) void serve (int num_tasks)
{
static int server = 1;
int taskid = 1;

printf("Starting %d tasks\n",num_tasks);

for (int i = 0; i < num_tasks; i++) {


printf (Task %d - Server %d \n", taskid, server);
server++;
if (server > 5)
server = 1;
taskid++;
}
printf("Done\n");
}

int main ()
{
serve (3);
serve (4);
return 0;
}
ie
Solution:
ok

Starting 3 tasks
Task 1 - Server 1
o

Task 2 - Server 2
pC

Task 3 - Server 3
Done
ar

Starting 4 tasks
Sh

Task 1 - Server 4
Task 2 - Server 5
Task 3 - Server 1
Task 4 - Server 2
Done

191
7. (7 marks) Consider the following functions: Marks: 3 + 4

(a)
int foo (int x, int y) {
foo(6,13):
if (x < y) x=6;y=13; thus
return x; x<y so returns x
else i.e. 6.
return foo (x - y, y);
}
For each call below, indicate what value is returned: foo(37,10) foo(27,10)

foo (6, 13) 6 foo(7,10) foo(17,10)

foo (37, 10) 7 here x=7,y=10 so x<y and


the function returns x
i.e. 7.
(b)
void baz (int n) {
if (n <= 1)
printf ("\%d ", n);
else {
baz (n/2);
printf (", \%d \\n", n);
}
}
ie
For each call below, indicate what output is printed:
ok

baz(4)
o

baz(4) baz(2)
pC

SOLUTION:
1, 2\n, 4\n
ar

prints , 2\n; prints 1;


OR baz(1)
Sh

1, 2
, 4 prints 4\n;

baz(30)
SOLUTION:
follow similar kind of
1, 3\n, 7\n, 15\n, 30\n
recursion tree as shown
OR
above.
1, 3
, 7
, 15
, 30

192
END SEM SPRING 2018

1. Assume that on a certain machine an int variable is of size 32 bits (or 4 bytes), char variable is of size
8 bits (or 1 byte), and each memory address is of size 32 bits. Assume further that the sizeof() function
call returns the size of its operand in bytes. Answer the following questions. Marks: 1 + 2 = 3
(a) Consider the following structure:
struct myStruct {
char name[20];
int account_number;
struct myStruct *next;
};
What does sizeof(struct myStruct) return on this machine?
A. 26 B. 28 C. 48 D. 46
(b) What will be the output of the C program?
#include<stdio.h> p is a pointer variable of integer. So
#include<stdlib.h>
it will have a size of 4bytes. The
int main()
malloc(20) is assigned to confuse
{
int *p; the students.
p = (int *)malloc(20);
printf("%d\n", sizeof(p));
free(p);
return 0;
}
ie
ok

A. 40 B. 20 . 4
C D. 80
o

Marks: 3 × 2 = 6
pC

2. Answer the following questions.


(a) Consider the following sequence of push and pop operations on an initially empty stack S.
ar

S = push(S,1);
Sh

S = pop(S);
S = push(S,2);
S = push(S,3); Stack follows the concept of
S = pop(S);
LIFO i.e. Last In Fast Out
S = push(S,4);
S = pop(S);
S = pop(S);
Which of the following is the correct order in which elements are popped?
A. 1 2 4 3 B. 1 3 2 4 C. 1 2 3 4 D. 1 3 4 2
(b) What is the output of the following program? Marks: 2
#include<stdio.h>
#include<stdlib.h>

int main()
{
int *ptr;
*ptr = 10;
*ptr = 20;
*ptr = 30; Actually this will result in an error as memory for
the pointer has not been alloted.

193
printf("%d\n",*ptr);
return 0;
}

A. 10 B. 20 C. 30 D. None of the above


(c) Under what condition will this program print the string “Hello”?
#include<stdio.h>
#include<stdlib.h>
If due to some reason such as memory
int main() unavailability or something ptr cannot
{ be assigned any memory then it will be
int *ptr; assigned the value NULL. If this
ptr = (int *)malloc(sizeof(int)*10); happens then the condition of if
if (ptr == NULL) statement will be true and thus Hello
printf("Hello\n"); will be displayed.
return 0;
}
A. if the memory could not be allocated to the pointer “ptr”
B. if the memory has been allocated to the pointer “ptr” successfully
C. it will never print
D. none of the above

3. Fill in the blanks to complete a C program that creates a singly linked list by repeatedly calling the
ie
function push(). It then counts the number of nodes present in the singly linked list recursively using
ok

the function getCount(). Each blank has at most one statement.


o

// Recursive C program to find length or count of nodes in a linked list


pC

#include<stdio.h>
#include<stdlib.h>
ar
Sh

/* Link list node */


struct Node
{
int data;
struct Node* next;
};

/* Given a reference (pointer to pointer) to the head of a list and


an int as parameters, the function pushes a new node on the front
of the list. */
void push(struct Node** head_ref, int new_data)
{
/* allocate node */
struct Node* new_node =
( struct Node* ) malloc( sizeof(struct Node) );

/* put in the data */


new_node->data = new_data;

/* link the old list off the new node */


new_node->next = (*head_ref);

194
/* move the head to point to the new node */
(*head_ref) = new_node ;

/* Counts the no. of occurences of a node


(search_for) in a linked list (head)*/
int getCount(struct Node* head)
The Solution is Self Explanatory.
{
Comments are Provided for proper
// Base case
head == NULL understanding.
if ( )
return 0;

// count is 1 + count of remaining list


return 1 + getCount(head —> next) ;
}

/* the main function*/


int main()
{
/* Start with the empty list */
struct Node* head = NULL;
int num;
char flag = ’Y’;
ie
// Use push() repeatedly to construct the list
while( flag == ‘Y’ (OR) flag != 'N' )
ok

{
o

printf("\n Enter the next number to be pushed into the stack: ");
pC

scanf("%d", &num);
push( &head , num);
ar

printf("\n Do you want to push more numbers into the stack?


Sh

(Answer Y to continue and N to stop pushing into the stack)");


scanf("%c", &flag);
}

/* printing the size of the list created */


printf("\n Number of nodes in the linked list is %d",
getCount(head) );
return 0;
}

4. Answer the following questions. Marks: 3 + 4 + 2 = 9


(a) A quadratic algorithm with processing time T (n) = cn spends 1 ms in processing 100 data items.
2

Time spent for processing 5000 data items = 2500 ms (OR) 2.5 sec

= or t=2.5s

(b) Consider the problem of exponentiation of integer x to the power of integer n (i.e., xn). A
straightforward way of doing this is to multiply x, n times. However a more efficient way to
solve this problem would be to see that xn = xn/2∗ xn/2. Assuming that n = 2k, we can write a
small recursive function to implement exponentiation.

195
int power(int x, int n){
if (n==0) return 1;
if(n==1) return x;
if ((n % 2) == 0) return power(x*x, n/2);
}
Let the time required to execute this program be T (n). Assume T (0) = c1 and T (1) = c2.
(i) The recursive expression is given by T (n) = T(n/2) + c3
(ii) The exact solution to the above recursive expression is c2 + (c3)log(n)

You can try with small values like 2 or 3. At first, The recursive function receives value
each time the previous value divided by 2(like for 8: 8,4,2,1). So the time depends on
. Now c3 is the time needed when whole function is executed.

(c) log(n!) = Θ( n ∗ log(n)).

log(n!)=log(n) + log(n-1) + ……. < log(n) +log(n)+ …..=nlog(n)

5. (a) What does the following function do on the elements of the array arr[]? Marks: 2
void whatdoIdo(int arr[], int size)
{
int i=0;
ie
for(i=0; i < size; i++)
ok

{ The solution is self explanatory


if( arr[i] % 2 == 0 )
o
pC

arr[i] = 0 ;
else
ar

arr[i] = 1 ;
Sh

}
}
The function whatdoIdo replaces even elements in the array with 0 and odd elements with 1 .

(b) The following function computes the median of an array of floats x[]. Assume that all the entries
in the array are distinct, and there is only a single digit after the decimal point for all the numbers.
Fill the blanks.
Each blank can have only ONE statement.Marks: 0.5 + 0.5 + 0.5 + 0.5 + 0.5 + 0.5 + 1 + 1 + 1 = 6
float median(int n, float x[]) {
float temp;
int i, j;
for(i=0; i<n-1; i++) {
for(j= i+1 ; j< n ; j++) {
if( x[j] < x[i] ) {
x[i] ; To find the median firxt we are sorting
temp =
the array using bubble sort.
x[i] = x[j] ;
x[j] = temp ;
}
} If the array contains even
number of elements then the
}
2 middle elements averaged
is the median or if the
if( n%2 == 0 ) { number of elements in the
return return
array is odd then the middle
} else { } element is the median of the
array.

196
} ( (x[n/2] + x[n/2 - 1]) / 2.0 ) ;

x[n/2] ;
(c) Now suppose you have two already sorted arrays ar1[] and ar2[] of EQUAL size n. The following
function attempts to find the median of the elements of the two arrays combined together. For
instance if ar1 = { 1.0, 12.0, 15.0, 26.0, 28.0
} and ar2 = {2.0, 13.0, 17.0, 30.0, 45.0} , you have to
{ 12.0, 15.0, 26.0, 28.0, 2.0, 13.0, 17.0, 30.0, 45.0
find the median of the elements 1.0, } . Fill in the
blanks. Each blank has only ONE statement.
Marks: (0.5 + 1) + (0.5 + 1) + 0.5 + (0.5 ∗ 3) + (0.5 ∗ 3) + 1.5 = 8

/* This function returns median of ar1[] and ar2[].


Assumptions in this function:
Both ar1[] and ar2[] are sorted arrays
Both have n elements */
float getMedian(float ar1[], float ar2[], int n)
{
int i = 0;
int j = 0;
int count;
float m1 = -1.0, m2 = -1.0; // contains medians from two arrays
for (count = 0; count <= n; count++)
{
if (i == n)
ie
{
m1 = m2 ;
ok

m2 = ar2[0] ;
o

break;
pC

}
else if (j == n)
ar

{
Sh

m1 = m2 ;
m2 = ar1[0] ;
break;
}
if (ar1[i] < ar2[j] )
{
m1 = m2 ;
m2 = ar1[i] ;
i ++ ;
}
else
{
m1 = m2 ;
m2 = ar2[j] ;
j ++ ;
}
}
return (m1 + m2)/2.0 ;
}

6. The following recursive function takes a string of given length as input and determines whether the
string is a palindrome. It returns 0 if the string is not a palindrome and 1 if it is. Fill in the blanks. Each
blank can have AT MOST one statement. Marks: 1 + 2 + 2 = 5

197
int ispalindrome ( char A[], int n )
We are checking whether the first and the last
{ element of the string A is same or not. If they
if ( n <= 1 ) return 1;/*base case*/ are found to be same we call the function once
if ( A[0] != A[n-1] ) return 0; again with arguments that have the address of
return ispalindrome( &A[1], n-1 ); the second character of the string and the
} length of the actual string decremented by 1.
So that when we are checking for the parity of
the first and last characters of the passed string
we are actually checking whether the 2nd and
the 2nd last characters of the actual string are
same or not. This goes on till we have checked
for the parity of all corresponding characters
and we return 1 if the string is palindrome else
we return 0 if in between the parity is missed,

ie
o ok
pC
ar
Sh

198
7. Complete the following program, where the main function takes three strings A, B,C as input from the
user and determines whether the string A contains the regular expression B · C, where ∗ stands for
any substring. For instance, if A = “abcdefg”, B = “bc” and C = “ef”, the function determines if an
occurrence of “bc” followed (not necessarily immediately) by an occurrence of “ef” can be detected in
“abcdefg”. In this case, the occurrence B∗C is detected at index position 1 in A, and the main function
gives this output. Either of B or C can also be null. The main function makes use of another function
locateSubstr that checks whether a string A contains another string B as a substring, and if so, returns
the match index of B in A. Thus, when B and C are non-empty, the main function first finds if B is a
substring of A, and if that is the case, whether C is a substring for the remaining portion of A, where the
match for B ends. Fill in the blanks.
Each blank can have AT MOST one statement. Marks: 1 + 1 + 2 + 1 + 1 + 1 + 1 + 1 + 1 = 10
#include <stdio.h>
#include <stdlib.h>
#define MAXLEN 1024
int locateSubstr ( char A[] , char B[] )
{
int i, j, match;
if (strlen(B) == 0) return 0;
for (i=0; i<= strlen(A) - strlen(B) ; ++i) if (A[i] == B[0]) {
match = 1;
Strlen(B) A[i+j] != B[j]
for (j=0; j< ; ++j) if ( )
{ match = 0; break; }
if (match) return i ;
ie
}
ok

return -1;
}
o

int main ()
pC

{
ar

char A[MAXLEN], B[MAXLEN], C[MAXLEN];


/* Assume the code to input strings from the users is here.
Sh

You need not write anything here */


/* i should store the matching index of B*C in A*/
int i,j,k;
if (strlen(B) == 0) i = locateSubstr(A,C) ;
else if (strlen(C) == 0) i = locateSubstr(A,B) ;
else{
j = locateSubstr(A,B);
if (j < 0) i=j;
else k = locateSubstr( &A[j+strlen(B)] , C );
if (k>=0) i= j ;
else i = —1 ;
}
if (i >= 0)
printf("The pattern B*C is found in A at idx %d\n", i);
else
printf("The pattern B*C is not found in A\n");
exit(0);
}

8. The following recursive function makes the base conversion. It reads two integers n and b from the
terminal (with n ≥ 0 and b > 1, both in base 10) and expresses n in base b. For example, the decimal

199
expansion of 345 in base 10 is 345 = 3 × 102 + 4 × 10 + 5. Note that in this case 5 = 345%10 and
34 = 345/10; The output as printed by the program should be: (345) 10 = (3, 4, 5) 10. However,
please note that for n = 10 and b = 2, the program should print (10) 10 = (1, 0, 1, 0) 2 and NOT
(10) 2 = (1, 0, 1, 0) 2 .
Fill in the blanks. Each blank can have AT MOST one statement. Marks: 2 × 5 = 10
#include <stdio.h> for converting a decimal
number n to a number in
void baseconv ( int n , int b ) base b we use recursion to
{ continuously divide the
/* n is too small. Simply print it and return. */ number by its base till its
if ( n<b ) { printf("%d",n); return; } smaller than the base b and
we display this first followed
/* Recursively print the more significant digits */ by the subsequent display of
baseconv(n/b,b) ; the previous remainders
(n%b) in the previous
/* Finally print the least significant digit */ recursively called functions
printf(",%d", n%b ); in down to top approach.
} That is in simple words we
keep dividing the original
number n by b till the
int main ()
quotient left is less than b,
{ we display this quotient first
int n, b; and then we backtrack the
remainders we got by
ie
printf("n = "); scanf("%d", &n); continuously dividing n by b
ok

printf("b = "); scanf("%d", &b); and print them in the reverse


order. This gives us a
o

if ((n < 0)||(b < 2)) { number in base b equivalent


pC

fprintf(stderr, "Error: Invalid input...\n"); to n base 10.


ar

exit(1);
} 2 10 0
Sh

2 5 1
(%d)_10 = ( ",n); 2 2 0
printf("
2 1 1
baseconv(n,b);
We stop here because the
printf(" )_%d ",b);
quotient achieved i.e 1 is less
than 2. So now we write the
exit(0); remainders in reverse order.
}

9. Answer the following questions. Marks: (4 × 1) + 2 + (3 × 2) + (3 × 2) = 18


(a) CIRCLE the correct choice.
[i] What value will be assigned to the variable a after the following two statements are executed?
int a = 7, b = 5, c = -3;
a = a - a % b * c;
A. 0 B. 9 C. 13 D. 14

[ii] What value is assigned to the variable var? Here T is 10+10 and not 20. So wherever T occurs in the
#define T 10+10 snippet replace T by 10+10 and not 20.
var=10+10*10+10=120. This is the way #define works.
var = T * T;

A. 400 B. 210 C. 200 D. 120

200
[iii] Which of the following is NOT a legal name of a C variable?
A. 12_pds B. _12pds C. pds_12 D. pds12_

[iv] What is the 8-bit 2’s-complement representation of −49?


A. 11001110 B. 11001111 C. 10110001 D. 11010001

(b) Find the 32-bit (single-precision) floating point representation of +41.6 in the IEEE 754 format.
Put only ONE binary digit in each gap/space provided below.
0 1 0 0 0 0 1 0 0
Sign Bit: Exponent:
0 1 0 0 1 1 0 0 1 1 0 0 1 1 0 0 1 1
Mantissa:
0 0 1 1 0

(c) Consider the following for loop:


for (k=1; k<100; ++k) k *= k+1;
[i] How many times is the statement “k∗ = k + 1;” executed? 3
[ii] How many times is the loop condition “k < 100” checked in the loop? 4
[iii] What is the value stored in the variable “k” immediately after the for loop terminates?
183
(d) What will be the output of the following programs?
[i]
ie
#include<stdio.h> iteration 1:
ok

int main() if statement is not satisfied because y is even.


{ x=4*2=8; y=6/2=3;
o

int x = 4, y = 6, z = 0; iteration 2:
pC

while (y != 0) { if statement is satisfied as y is odd, so


z=z+x=0+8=8
ar

if (y % 2) z += x;
x=8*2=16;y=3/2=1(as y is int)
x *= 2;
Sh

iteration 3:
y /= 2; if statement is satisfied as y is odd, so
} z=z+x=8+16=24
printf("%d\n", z); x=16*2=32;y=1/2=0(as y is int)
return 0; now since y is 0 the loop terminates and the final value
} of z=24
Answer: 24
[ii]
#include <stdio.h>
(a>=b) is true so results in 1. a<=b is true again
int main ()
as a==b so it again results in 1 but p>q is false
{ so results in 0. so when ((a<=b)&&(p>q) is
int a = 5, b = 5, c; evaluated it is (1&&0)
char p = ’p’, q = ’q’; which results in 0. So when
c = !( (a>=b) || ((a<=b)&&(p>q)) ); ((a>=b)||((a<=b)&&((p>q))) is evaluated it is
printf("%d\n", c); 1||0 which results in 1 and when negated i.e. !1
is done it gives 0.Thus c=0.
return 0;
}
Answer: 0
[iii]
#include <stdio.h>
int main ()

201
{
int p, q;
for (p=q=0; p<10; ++p) {
q = p + q;
If you see the loop closely what is happening is
++p;
p is being incremented by 2 after each iteration
}
and that is being summed over and stored in q
printf("%d\n", q);
till p<10. So q=2+4+6+8=20.
return 0;
}
Answer: 20

10. The following program computes the sum of the square of digits in the decimal representation of a non-
negative integer. For example, the sum of the square of digits for 320127 is 32 + 22 + 02 + 12 + 22 + 72 =
67. Fill in the blanks with appropriate C constructs.
Each blank can have AT MOST one statement. Marks: 1 + 1 + 1 + 1 + 2 + 1 = 7
#include <stdio.h>
int main ()
{
unsigned int n, d, sum;

/* Read the unsigned integer n*/


scanf( “%u” , &n );
The solution is Self Explanatory
ie
/* Initialize sum */
ok

sum = 0 ;
o
pC

/* Loop as long as n is not reduced to zero*/


while ( n > 0 (OR) n != 0 ) {
ar

/* Store in d the least significant digit of n*/


Sh

d = n%10 ;

/* Add the square of this least significant digit to sum*/


sum = sum + d*d ;

/* Remove this digit from n */


n = n/10 ;
}

/* Print the sum of the square of digits of the input integer*/


printf("The sum of the square of digits is %d \n", sum );

return 0;

202
Indian Institute of Technology Kharagpur
Programming and Data Structures (CS10001)
Autumn 2017-18: End-Semester Examination
Time: 3 Hours Full Marks: 100
INSTRUCTIONS
1. Answer ALL questions.
2. Please write the answers either within the boxes provided or on the blank lines to be filled up. Any
answer written elsewhere will not be evaluated.
3. You may use the blank pages at the end for your rough work.
4. In the C programs, assume that all appropriate header files are included.

QUESTION 1. Answer the following questions as directed.

a) Given that the ASCII codes for lowercase letters in the English alphabet ‘a’ to ‘z’ are 97 to 122 (in
decimal), what will be displayed when the following C program is executed? [1 Mark]

int main() { 204


int a;
The ASCII code of ‘d’ is 100. Thus on
a = ‘d’ * 2 + 4;
performing the operation ‘d’*2+4 its
printf(“%d”, a);
evaluates 100*2+4=204.
return 0;
}
ie
b) What will be displayed when the following C program is executed? [1 Mark]
ok

int main() { 0 Simple application of ternary operator.


o

int a = 5; Since a==5 is true so the first value i.e


pC

printf(“%d”, a == 5 ? 0 : 5); 0 is printed. If the condition was to be


ar

return 0; false then 5 would have been printed.


}
Sh

c) What will be displayed when the following C program is executed? [1 Mark]


int main() { 1
int a = 0; a is first assigned 0. So when a==0 it results in ‘true’
printf(“%d”, a == 0); but since the data type of the printed value is
return 0; integer so ‘true’ results in 1 which is displayed.
}

d) What will be displayed when the following C program is executed? [1 Mark]


int main() { 6 iteration 1: sum=0; a=1; (a is post incremented)
int a, sum = 0; a is again incremented by 1 in the loop definition so a
becomes 2.
for (a = 0; a < 5; a++)
iteration 2: sum=2; a=3; (a is post incremented)
sum = sum + a++; a is again incremented by 1 in the loop definition so a
printf(“%d”, sum); becomes 4.
return 0; iteration 3: sum=2+4=6; a=5; (a is post incremented)
} a is again incremented by 1 in the loop definition so a
becomes 6.
now since 6<5 is not satisfied in the loop condition so the
loop terminates and the sum 6 is displayed.

203
e) What will be displayed when the following C program is executed? [1 Mark]

int main() {
int a[4] = {10, 20, 30, 40}, count, sum = 0; 200
for (count = 0; count < 4; count++)
sum = sum + a[count] + a[3-count];
If we observe closely what the a[count]+a[3-
printf("%d", sum);
count] is doing when count is looped from 0 to
return 0;
4 is is equal to 2*(sum of the elements of the
}
array)=2*(10+20+30+40)=200

f) What will be displayed when the following C program is executed? [1 Mark]


int a = 1; 3 This questions test your knowledge of scope and
void myProc() { 1 understanding of global and local variables. First, a=1
if ( a != 2 ) {
1 is assigned(this a is global) then myProc() function is
int a = 3; printf ("%d\n", a); called and the if condition is checked. a here is 1 so
} a!=2 is true so control enters the if block. here a
else { int a = 4; } local variable a is assigned 3 which is then displayed.
printf ( "%d\n", a); Then the control exits the if block and then after
} else block the value of a is printed which is 1( here
int main() { the value of the global variable a is printed as outside
the if block there is no existence of local variable
myProc(); printf ("%d\n", a);
a=3.) then again in the main function the global value
return 0;
of a is displayed i.e. 1.
ie
}
o ok

g) What will be displayed when the following C program is executed? [1 Mark]


pC

int main() { 20 20
ar

int *a, **b, c; Very simple use of single and double pointers. and b
are single and double pointer respectively. a is pointer
Sh

b = &a; a = &c; c = 20;


to c and b is pointer to a. So *a and **b will display the
printf ("%d %d", **b, *a);
same value as that of c which is 20. SO 20 will be
return 0;
displayed twice for **b and *a respectively.
}

h) What will be displayed when the following C program is executed? [1 Mark]


3 4 The array name i.e. b is itself a pointer to the array
int main() {
and here after b is assigned to a, a becomes a
int *a, b[5] = {1, 2, 3, 4, 5}; pointer to the array as well. so *(b+2) and *(a+3)
a = b; will display the 3rd and the 4th element of the array
printf ("%d %d", *(b+2), *(a+3)); i.e. 3 and 4 respectively.
return 0;
i) }
Write a single C statement that declares a to be an integer type variable, b to be an integer pointer, and b
is initialized to contain the address of a. [1 Mark]

int a, *b = &a;

204
j) Consider that a text file named file1.txt contains a single line with just one word Avishek in it. What will
be displayed when the following C program is executed? [1 Mark]

int main() {
FILE *fptr; char line[6]; Av
if ((fptr = fopen("file1.txt", "r")) != NULL) is
while (fgets(line, 3, fptr) != NULL) he
printf ("%s\n",line); k
return 0;
}

QUESTION 2. Answer the following questions as directed.

a) Write a single C statement that declares str1 to be an array of three pointers pointing to constant strings
“Sarthak”, “Anita” and “Dinesh”, respectively. [2 Marks]

char *str1[ ] = {“Sarthak”, “Anita”, “Dinesh”}; OR


char *str1[3] = {“Sarthak”, “Anita”, “Dinesh”};
ie
ok

b) What will be displayed when the following C program is executed? [2 Marks]


o

int main() { 0 in the loop in the first statement x is always


pC

int i; 0 assigned 0 so the if statement is never true so the


ar

for (i = 0; i < 4; i++) { 0 printf statement in else block is executed where x


Sh

int x = 0; 0 is displayed which is always 0. Then x is


if (x==2) break; incremented by 1 but again when the loop is
else printf ("%d\n",x); reiterated x is assigned 0 and same process
x++; follows until the loop is terminated i.e. as a result
} four 0s are printed.
return 0;
}

c) What will be displayed when the following C program is executed? [2 Marks]

struct abc {int a; int b;};


struct abc def[5]; 6 (*def).a will result in 5 so ++(*def).a will be
int main() { ++5 which will result in 6.
def[0].a = 5; def[0].b = 10;
def[1].a = 15; def[1].b = 20;
printf ("%d\n", ++(*def).a);
return 0;
}

205
d) What will be displayed when the following C program is executed? [2 Marks]

int main() { b abc is a pointer to the string “bef” so *abc will


char *abc = "bef"; be ‘b’ and thus *abc+1 will be ‘c’ so the if
if (*abc + 1 == 'c') printf ("%c", *abc++); statement will be true and will print *abc++ i.e.
else printf ("%s", abc); b. Note here post increment is used so first
return 0; *abc i.e. ‘b’ will be printed then it will be
} incremented.

e) What is the hexadecimal representation of the decimal number 140.5? [2 Marks]

8C.8

f) What is the decimal representation of the result if 11101100 is subtracted from 01010000? Assume that
both the binary numbers are represented in 8-bit 2’s complement form. [2 Marks]

100
ie
ok

g) What will be the displayed when the following C program is executed? [2 Marks]
o

int main() { 20 This is a case of fall through. The control


pC

char b = 'a'; 30 in switch case is first directed to case ‘a’


switch (b) {
ar

and since there is no break statement for


default: printf ("10\n"); case ‘a’ so the control moves on to the
Sh

case 'a': next case i.e. case ‘A’ and prints 20 and
case 'A': printf ("20\n"); since again there is no break control
case 'b': moves to case ‘b’ then to case ‘B’ and
case 'B': printf ("30\n"); prints 30 then encounter counter break
break; statement and the control breaks from
} the switch case. Note That default will
return 0; only be executed when no of the cases
} match which is not the scene here.

h) What will be displayed when the following C program is executed if each address is of size 8 bytes in the
computer? [2 Marks]

int main() { 3 3
char a[5] = {'d', 'e', 'f', 'g', 'h'};
printf ("%u %u\n", (a + 3) - a, *(a + 3) - *a);
return 0;
}

206
i) What will be displayed when the following C program is executed from the command line as:
./a.out A 5 [2 Marks]

int main(int argc, char *argv[]) {


char c; int a; F
sscanf(argv[1], "%c", &c);
sscanf(argv[2], "%d", &a);
printf ("%c", c + a);
return 0;
}

j) What will be displayed when the following C program is executed? [2 Marks]


struct node {
int data; struct node *next; 10 30
};
int main() {
struct node a, b, c;
a.data = 10; b.data = 20; c.data = 30;
a.next = &b; b.next = &c; c.next = NULL;
printf ("%d %d", a.data, a.nextnextdata);
return 0;
ie
}
ok

Answer the following questions as directed.


o
pC

QUESTION 3.
a) The following C function is used to compute the product of two matrices, with each matrix represented
ar

as a 2-d array. The function takes as parameters a 2-d array A with n_r_A rows and n_c_A columns and a
Sh

2-d array B with n_r_B rows and n_c_B columns. The function returns a pointer to the first element of
the product matrix C. Fill in the missing lines. [1+1+1+1+1+1 = 6 Marks]

int **mult(int **a, int n_r_a, int n_c_a, int **b, int n_r_b, int n_c_b) {

int **c ; //Declare suitable variable for returning result


int sum, i, j, k;

. c = (int **) malloc (sizeof(int *)*n_r_a);


for (i = 0; i < n_r_a; i++) {

c[i] = (int *) malloc(sizeof(int)*n_c_b);


for (j = 0; j < n_c_b; j++) {
c[i][j] = 0;
for (k = 0; k < n_c_a ; k++)

c[i][j] = c[i][j] + a[i][k] * b[k][j];


}
}
return c;
}

207
b) The following C program is written to compute the sum of all the elements of a 2D array. However,
when compiled, a syntax error is detected. Encircle the line where there is a syntax error and write it
correctly in the box provided. Even after correcting the syntax error, when the program is executed, the
sum is not printed correctly since there is also a logical error in one of the statements in the get_sum
function. Encircle the statement which has a logical error and write it correctly in the box provided.
[2+2=4 Marks]
int get_sum(int x[2][]) {
int sum = 0, i, j;
for (i = 0; i < 2; i++) int get_sum(int x[ ][3]) {
for (j = 0; j < 3; j++)
sum =+ x[i][j]; sum += x[i][j];
return sum;
}
int main() {
int A[2][3] = {{1, 2, 3}, {4, 5, 6}};
printf ("%d\n", get_sum(A));
return 0;
}

QUESTION 4.
ie
a) The following program uses quicksort to sort an array of integers in decreasing order. What will be
ok

displayed when the program is executed? [5 Marks]


o

int partition (int *A, int lt, int rt) {


pC

int pivot, i, j, temp;


pivot = A[lt]; i = lt; j = rt; 45
ar

while (i < j) { 17
Sh

while (A[j] < pivot) j--; while (A[i] >= pivot && i <= rt) i++; 38
if (i < j) { 101
temp = A[j]; A[j] = A[i]; A[i] = temp; j--; 115
}
}
temp = A[lt]; A[lt] = A[j]; A[j] = temp;
return j;
}
void quicksort (int *A, int lt, int rt) {
int ind;
if (lt >= rt) return;
ind = partition(A,lt,rt); printf ("%d\n", A[ind]);
quicksort(A,ind+1,rt); quicksort(A,lt,ind-1);
}
int main() {
int X[9] = {45, 65, 24, 38, 17, 101, 4, 203, 115};
quicksort (X, 0, 8);
return 0;
}

208
b) The following recursive function is used to implement insertion sort. Fill in the missing lines.
[1+1+1+1+1=5 Marks]
void recur_insertion_sort (int A[], int n) {
int j, key;

if (n <= 1) return; //Base case


//Recursively sort first n-1 elements

recur_insertion_sort (A, n-1);


// Insert the last element at its correct position
key = A[n-1];
// Move each array element greater than key one position to its right

j = n-2;
while (j >= 0 && A[j] > key) {

A[j+1] = A[j];
j--;
}
A[j+1] = key;
}
ie
ok

QUESTION 5.
o

a) When the following C program is executed, it produces seven lines of output. The first, third, fourth and
pC

fifth lines of the output are shown in the box below (the addresses are printed in hex). Write what will be
printed as the second, sixth and seventh lines of the output. Assume that each address is of size 4 bytes in
ar

the computer. [2+2+2=6 Marks]


Sh

int **allocate (int h, int w) {


int **p, i, j;
p = (int **) malloc(h*sizeof (int *));
printf ("p = %p\n", p);
printf ("p+1 = %p\n", p+1);
for (i = 0; i < h; i++) p[i] = (int *) malloc(w * sizeof(int));
for (i = 0; i < h; i++) printf("p[%d] = %p\n", i, p[i]);
return(p);
p = 00980FF8
}

p+1 = 00980FFC
int main() {
p[0] = 00981010
int **q;
int M = 3, N = 3; p[1] = 00981028
q = allocate (M, N); p[2] = 00981040
printf ("&q[1][2] = %p\n", &q[1][2]);
printf ("&q[0] = %p\n", &q[0]);
&q[1][2] = 00981030
return 0;
&q[0] = 00980FF8
}

209
b) What will be displayed when the following C program is executed? [2+2=4 Marks]

struct abc {int a; int b;} *def;


int main() {
char *ch = (char *) malloc (4*sizeof(int)); 10
def = (struct abc *) malloc(5*sizeof(struct abc)); 0
defa = 5; defb = 10;
def[1].a = 10; def[1].b = 11;
++defa;
++def;
printf ("%d\n", defa);
printf ("%d\n", sizeof(def) - sizeof(ch));
return 0;
}

QUESTION 6.
a) What will be displayed when the following C program is executed? [1+1+2+2=6 Marks]

int rec1 (int);


ie
int rec2 (int);
ok

int main() {
printf("%d : %d : %d : %d\n", rec1(0), rec1(1), rec2(1022), rec2(2001));
o
pC

return 0;
}
ar

int rec1 (int n) {


1 : 0 : 0 : 1
Sh

if (n == 0) return 1;
else return ( rec2(n-1) );
}
int rec2 (int m) {
return ( !rec1(m) );
}

b) The following C function recursively computes the product of two input parameters a and b. Fill in the
missing lines. [2+2=4 Marks]

int recur (int a, int b) {


if (b == 0) return 0;

if (b % 2 == 0) return recur (a+a, b/2);

return recur (a+a, b/2) + a;


}

210
QUESTION 7.
a) We want to convert the cartesian form (a, b) of a complex number a + ib to the corresponding polar form
r(cos + i sin) representation denoted as (r, ). Here r and  are related to a and b as: r = (a2 + b2)1/2
and  = tan-1(b/a) in radian. For example, if the complex number 3 + i4 is represented in cartesian form
as (3, 4), its polar form representation is (5, 0.927295) since (32+42)1/2 = 5 and tan-1(4/3) = 0.927295
radian.

The following C program uses a function cart2pol to convert from cartesian to polar form of a complex
number. Fill in the missing lines so that the main function can print the correct polar form corresponding
to a complex number taken as input in Cartesian form. [1+1+1+1+1=5 Marks]

typedef struct {
float rel; float img;
} cartesian;

typedef struct {
float mag; float theta;
} polar;
ie
polar *cart2pol (cartesian a) {
ok

//Dynamically allocate memory


o
pC

polar *b = (polar *) malloc(sizeof(polar));


//Assign value for magnitude and theta in structure
ar
Sh

bmag = sqrt(a.rel*a.rel + a.img*a.img);

btheta = atan(a.img/a.rel);

return b;
}

int main() {
cartesian x; polar * y; scanf (“%f%f”, &x.rel, &x.img);

y = cart2pol(x); //Call cart2pol function


printf("%f %f", ymag, ytheta);
return 0;
}

211
b) The following function takes an array of students as input along with the length of the array and sorts the
array in lexicographically ascending order of the lastname. Fill in the missing lines. You may use
appropriate string library function(s). [1+1+1+1+1= 5 Marks]

typedef struct {
char firstname[30]; char lastname[30];
} fullname;

typedef struct {
int roll; fullname name;
} student;

void selectionsort (student A[], int size) {


int i, j, pos;
student min, temp;
for (i = 0; i < size - 1; i++) {
pos = i;

min = A[i]; //Save current minimum


for (j = i + 1; j < size; ++j) {
//Does lastname come before lastname in current minimum
ie
if (strcmp(A[j].name.lastname,min.name.lastname) < 0 )
ok

{
o

min = A[j]; //Update current minimum


pC

pos = j;
ar

}
Sh

}
//Bring the smallest data to the current location of i

temp = A[i];

A[i] = A[pos];
A[pos] = temp;
}
}

212
QUESTION 8.
a) The following C recursive function reverse_list is used to reverse the contents of a linked list. It takes as
parameter a pointer to a pointer to the first element of the list. Thus, starting from the head, if the list
contains the following sequence of data: {56, 78, 20, 88, 80}, after the function is completed, the list will
contain the following sequence of data: {80, 88, 20, 78, 56}. Fill in the missing lines.
[1+1+1+2=5 Marks]
struct node {
int data; struct node *next;
};
void reverse_list (struct node **head) {
struct node *previous = NULL; This question had a small mistake. The problem
statement says “following C recursive function”
whereas the code given is non-recursive. This may
struct node *current = *head;
have created a confusion for some students, though
struct node *next; chances are less as the code was quite clearly non-
while (current != NULL) { recursive and we have not received a single question
or comments on the copy. Nevertheless, it is decided
next = currentnext; to give full marks to ALL students in this question.
currentnext = previous;
previous = current;

current = next;
}
*head = previous;
}
ie
ok

b) The following recursive function merge_sorted takes as input the head pointers to two linked lists. In
o

each linked list, the elements are in ascending order. The function returns the head pointer to a new
pC

linked list which contains all the elements of the two input lists in ascending order. Fill in the missing
lines. Assume that the same definition of struct node as given in Question 8(a) is used here.
ar

[1+1+1+1+1=5 Marks]
Sh

struct node *merge_sorted (struct node *a, struct node *b) {


struct node *result = NULL;
// Base cases

if (a == NULL) return (b);

else if (b==NULL) return (a);


//Pick either from a or from b and recur

if (adata <= bdata ) {


result = a;
resultnext = merge_sorted (anext, b);
}
else {
result = b;
resultnext = merge_sorted (a, bnext);
}
return(result);
}

213
QUESTION 9.
a) The following C program uses a stack of size 100 characters with the given definitions of push() and
pop() to determine if an input string is a palindrome or not. A string is said to be a palindrome if it reads
the same forward and backward. For example, abcba is a palindrome while abcbc is not. Assume that
push() will never be called on a full stack and pop() will never be called on an empty stack. Fill in the
missing lines. Use appropriate C string library function(s) if needed. [1+2+2+1= 6 Marks]

char stack[100]; int top = -1;


void push (char c) { stack[++top] = c; }
char pop() { return (stack[top--]); }

int main() {
char input_string[30]; int i, count = 0, length;
scanf("%s", input_string);

length = strlen(input_string);
//Push characters in stack

for (i = 0; i < length; i++) push(input_string[i]);


for (i = 0; i < length; i++) {
//Compare with original string by popping from stack

if (input_string[i] == pop())
ie
count++;
ok

}
o

if (count == length) //Check condition for palindrome


pC

printf("Palindrome”);
else printf("Not palindrome");
ar

return 0;
Sh

b) What will be the displayed when the following C program is executed? [4 Marks]

typedef struct { int list[10]; int front, rear; } queue;


void enqueue(queue *q, int e) { 8 9 10
qrear = (qrear + 1);
qlist[qrear] = e;
}
void dequeue (queue *q) { qfront = (qfront + 1); }
int main() {
int i = 0; int e = 4; queue *newqueue;
newqueue = (queue *)malloc(sizeof(queue));
newqueuefront = 0; newqueuerear = -1;
for (i = 0; i < 7; i++) { ++e; enqueue(newqueue,e); }
for (i = 0; i < 3; i++) dequeue(newqueue);
for (i = newqueuefront; i < newqueuerear; i++) printf ("%d ", newqueuelist[i]);
return 0;
}

214
END SEMESTER AUTUMN 2016-2017

1. Answer in one word. Marks: 10 × 1 = 10

(a) What would be returned to the pointer if dynamic memory allocation fails?

NULL When no memory is allocated, it does not find the


address and thus no address is returned

(b) What is the minimum array size required for storing a string ”IIT”?

4 3 bytes and 1 for NULL character

(c) What is the name of the operation for deleting an element from a stack?

POP

(d) Address of which element of an array is also denoted by its name?

First element of index 0

(e) Write the octal code of the binary string 11100001.


ie
341 Make groups of three from the right hand side. Here is
011 100 001 when converted to decimal gives 341
ok

011-3 100-4 001-1


o
pC

(f) Which header file is required to be included for dynamic memory allocation?
ar

stdlib.h malloc.h
Sh

(g) What is the maximum unsigned integer represented by a 16 bit binary number?

𝟐𝟏𝟔 -1 All the bits are 1. So when converted to decimal it gives the sum
of the GP series 215 +214 +213 +212 +……+20

(h) What should be passed as parameter to the function that is called-by-reference?

Address of the variable or the data passed

(i) Which statement should be preferably used for conditional branching at different values of a
variable?

switch statement switch statement passes the value and matches the condition
according to the passed value

(j) Name the operator for determining the size of a data type.

sizeof(<data type>)

215
2. State whether the following statements are True/False. Justify your answer in at most two simple
sentences. Marks: 5 × 2 = 10
(a) A structure should preferably be passed as a parameter to a function using call-by-reference.

TRUE If a structure is passed by call-by-value, then the changes made in the


structure will be local to the function and changes will not be reflected in the
function form where the structure was passed.

(b) A string cannot be assigned to another string.

TRUE Normal equal operators do not allow such assignment operation. But string can
be copied using string.h library using strcpy(target,source) function.

(c) Iterative function call is more efficient than a recursive function call for the same computation.

TRUE Iterative call uses much less memory than recursive function call. When
recursion occurs, when function call occurs, the previous function remain in the
memory and thus the memory consumption is much higher. This do not occur
in recursion.

(d) Dynamically allocated memory should be freed after its use.

TRUE If memory is kept allocated, then once the reference of that memory is lost the
data remains allocated and cannot be used further. But in ANSI C the memory
gets free after the code stops execution
ie
(e) Linked list is a more dynamic data structure than a list implemented by an array through dynamic
ok

memory allocation.
o
pC

TRUE For dynamic link list, we can always increase the memory or decrease the
memory. There is no extra memory allocation and memory.
ar
Sh

Page 2

216
3. Write statements (corresponding to a C program segment) for the following: Marks: 5 × 2 = 10
(a) Declare an integer variable i and a pointer to the corresponding data type p. Assign address of i to
p.

int i;
int *p;
p=&i;

(b) Define a structure consisting of an integer variable x and a real variable y. Declare a variable s
corresponding to the structure.

typedef struct data


{
int x; float y;
}s;

(c) Declare p as a pointer to a pointer of data type int, and allocate an array of 20 pointers to p by
dynamic memory allocation.

int **p; As p is a pointer to a pointer, it is a double pointer.


p=malloc(20*sizeof(int*)) As it is an array of pointers.so the sizeof is of an int
pointer.
ie
(d) Define a node of a linked list which stores a name of a student as a string not exceeding 50
characters and the student’s CGPA as a real number. Declare a variable for a node using the
ok

definition.
o

typedef struct
pC

{
char name[50];
ar

float cgpa;
Sh

node* next;
}list;

(e) Declare a function prototype named swap for swapping two integer variables passed as parameters
to it. No need to define the function.

void swap(int* x, int* y)

217
4. What will be printed when the following programs/ program segments execute? Marks: 3 + 3 + 4 = 10
(a) char frname[12]="Pineapple", drname[12];
int i=0,j=0;

while(frname[i]!=’\0’) {
if(i%2==0)
drname[j]=frname[i];
i++; j++;
}
drname[j]=’\0’;
printf("i=%d j=%d drname=%s \n",i,j,drname);

i=9 j=9 drname=Pn The while loop exits out when the null character is reached. Also j
runs along with i. So the odd characters are not assigned.
Depending on the compiler, junk characters are assigned. So if
NULL character is assigned, then the drname is terminated.

(b) int compute( int n)


{
if(n<1)
return(1);
return(n*compute(n-2));
}
ie
void main()
ok

{
o
pC

printf("val1=%d val2=%d val3=%d\n",compute(5),compute(4),compute(-5));


}
ar
Sh

val1=15 val2=8 val3=1

(c) struct _st {


int x,y;
struct _st *lnk1,*lnk2;
} a,b,c, *p;

a.x=b.y=10; a.y=b.x=15;
c.x=a.x+b.x; c.y=a.x+b.y;
a.lnk1=&b; b.lnk1=&c; a.lnk2=&c; b.lnk2=&a;
c.lnk1=c.lnk2=NULL; p=&b;
printf("val1=%d val2=%d \n",p->lnk1->x,p->lnk2->y);
printf("val3=%d val4=%d \n",p->lnk2->lnk1->x, p->lnk2->lnk1->y);

val1=25 val2=15
val3=15 val4=10

218
5. Write C program statements for the following operations. Marks: 4 + 6 = 10

(a) Define a node of a circular linked list which contains a complex number.
typedef struct node
{
float real,imaginary;
node *next;//pointer to the next node
}node,*list;

(b) Assume that the head of the above circular list is pointed by a pointer named head. Write a
function which takes the head of the list as argument and returns the sum of complex numbers in
the list.

node sum(list head)


{
node sum;
sum.real=sum.imaginary=0.0f;
while(head->next != NULL) //Traversing to the next node until NULL found
{
sum.real+=head->real;
sum.imaginary+=head->imaginary;
ie
}
return sum;
ok

}
o
pC
ar
Sh

219
6. Write C program segments/statements to serve the following purposes. Marks: 3 + 3 + 4 = 10

(a) Define a structure for implementing a stack of integers using an array.

typedef struct {
int data[SIZE]; //the array of the data
int tos; //the pointer of the top
}stack;

(b) Write a function for creating a stack.

void create()
{
stack* s; //creating the stack
s->tos = -1; //initializing its head to -1
}

ie
ok

(c) Write a function for pushing an integer into the stack.


o
pC

int push(stack *s,int n)


{
ar

if((s->tos == SIZE-1) // header reached its maximum


Sh

{
printf("Sorry stack is FULL\n");
return ERROR;
}
++(s->tos); //increasing the head pointer
s->data[s->tos]=n; //entering the data
return 1;
}

220
7. Write a C function which takes an array of N integers and returns the range of its value (i.e., maximum
value - minimum value). Marks: 10

int range(int a[],int n)


{
int max,min;
max=min=a[0]; //Initializing the both max and min to the first element of the array
for (int i = 1; i < n; ++i)
{
if (a[i]>max)
max=a[i]; //Initializing the maximum to ar[i] when greater element found
if (a[i]<min)
min=a[i]; //Initializing the minimum to ar[i] when lesser element found
}
return (max-min);
}

ie
o ok
pC
ar
Sh

221
8. Write C program statements for the following operations. Marks: 2 × 5 = 10

(a) Dynamically allocate a 2-D array of characters to a pointer for storing 100 strings each of maximum
character length 80.

char **p; //Declaring the two dimensional array


p=(char**)malloc(100*sizeof(char*)); //Array of 100 strings
for(int i=0;i<100;i++)
p[i]=(char*)malloc(80*sizeof(char));//Array of 80 characters

(b) Read four command line arguments and print them as strings in the same order.

#include <stdio.h>
int main(int argc, char const *argv[])
{
ie
//The first one is the executive file ./filename
ok

printf("%s\n",argv[1] );
o

printf("%s\n",argv[2] );
pC

printf("%s\n",argv[3] );
printf("%s\n",argv[4] );
ar

return 0;
Sh

222
9. Write a C program with loop to evaluate the following series summation with an accuracy of 5th decimal
place. Marks: 10

x2 x3 x4
x
x− + − + . . .
2! 3! 4!

#include <stdio.h>
#include <math.h>

int main()
{
float sum=0.0f;
float diff=1.0f,x; //The difference between the consecutive terms
int cnt=0; // the counted no of terms
float fact=1; //Calculating the factorial of the terms
float termPrev=0; // As a copy for the previous term
printf("Enter value of x\n");
scanf("%f",&x);
while(diff>.00001)
{
cnt++;
fact=fact*cnt; //finding the factorial
ie
float term=pow(x,cnt)*pow(-1,cnt-1)/fact; //producing the terms
ok

diff = fabs(term-termPrev); //difference between the consecutive terms


termPrev=term;
o

sum+=term;
pC

}
printf("required sum = %f\n",sum);
ar

}
Sh

223
10. Fill in the gaps of the following statements for the operations mentioned in parantheses beside the
questions. Marks: 10 × 1 = 10

(a) int x= (float) 5.0; (type cast).

(b) struct cplx{ float a,b}; struct cplx m= cplx(4.5,4.9) .


(Initialize to values of member variables a and b to 4.5 and 4.9, respectively.)

(c) int x,*p; p=&x; &p = 10;


(Assign 10 to the variable x accessed through the pointer p.)

(d) char name[20]="Kharagpur",dst[20]; strcpy( dst , name )


(Copy the content of the string name to dst using string library function.)

(e) int x; float y; scanf(“ %d %f ” ,x , y)


(Read variables x and y from keyboard.)

(f) char line[80]; scanf( “%[^\n]”, line )


(Read a line in the character array line as a string from the keyboard. The line may or may not
ie
contain space(s). You need to store space(s).)
o ok

(g) char *p, string="Mango"; p= &string[0] ;


pC

(The string should be accessed through the pointer p.)


ar
Sh

(h) #define sqr(x) x*x


(Implement squaring of x using the macro definition.)

(i) int p[3][2]= Please clarify from your professor


(Initialize the array with natural numbers starting from 1.)

(j) FILE *fp= fopen(“input.txt”,”r”) ;


(Open the file named ”input.txt” in read mode.)

224
CHEMISTRY
MID - SPRING SEMESTER EXAMINATION2017-2018

Explanation: T − p(δT /δp)S = T − p(δV /δS)p


δU = −pδV + T δS

(δU/δS)p = −P (δV /δS)p + T

e
ki
oo
pC
ar

η = 1 − (Tc /Th ) It depends just only on Source and the Sink, Tc is temperature of sink and Th is the
Sh

temperature of the source.

225
For binary mixture , n1 δµ1 + n2 δµ2 = 0
⇒ δµ1 = −(n2 /n1 )δµ2
Thus δµ1 increases then δµ2 has to decrease ( increase in -ve value)

workdone(w) Tsink
η= heatgiven(Q) =1- Tsource
⇒ work done (w) = Q( 1 - TTsource
sink
)
3+273
⇒ W = 100 ( 1 - 20+273 ) e
ki
17
⇒ W = (100× 293 )KJ ⇒ W = 5.8KJ
oo
pC
ar
Sh

226
glhkgk

Sh
ar
pC
oo
ki
e

227
Ans. (δS/δV )T = (δP/δT )V [Maxwell Relation]
e
ki
P = nRT/(V-nb) - n2 a/V 2 = RT/( Vm − b) - a/Vm2
oo
pC

Hence, (δS/δV )T = (δP/δT )V = R/ ( Vm - b)


ar
Sh

228
(a) Diff in slope of µ vs T,

(δµ/δT )P,liq − (δµ/δT )P,solid

= ( −Sliq ) − (−Ssolid ) = −∆Strs

= −∆H∆ trs/Ttrs

Slope = -( 6 KJ/mol)/273K = - 21.97 J/mol . K

(b) ln ( P2 /P1 = −∆Hvap /R( T12 − 1


T1 )

p1 = 258.9torr T1 = 50.14K e
ki
oo

p2 = 161.2torr T1 = 277.5K
pC

∆Hvap = −0.241 KJ/mol


ar
Sh

ANS: Molecular mass of air = 28.97 g/mol


⇒ number of moles = 1000g /28.97 g/mol
= 34.52
Isobaric process ⇒ P constant ⇒ VT11 = VT22
V1
Final volume V2 = 3V1 ⇒ 300 = 3V
T2
1

Final temperature = 900K


q = nCp ∆T =34.52 × 29 × (900 − 300)
= 600.65 KJ
∆V = nCp ∆T =34.52 × (29 − 8.314) × (900 − 300) = 428.45KJ
From first law ∆U = q + W ⇒ W = ∆U − q = (428.45 - 600.65)KJ = -172.2KJ [ Alternatively , W may be
calculated from using then expression W =P∆V ]

229
ANS: dH = (δH/δP )T dP + (δH/δT )P dT
(δH/δT )V = (δH/δP )T (δP/δT )V + CP ( from definition of CP = (δH/δT )P )

Also α = V1 (δV /δT )P and κ = −1 V (δV /δP )T


From the cyclic rule applied to P,V,T
(δV /δP )T (δV /δT )P (δT /δP )V = −1
α 1
κ = V (δV /δT )P .(−V )(δP/δV )T = (δP/δT )V
⇒ (δH/δT )V = (δH/δP )T ακ + CP
(δH/δT )V = (δH/δP )T ακ + CP

e
ki
oo
pC
ar
Sh

230
Sh
ar
pC
oo
ki
e

231
Sh
ar
pC
oo
ki
e

232
CHEMISTRY
MID - SEMESTER EXAMINATION2016-2017

e
ki
oo
pC
ar
Sh

233
Sh
ar
pC
oo
ki
e

234
e
ki
oo
pC

Ans. 2.(a) n = 2moles


Vi = 44.8lit
ar

Ti = 27.3K
Sh

Pi = 1atm
expanded reversibly and isothermally
So ∆U = q + W
W = −nRT ln( vv21
→ q = −W
→ 3000 = nRT ln( vv21
→ ln( vv21 = 2×8.314×273
3000

v2 = 86.755lit
Final volume of the gas = 86.755lit.

2.b) (I)

235
(II) dS = dqTrev = dU −W rev
= Cv dTT+P dV = CvTdT + nR dV
R R dT RT dV V
∴ dS = Cv T + nR V
→ ∆S = Cv ln( TT21 ) + nRln( PP12 )
For step (I)
n=2
Cp = 2.5R P2 = 2atm
∴ ∆S1 = 2.5Rln(1) + nRln(1/2) = −11.53J/K
P1 = 1atm
T (constant)
∆S1 = −11.53J/K

2.c) lnP = −4124.4


T − 1.82(lnT ) + 34.48
Clausius clayperon Equation :
d(lnP ) ∆H
dP = RTvap
2

∴ d(lnP
dT
)
= 4124.4
T2
− 1.82 ∆
T = H sub RT
2
4 ∆Hsub 1.82
→ T 2 [4124.4 − R ] = T
Hsub
→ 4124.4 − 8.314 = 1.82 × 170
Hsub
→ 8.314 = 4124.4 − 309.4
= 3815
→ ∆Hsub = 3815 × 8.314 = 8172KJ
molar enthalpy of sublimation of ammonia at 170K = 8172KJ e
ki
oo
pC
ar
Sh

3.(a) To prove,
( δH
δP )T = Vm (1 − αT )
m

Using maxwell relation we know,

236
δH = V δP + T δS
→ ( δH δS
δP )T = V + T ( δP )T
We know ,
δS
( δP )T = −( δV
δT )P
∴ ( δP )T = V − T ( δV
δH
δT )P
but Vm ( δT )P = α → ( δV
1 δVm
δT )P = αVm
m

δHm
∴ ( δP )T = Vm − Vm αT
∴ ( δHδP )T = Vm (1 − αT )
m

( Proved )
δz
3.b) To prove: Cpm µgT = (RT 2 )( δT )P
We know , Cpm × µgT = ( δT )P ( δP )T as [ ( δH
δH δT δT δP
δT )P ( δP )H ( δH )T = −1]
δH = T δS + V δP
( δH δS
δP )T = T ( δP )T + V
δS
from maxwell’s relation ( δP )T = −( δV
δT )P
δH δV
→ ( δP )T = V − T ( δT )P
= Cpm µgT = T ( δV δT )P − V
now Z = RT = 1 + (b−a/RT
PV
RT
)P

→ P V = RT + (b − a/RT )P

3.(c) TheRprocess is irreversible


∆Ssys = dqTrev
− 10◦ (supercooled water ) (Q1 ) −→ 0◦ (water)Q2 −→ (Q3 ) − 10◦ (ice)
e
ki
dQ1 = 2lp R m dTdT
∆l1 = 2 Cpm = 150ln(T )]273
oo
T 263
273
∆S1 = 150ln( 263 )J/K
pC

dQ2 = −2∆Hf us = −2 × 6006 = −12012J


∆S2 = −12012 273 = −44J/K
ar

dQ3 = 2Cp R m dTdT


∆S3 = 2 Cpm = 76ln(T )]263 263
Sh

T 237 = 76ln( 273 )J/K


Entropy is a state function,
273
∆Srev = ∆Sirrev = 150ln( 263 ) − 76ln( 273
263 ) − 44
273
→ ∆Ssys = 74ln( 263 ) − 44
= −41.24J/K
Answer : ∆Ssys = −41.24J/K
→ V = RT P + (b − RT )
a
a
→ (V − b + RT ) = RT P
differentiating both sides w.r.t T at constant P.
→ ( δV a
δT )P − RT 2 = P
R

multiplying both sides by T.


→ T ( δV a
δT )P − RT = P
RT

→ V − T ( δV a RT
δT )P = V − RT − P − − − − − − − −
replacing V as in equation(2) in (1)
→ V − T ( δV δT )P = b − RT
2a
2a
∴ Cpm × µg T = RT − b(L.H.S)
(R.H.S)
z = 1 + (b−a/RT RT
)p
δz −bT 2aP
( δT )P = RT 2 + R2 T 3
RT 2 δz 2a
P ( δT )P = −b + RT
2
∴ RT δz 2a
P ( δT )P = RT − b(R.H.S)
(L.H.S) = (R.H.S)

237
2
∴ Cpm × µg T = ( RT δz
P )( δT )P
(proved)

e
ki
oo
pC
ar

.................
Sh

4.(a) Combustion reaction of methane :


CH4 + 2O2 −→ 2H2 O + CO2
∆G◦reaction = ∆G◦H2 O + ∆G◦CO2 − ∆G◦CH4
∆G◦O2 = 0(pure element)
→ ∆G◦reaction = [2 × (−237.3) + (−394.5) + 50.8]KJ/mol
= −818.3KJ/mol
Thus maximum electrical work that can be obtained from 1 mole of methane gas = 818.3KJ.
4.a) CH4 + 2O2 −→ CO2 + 2H2 O
∴ ∆Gnet = −394.5 + 2(−273.3) − (−50.8)
[∆Gproduct − ∆Greactants −→ Hess Law
=-818.3KJ 4.b) U (s) + 32 H2 (g) −→ U H3 (s)
PU H3 (s)
Kp = 3/2
PH PU (s)
2
PU H3 = 1 = PU as U and U H3 are solid
Kp = (PH3 )−2/3 = 0.97T orr−2/3
∆G = −RT ln(Kp ) = −R × 500ln(0.97)
∴ ∆G = 15.23R
4.c)Co3+ + e− −→ Co+2 E1 = 1.81V
2+ −
Co + 2e −→ Co E2 = 0.28V
∴ Enet = E1 +2E
3
2
= 0.417
∴ Co3+ + 3e− −→ Co 0.417V

3Cd −→ 3Cd + e − − 0.22V

238
∴ Enet = (0.417 − 0.22)V = 0.197V
When half cells are Enet = n1 En11 +n
+n2
2 E2

When cell reactions are added Enet = E+ + E− without multiplication of coefficient

e
ki
oo
pC
ar
Sh

239
CHEMISTRY
MID-AUTUMN SEMESTER 2016-17

e
ki
oo
pC
ar
Sh

240
Ans 1)  
∂G
(i) Ans: (a) µ1 =
∂n T,P,nj
Reason: Definition

(ii) Ans: (c) 4(γ± m)3


Reason: mAB2
mA + 2mB
activity = (γ± m)(γ± 2m)2

(iii) (c) µA (l) = µA (g) and µB (l) = µB (g)


Reason: For the components to be in equilibrium the chemical potential must be same in liquid and vapour
state.

(iv) Ans: (a) Melting of ice at 0o C and 1 atm.


e
ki
oo
(v) Ans: (a) µJT = 0
Reason: Definition
pC

(vi) Ans: (c) µ(O2 ) will remain unchanged.


ar

Reason: µ is an intensive property, does not depend on moles.


Sh

(vii) Ans: (a) 4GT,P < 0 , Ecell > 0


Reason: 4G < 0 means spontaneous.
4G = −nF Ecell
∴ for 4G to be -ve Ecell has to be > 0

(viii) Ans: (b) 4T /Th


Tsink
Reason: η = 1 − η = efficiency
Tsource
4T
η=
Th
w 4T
⇒ =
qh Th
4T
Fraction of heat converted to work is
Th
P
(ix) Ans: (b) dG = V dP − Sdt + µdn
Reason: Definition

241
Ans 2a): To  prove:
  
∂G ∂G
U =G−P −T . . . . . . . . . (1)
∂P T ∂T P
From the Gibbs equation,
∂G =V ∂P − S∂T

∂G
= V . . . . . . . . . (2)
 ∂P T
∂G
= −S . . . . .. . . . (3)
∂T P
Putting (2) and (3) in RHS of (1)
e
ki
U = G − P V + ST
Now substituting in G = H − T S
oo

⇒ U = H − T S − P V + ST
pC

⇒ U = H − PV    
∂G ∂G
⇒U =G−P −T [proved]
ar

∂P T  ∂T P 
Sh

  
∂H 2 ∂P ∂(T /V )
Ans 2b): To Prove: = −V
∂V T ∂T V ∂V P
RHS:  
∂P ∂(T /V )
−V 2
∂T
   ∂V P 
2 ∂P V ∂T − T ∂V
= −V 2
 ∂V V  V ∂V  P
∂P ∂T
= T −V
∂T
 V  ∂V P 
∂P ∂T ∂P
=T −V
∂T V ∂V P ∂T V
Usingcyclic
  rule,
  
∂V ∂P ∂T
= −1
∂P
 T ∂T  V ∂V P 
∂T ∂P ∂P
∴ =−
∂V
 P ∂T  V  ∂V T
∂P ∂P
=T +V
∂T V ∂V T   
∂P ∂S
from Maxwell relation, =
    ∂T V ∂V T
∂S ∂P
=T +V
∂V T ∂V T
from Gibbs equation,
∂H = T ∂S + V ∂P

242
     
∂H ∂S ∂P
⇒ =T +V
∂V T ∂V T ∂V T
∴ LHS = RHS [proved]

Ans 2c): Vi = 75 litres


Vf = 20 litres
Ti = 45o C = 318K
q=0 (adiabatic process)
Cv = 2.5R
Cp = 3.5R
Cp 7
γ= =
Cv 5
PV γ = c
c
⇒ T V γ−1 = c0 [where c0 = ]
cR
⇒ Ti Vi γ−1
=c 0

⇒ 318 × (75)2/5 = c0
⇒ c0 = 1788.35 lit2/5
Z K
Vf
Work done W =− P dV
Vi
Z Vf
cdV
=−
Vi Vγ
V
c[V1−γ ]Vfi
=−
 1−γ
c (Vf )1−γ − (Vi )1−γ

e
ki
=
 γ−1
oo

c0 (20)−2/5 − (75)−2/5

=
pC

nR 0.4
1788.35 (0.1239)
=
ar

2 × 0.082 0.4
= 3377.69 atm lit
Sh

= 342.23 KJ
∴ W = 342.23 KJ
4U=q+W
∴ 4U = 342.23 KJ
Tf Vfγ−1 = Ti Viγ−1
 γ−1
Vi
Tf = Ti = 539.56 K
V −f
4H = 4U + 4(V P )
=4U + nR 4 T
=342.23 + 2 × 8.314
=(342.23 + 3.68) KJ
= 345.9 KJ
∴ 4H = 345.9 KJ
q=0
W = 342.23 KJ
4U = 342.23 KJ
4H = 345.9 KJ

243
e
ki
oo
pC
ar
Sh

R
And 3a): Q = T dS
For AB, 4S = 100 J/K
Q = 600 × 100 = 60 KJ
For BC and DA, 4S = −100 J/K
Q = −300 × 100 = −30 KJ
Net heat delivered by source = 60 KJ
Net heat taken by the engine = 60 − 30 KJ = 30 KJ
4U = q + W
⇒ −W = q
⇒ W = −q
⇒ W = −30 KJ
(i) Work delivered by the engine = 30 KJ
(ii) Heat taken from the source in each cycle = 60 KJ
(iii) 4Ssystem + 4Ssurrounding = 0 [for reversible process]
In the process CD, surrounding = sink
∴ 4Scarnot−engine + 4Ssink = 0
or −100 J/K + 4Ssink = 0
or 4Ssink = 100 J/K
1625
Ans 3b): log(P1 ) = 3.750 − . . . . . . (1)
T
244
(i) According to Clausius-Clapeyron equation,
1 ∂P 4Hvap
= . . . . . . . (2)
P ∂T RT 2
differentiating eq (1),
1 ∂P1 1625
= . . . . . . . . . . (3)
P1 ∂T T2
Comparing (2) and (3)
1625 4Hvap
2
=
T RT 2
⇒ 4Hvap = 1625R
⇒ 4Hvap = 13.5 KJ/mol
(ii) at normal boiling point,
vapour pressure (Pi ) = 1 atm = 760 torr
1625
log(Pi ) = 8.750 −
T
at T = 15o C = 288 K
1625
log(Pi ) = 8.750 −
288
⇒ log(Pi ) = 3.11 → at 15o C
Clausius-Clapeyron
  equation,
P1 4 1 1
ln = R −
P2 H vap T2 T1  
4Hvap 1 1
⇒ ln(760) − ln(P2 ) = −
R 288 T
1625R 1 1
⇒ 6.63 − 3.11 = −
R 288 T
e
ki
1 1
⇒ 2.16 × 10−3 = −
288 T
oo

On solving,
⇒ T = 762.1 K
pC

(The answer however seems to be unrealistic. Please consult your professor.)


ar

Ans 3c): p → Triple point


q → melting/freezing point
Sh

r → boiling point
s → critical point

Ans 4a): (i) Increase in pressure will result in increase in density because same amount of mass will be
confined within smaller amount of volume. Thus density will increase.

245
Thus increase in pressure will favour conversion of graphite into diamond.
(ii) dG = V dP − SdT
At T = 298 K, dT = 0
∴ dG = V dP Now 1 mol = 149
∴ d(Gdiamond − Ggraphite ) = (Vgraphite − V diamond)dP
At equilibrium ,
Ggraphite = Gdiamond
∴ dGdiamond = Gdiamond − Go diamond
Similarly for graphite.
∴ Godiamond − Gographite = (Vgraphite − Vdiamond )dP
 
1 1
or 2900 = 14 − × 10−6 (Pf − Pi )
2.25 3.51
14 × 1.26
or 2900 = × 10−6 (Pf − Pi )
2.25 × 3.51
or Pf − Pi = 12983 bar
Pi = 1 bar
∴ Pf = 12984 bar

Ans 4b): (i) 2e− + 2H+ −→ H2 : anode


1
2OH− −→ H2 O + O2 + 2e− : cathode
2
1
Cell reaction : 2H+ + 2OH− −→ H2 O + O2 + H2
2
∴ 4f Go = −nF E
or −237.13 × 103 = −1 × 96500 × E e
ki
or E = 2.46 V
oo

∴ −nF E = −RT ln K
nF E 4Go
or ln K = =
pC

RT RT
237130
or ln K =
ar

8.314 × 298
or K = 3.686 × 1041
Sh

246
CHEMISTRY
END-SPRING SEMESTER 2016-17

Ans 1a):

e
ki
oo
pC
ar
Sh

Ans 1c): Work Function of metallic Cesium (wo ) = 1.14 eV


Wavelength of light (λ) = 700 nm = 7 × 10−7 m
hc
Energy associated with light = = 2.84 × 10−19 = 1.77 eV
λ
Energy associated with light > wo
The light of wavelength 700 nm can eject electrons from metallic Cesium of 1.14 work function.
Ans 1b):

247
Ans 2):
10 − 8
Bond Order of O2−
2 = =1
2
10 − 6 e
ki
Bond Order of N2−
2 = =2
2
oo

10 − 6
Bond Order of O2 = =2
2
pC

10 − 4
Bond Order of N2 = =3
2
ar

B.OO2− < B.OO2


2
Sh

Stability of O2−
2 < Stability of O2

B.ON 2− < B.ON2


2
Stability of N2−
2 < Stability of N2
Ans 3a): Bridging ligands :
Chelating Ligands :

Ans 3b): Complex I :


4G = −RT ln β = −2.01RT
Complex II:
4G = −RT ln β = −2.36RT
4G = 4H − T 4 S
Although complex I and Complex II have comparable 4H there is a substantial difference in their formation
constant (β) because of difference in their 4S o . Complex II with higher value of lnβ most probably exhibit
the phenomenon of chelation.(higher value of 4S thus higher value of lnβ)
Ans 4a): Two types of Jahn-Teller distortions are Z-in and Z-out.

248
e
ki
oo
pC

Z-out distortion is most preffered.


ar

Ans 4b): Splitting of [Co(H2 O)6 ]2+ :-


3 2 6 4
Sh

C.F.S.E. = 2 × 4o − 4o ×5 + 2P = 4o −2 4o +2P = − 4o +2P


5 5 5 5
[Co(H2 O)6 ]+2 + HCl(conc.) −→ [Co(Cl)4 ]−2
Water is a comparatively stronger ligand as compared to Cl− . So the splitting energy of [Co(H2 O)6 ]+2 is higher
than that of [Co(Cl)4 ]−2 and thus e− in [Co(H2 O)6 ]+2 need more energy to get excited so it will absorb higher
energy light waves and reflect lower energy light waves to look pinkish, whereas for [Co(Cl)4 ]−2 the splitting
energy gap between eg and t2g orbitals is smaller so tit will absorb lower energy light waves to excite electrons
and emit the higher energy light waves and thus appear bluish.

249
e
ki
oo
pC

Ans 5a):
ar
Sh

250
e
ki
oo
pC
ar
Sh

Ans 5b) Pre-catalyst of Monsanto acetic acid process is: [Rh(CO)2 I2 ]−


(repeat question from 2015 endsem question paper)
Ans 6a): (a) 2ClO− (aq) + 2H2 O(l) + 2e− −→ Cl2 (g) + 4OH− (aq) Eo = 0.4 V
Cl2 + 2e− −→ 2Cl− Eo = 1.35 V
1 × 0.04 + 1 × 1.35
Eo = A = = 0.875 V
2
A = 0.875 V
0.40 1.35
(b) ClO− −−→ Cl2 −−→ Cl−
The potential on the left of Cl2 is less positive than that on the right; therefore Cl2 can oxidize and reduce
itself; thus undergoes disproportionation.
Half cell reaction :
Cl2 (g) + 4OH− (aq) −→ 2ClO− (aq) + 2H2 O + 2e− Eo = 0.40 V
CL2 + 2e− −→ 2Cl− Eo = 1.35 V
Full cell reactions :
2Cl2 + 4OH− −→ 2ClO− + 2H2 O + 2Cl−
Cl2 + 4OH− −→ ClO− + H2 O + Cl− Eo = 0.95 V

251
Eo
(c) ClO−4 −→ Cl2
0.35 × 2 + 0.3 × 2 + 0.70 × 2 + 0.4
Eo =
7
nE = 3.1
e
ki
n=7
Thus ClO− 4 is the rightmost element on the frost diagram with highest nE value, so is the strongest oxidizing
oo

agent.
pC

Fe3+ + e− −→ Fe+2 →(1) EoF e+3 /F e+2 = 0.77 V



Ans 6b):
ar

Sn+4 + 2e− −→ Sn+2 →(2) EoSn+4 /Sn+2 = 0.15 V



Sh

(1)× 2 - (2) :
2Fe3+ + Sn+2 −→ 2Fe+2 + Sn+4 EoF e+3 /F e+2 = 0.62 V


Eo is positive so 4G is negative so the above reaction is spontaneous. So Sn+2 will be oxidized by Fe+3 to
Sn+4 .

Question 7 is out of syllabus.

252
Sh
ar
pC
oo
ki
e

253
Sh
ar
pC
oo
ki
e

254
Sh
ar
pC
oo
ki
e

255
e
ki
oo
pC
ar
Sh

conc. of equatorial conformer


K=
conc. of axial conformer
Energy difference between axial and equatorial conformers (4H) = 2 × 0.9 × 4.2 = 7.56 KJ/mol
∵ it is at equilibrium,
4Go = 4H o − T 4 S o [but T 4 S o = 0]
⇒ 4G = 4H
⇒ −RT ln(K) = −7.56 × 103
1
K = 20.7 Ans: = 0.0483
20.7
256
Sh
ar
pC
oo
ki
e

257
Sh
ar
pC
oo
ki
e

258
Sh
ar
pC
oo
ki
e

259
Sh
ar
pC
oo
ki
e

260
e
ki
oo
pC
ar
Sh

Ans 4a): (i) ψx − HOMO ψy − LUMO


(ii) Q denotes supra-supra ring closure.
(iii) P denotes supra-antara ring closure.
(iv) Process Q will be allowed thermally.

261
Sh
ar
pC
oo
ki
e

262
CHEMISTRY
END-AUTUMN SEMESTER 2015

INORGANIC

Ans 1a): Work function (wo ) = 2.5eV


Wavelength of photon (λ) = 6800Å ' 6.8 × 10−7 m
hc
Energy of the photon =
λ
=2.923 × 10−19 J
=1.827eV
Energy of the photon < Work Function (wo )
Photon with wavelength 6800Å is not suitable to eject photoelectrons from sodium of work function 2.5eV.
Ans 1b):
e
ki
oo
pC
ar
Sh

263
Ans 2a): B2 ’s molecular arrangement :
π2p1x ∗
π2p x
∗ ∗ ∗
σ1s2 σ1s2 σ2s2 σ2s2 σ2pz π2p1 σ2pz ∗
σ2p
x z
π2p1y ∗
π2p y

e
ki
oo
pC
ar
Sh

MO diagram of B2

There are 2 unpaired electrons and therefore this explains B2 ’s paramagnetism.


The suffix g and u are use in case of symmetrical and antisymmetrical with respect to its center, respectively.
ΨA (1s) + ΨB (1s) . . . . . . . . . g
ΨA (2pz ) + ΨB (2pz ) . . . . . . . u
σ −→gerade π −→ungerade
σ ∗ −→ungerade π ∗ −→gerade

Ans 2b): For O2 :


No. of anti-bonding e− (A.B.E)= 6
No. of bonding e− (B.E) = 10
BE − ABE 10 − 6
bond-order = = =2
2 2
Bond Order = 2
For O2− :
No. of anti-bonding e− (A.B.E)= 7
No. of bonding e− (B.E) = 10
BE − ABE 10 − 6
bond-order = = = 1.5
2 2
Bond Order = 1.5

264
1
bond − length ∝
bond − order
as Bond-OrderO2 > Bond-OrderO−
2
So, Bond-LengthO2 < Bond-LengthO−
2
Thus O2− has longer bond length as compared O2 .

Ans 3a): Metallic copper is a good conductor due to the following reasons :-
• It has free electrons.
• The conduction and valence band overlap. There is no band gap so electrons are free to move.
Ans 3b): Ligands which can donate electrons to ligands (sigma donor) and also accept electron from metal
are called pi acid ligand. the latter is called back bonding and the effect is called synergic effect.
Example: Ni(CO)4
Ans 3c):

e
ki
oo
pC
ar
Sh

Carbonyl ligand is considered as a weak 2-electron σ-donor and very strong π-acceptor. Two types interactions
are involved int he complexation of carbonyl with transition metal ion. The carbonyl donates its lone pair to the
vacant metal d(σ) orbital and back bonding occurs from metal dπ to C-O π ∗ orbital. Thus via the phenomenon of
back π bonding and synergic effect π-acid ligands like CO stabilizes metal ions with low oxidation state forming
stable organometallic complexes. To stabilize low oxidation state we require ligands which can simultaneously
bind the metal center and also withdraw electron density from the metal ion which is very high in electron density.

0.90 1.28 2.9 0.95 1.5 -1.18


Ans 4a): MnO− −
−→ MnO2 −−→ Mn+3 −−→ Mn+2 −−−→Mn
4 −−→HMnO4 −−→ H3 MnO4 −
-1.18
(I) Mn+2 −−−→ Mn
n=2

265
nEo = -2.36 V
E
(II) Mn+3 −→ Mn
2 × (−1.18) + 1 × 1.5
Eo = = −0.287V
o
3
nE = −0.86V
n=3

(+4) E
(0)
(III) Mn O2 − → Mn
2 × (−1.18) + 1.5 × 1 + 1 × 0.95
Eo =
4
nEo = 0.09V
n=4

(+5) E
(0)
(IV) H3 Mn O4 − → Mn
0.09 + 2.9 × 1
Eo =
5
nEo = 2.99V
n=5

(+6) E
(0)
(V) H Mn O− 4 −
→ Mn
2.99 + 1.28 × 1
Eo =
6
nEo = 4.27V e
ki
n=6
oo

(+7) (0)
pC

E
(VI) Mn O− 4 −→ Mn
4.27 + 1 × 0.09
Eo =
ar

7
nEo = 5.17V
Sh

n=7

266
Ans 4b): From the Frost diagram it is clear that Mn+3 lies above the line joining Mn+2 and MnO2 . So Mn+3
in acidic medium is unstable and has the tendency to disproportionate into Mn+2 and MnO2 .

Ans 5a): LA is the strong field ligand.


Reason: Causes pairing, so less unpaired electrons and thus [Fe(LA )6 ]−3 has comparatively lesser magnetic
moment.
[see part (b) for detailed diagramatic analysis]

Ans 5b): [Fe(LA )6 ]−3 −→ Fe+3 −→ 3d5 4s0

e
ki
oo
pC
ar
Sh

unpaired
p electrons(n) = 1
µ = n(n + 2) = 1.73 BM

[Fe(LB )6 ]−3

(no pairing as LB is weak field ligand)


unpaired electrons(n) = 5

267
p √
µ = 5(7) = 35 = 5.92 BM
Ans 5c): CFSE for [Fe(LA )6 ]−3 :
3 2
4o ×0 − 4o ×5 + 2P = −2 4o +2P where P = pairing energy
5 5
CFSE for [Fe(LB )6 ]−3 :
3 2
4o ×2 − 4o ×3 = 0
5 5
CFSE for [Fe(LA )6 ]−3 = −2 4o +2P
CFSE for [Fe(LB )6 ]−3 = 0

Ans 6a):

e
ki
oo
pC
ar
Sh

In Cr+2 , there is degeneracy (assymetry) in eg orbitals, so it will show strong Jahn-Teller distortions.
In Fe+2 there is degeneracy (assymetry) in t2g orbitals so it will show weak Jahn-Teller distortions.
Ans 6b): When CN− is treated with Ni+2 (aq)
Ni+2 + 2CN− + nH2 O −→ Ni(CN)2 .nH2 O (green colour)
Now if excess CN− is added,
Ni(CN)2 + 4CN− −→ [Ni(CN)6 ]4−
Configuration of Ni2+ in [Ni(CN)6 ]4− is 3d8 4s0
Splitting of d-orbital and electron distribution for [Ni(CN)6 ]4−

268
Ans 7a): For the given complexes [Co(NH3 )6 ]+2 , [Co(H2 O)6 ]+2 and [CoCl4 ]−2 the order of ligand strength is
Cl− <H2 O < NH3
Thus 4o [Co(N H3 )6 ]+2 > 4o [Co(H2 O)6 ]+2 > 4o [CoCl4 ]−2

Now, since 4o [Co(N H3 )6 ]+2 is highest among the given three complexes, so to excite its electrons more energy
e
ki
is required (more 4o means more gap between eg and t2g ) therefore it must absorb high energy (high frequency)
light wave such as violet so from complementary colour chart if it absorbs violet then observed colour must be
oo

yellow.
pC

Similarly, 4o [Co(H2 O)6 ]+2 is second highest and so it must absorb high energy light wave and thus its observed
colour must be lesser in energy. Among pink and blue, pink is lesser energy so [Co(H2 O)6 ]+2 must be pink and
ar

[CoCl4 ]−2 must be blue.


Sh

Remember: Higher the splitting energy (4o or 4T ) higher is the gap between eg and t2g and higher is the
energy required to excite electrons, so a complex with higher splitting energy will absorb high energy light wave
but will have a lower energy colour in accordance with complementary colour chart.
1
Splitting energy ∝ ∝ Energy of absorbed colour
Energy of observed colour

2”en” 4MeNH
Ans 8a): [Cd(en)2 ]+2 ←−−−−−−−−−−−−−−−−−−Aqueous Cd2+ −−−−−−−−−−−−2−−−−−→ [Cd(MeNH2 )4 ]+2
logβ=10.6;4H=−56KJ/mol logβ=6.5;4H=−57KJ/mol
(I) (II)
4G = 4H - T4S

269
⇒ -RT ln(β) = 4H - T4S → (i)
now, for (I)
G = -RTln(β)
I = -10.6RT
for (II),
G = -RTln(β)
II = -6.5RT
now, G(I) < G(II)
So (I) is more spontaneous than reaction (II).
from (i),
−4H 4S
ln(β) = +
RT R
−4H
for both (I) and (II) reactions is almost same but the marked differences in ln(β) is because of difference
T
in 4S.
In (I) ’en’ is a chelating ligand , so 4S is very high for the reaction where as for Cd2+ to [Cd(MeNH2 )4 ]4+ the
change in S is not so high so 4S is low.

Ans 8b): The EDTA4− when injected in human body readily forms complexes with Pb+2 which due to
chelation have very high stability constant. Therefore the complexes can be easily flushed out of human body and
thus removing poisonous metals like Pb2+ .

e
ki
oo
pC
ar
Sh

Ans 9a): The active catalyst used in Monsanto acetic acid process is rhodium carbonyl iodide catalyst
[Rh(CO)2 I2 ]− .

Ans 9b):

270
Ans 9c):

Type of reaction: Oxidative addition of CH3 CH2 I.

Question (10) is out of syllabus.

e
ki
oo
pC
ar
Sh

Ans 1a):

271
Ans 1b):

e
ki
oo
pC
ar
Sh

272
Ans 1c): No. of stereogenic centers (compound with N)= 5
No. of stereogenic centers (compound with P) = 3

e
ki
oo
pC
ar
Sh

273
e
ki
oo
pC
ar
Sh

The OR part of the question is not in syllabus.

274
e
ki
oo
pC
ar
Sh

cis is optically inactive due to plane of symmetry, but due to two methyl groups being in equatorial position
cis 1,3-dimethyl cyclohexane is more stable compared to its trans version which by the way is optically active. cis-
1,3 dimethyl cyclohexane have no gauche-butane interaction while trans-1,3 dimethyl cyclohexane have 2 gauche
butane interaction owing to which it is comparatively less stable.
1 gauche-butane interaction aguments the energy of cyclohexane by 0.9 Kcal/mol.
So 2 gauche-butane interactions are responsible for 1.8 Kcal/mol more energy for the trans counterpart thus
making it coparatively less stable.

275
Ans 3a): (i) Faster SN 2 with NaOMe :
1-bromo-3-methylbutane ; reason: alkylhalides favours SN2
(ii) Faster SN 1 with EtOH :
2-bromo-2-methylbutane ; reason: 3o alkylhalides favour SN 1
(iii) optically active product when reacted with NaN3 via SN 2 mechanism:
2-bromo-3-methylbutane ; reason: the resultant product will be optically active

e
ki
oo
pC
ar
Sh

In (3) we clearly see that no chlorine is there in axial position, so no anti-elimination is possible with it. E2 is
favoured by anti-elimination, so (3) is least favoured to undergo E2 mechanism.

276
Sh
ar
pC
oo
ki
e

277
Sh
ar
pC
oo
ki
e

278
Sh
ar
pC
oo
ki
e

279
CHEMISTRY
END-AUTUMN SEMESTER 2012

e
ki
oo

Ans 1a):
pC
ar
Sh

n1 E1o + n2 E2o
Eo =
(n1 + n2 )
1 × 1.63 + 1.36 × 1
=
2
= 1.495V
The Eo value for reduction of HClO to Cl− is 1.495 V.
+0.70 +1.76
Ans 1b): O2 −−−→ H2 O2 −−−→ H2 O
Eo
∴ O2 −→ H2 O
(0) (−2)
∴ n = −2

280
0.07 × 1 + 1.76 × 1
Eo = = 1.23V
o
2
nE = −2.46V
+0.70
∴ O2 −−−→ H2 O2
∴ n = −1
nE o = −0.70V

e
ki
Ans 1c): Velocity of the atom (v) = 1000ms−1
h
oo

De-Broglie wavelength (λ) =


mv
(mass of e− can be neglected)
pC

mass of the atom = 2× mass of proton + 2×mass of neutron


= 6.63 × 10−27 Kg
ar

mv = 6.63 × 10 −24 Kgms−1


h 6.626 × 10−34
Sh

λ= = = 1 × 10−10 = 1Å
mv 6.63 × 10−24
Ans 1d): Work function of metal (wo ) = 3.44 × 10−19 J
velocity of ejected photon = 1.03 × 106 ms−1
From Einstein’s photoelectric equation,
1 hc
mv = − wo
2 λ
hc 1
⇒ = × 9.1 × 10−31 × (1.03 × 106 )2 + 3.44 × 10−18
λ 2
hc
⇒ = 4.83 × 10−19 + 34.4 × 10−19 = 39.23 × 10−19
λ
hc 6.626 × 10−34 × 2.997 × 108
⇒λ= −19
= = 0.51 × 10−8 m = 51Å
39.23 × 10 39.23 × 10−19

Ans 2a): Bond Order ∝ Bond Strength

281
e
ki
oo
pC
ar
Sh

Since, Bond Order of N2 > Bond Order of N+ 2


so Bond Strength N2 > Bond Strength of N+ 2
∴ more energy is required to break the bond of N2 than N+ 2
The orbitol diagram of N2 and N+ 2 are given in the next page,
∴ dissociation energy of N2 is greater than N+ 2.

Ans 2c): Molecular orbital configuration of O2 , N2 and B2

282
π2p2y ∗
π2p 1
y
O2 : σ1s2 ∗
σ1s σ2s2 ∗
σ2s σ2p2x
2 2

π2p2z ∗
π2p 1
z
10 − 6
B.O. = =2
2
π2p2y
N2 : σ1s2 ∗
σ1s σ2s2 ∗
σ2s σ2p2x
2 2

π2p2z
10 − 4
B.O. = =3
2
π2p1x
σ1s2 ∗
σ1s σ2s2 ∗
σ2s
B2 : 2 2
π2p1y
6−4
B.O. = =1
2
We know B.O ∝ Bond dissociation energy
Since, B.O.N2 > B.O.O2 > B.O.B2
N2 , O2 and B2 in decreasing order of bond dissociation energy is:
N2 > O2 > B2
On removing an electron , the electron will be removed from Bonding orbitals in case of N2 and B2 , but
for O2 it will be removed from an anti-bonding orbital. (see MO configuration)
Thus on removing an electron bond order of O2 will increase so O+ 2 will be more stable than O2 .
+ +
e
But for N2 and B2 , N2 and B2 will have lesser bond order respective to their parent element and so will
ki
be less stable with respect to them.
oo
pC
ar
Sh

Ans 3a): Down the group the metal orbitals are more diffused so the overlap with the ligand orbital is
better and therefore down the group 4o increases.
So for [Fe(CN)6 ]−3 , [Ru(CN)6 ]−3 and [Os(CN)6 ]−3
the value of 4o in increasing order would be
[Fe(CN)6 ]−3 > [Ru(CN)6 ]−3 > [Os(CN)6 ]−3

Ans 3b): Cd+2 + 2en = [Cd(en)2 ]+2


4G = −60 KJ/mol , 4H = −55 KJ/mol , T= 25o C= 298 K
4G = 4H − T 4 S
4H − 4G −55 + 60
⇒ 4S = = = 0.168 KJ mol−1 K−1 ≈ 16.8 J/mol/K
T 298
Ans 3c): Orbital splitting of Co+3 under the influence of weak and strong field ligands is shown tn the
next page:

283
e
ki
oo
pC
ar
Sh

Carbonyl is cosidered a weak 2 electron σ donor adn π acceptor. Two types of interactions are involved in
the complexation of carbonyl with transition metal ion. The carbonyl donates its lone pair to vacant metal
d(σ) and back donation occurs from metal dπ orbital to C-O π* orbital.
Ans 4c): (i) [Cu(H2 O)6 ]+2 + en = [Cu(H2 O)4 (en)]+2 + 2H2 O
log β1 = 10.6
(ii) [Cu(H2 O)]+2 + 2NH3 = [Cu(H2 O)4 (NH3 )2 ]+2 + 2H2 O
log β2 = 5.0
the formation constant β1 in reaction (i) is higher compared to reaction (ii) because in reaction (i) the
product formed has chelating ligand ”en”. Thus 4S of reaction (i)log β1 is higher compared to log β2 .

284
A: meso ; B: chiral (no plane of symmetry) ; C: chiral (no symmetry) ; D: chiral ; E: chiral

e
ki
oo
pC
ar
Sh

285
e
ki
oo
pC
ar
Sh

Ans 3) (A) Ph3 CCl3 will react under SN 1 mechanism because via the mechanism Ph3 C+ (tri-phenyl
cabocation) will be formed which is highly stable intermediate.
(B) B will react under SN 2 mechanism as the carbon attached to Bromine is 1o and SN 2 is favoured in
1o carbon. (also the carbocation intemediate for SN 1 would be highly unstable due to the O attached to the
adjacent carbon)

286
(C) C will react under SN 2 mechanism. Although the carbocation intermediate for SN 1 is benzyl carbo-
cation but due to the presence of strong -M.E. ’N2 O’ group the carbocation would become unstable. So SN 2
will be favoured more.
(D) D can react via both SN 1 and SN 2 because 1o carbon but the carbocation can be stabilized byconfu-
gation with lone pair of Sulphur .
(E) E can react via both SN 1 and SN 2. the carbocation will be stabilized fue to conjugation with the double
bond (allyl carbocation) but the nature of alkyl halide is 1o .

e
ki
oo
pC
ar
Sh

287
Sh
ar
pC
oo
ki
e

288
e
ki
oo
pC
ar
Sh

Therefore the product will be ’S’ configured.


Note: SN 2 does not guarantee an inversion of absolute configuration. You will have to consider the arrange-
ment and priority order of species around the chiral carbon in the product and assign ’R’ or ’S’ accordingly.

289
Ans 8):

Sh
ar
pC
oo
ki
e

290
MECHANICS
MID-SPRING SEMESTER EXAMINATION 2017-2018



Ans: The given force system consists of a force F = 1000ĵ N passing through (0,0) and a couple of moment

− e
ki
C = 0.2 × 50k̂ (N-m)= 10k̂ (N-m)
oo

Let us find the equivalent force-couple system at (x,0) where x is expressed in meter.
 The equivalent force-
pC


ˆ
couple system is a force R̂ = 1000ĵ (N) and a couple of moment Ceq = Ĉ − (1000x)k̂ = 10 − 1000x k̂ (N-m)
ar

10
If x = = 0.01 m, then the equivalent force system consists of a single free R̂ , only. Hence,
Sh

1000
(a) The magnitude of R̂ is 1000(N) and it’s direction is along positive y-axis i.e ĵ

(b) the equation of the line of action is x = 10 (where x is in mm)

Ans a): The FBD if the cantilever beam is drawn below

291
b) Only two equations of equilibrium are obtained from the above free body diagram that relate the vertical
forces and the moment.

They are,
R2 + R3 = F sin(30o )
R2 × (L/2) = R2 × (L/2) + M
Thus it is not possible to get R3 , the reaction force at the roller.

Ans a): The FBDs are as shown below

e
ki
oo
pC
ar
Sh

b): Given |FBD | = 5kN


Considering FBD of member ABC and using the following equilibrium equations
Mc = 0 ⇒ FBD cos(θ) × 0.2 − P × 0.12 . . . . . . . . . . . . . . (1)
0.2 × cos(θ)
We get P = FBD × = 1.2804 FBD
0.12
Since P¿0 (for the direction is given), we take FBD = 5kN and accordingly P = 6.402 kN
c): Consider the FBD of member ABC. The following equations of equilibrium are written
Cx + FBD sin(θ) = P . . . . . . . . . (2)
Cy + FBD cos(θ) = P . . . . . . . . . (3)
From equations (2) and (3) we get
Cx = 3.2kN
Cy = 3.84kN q
The reaction force at C has magnitude Cx2 + Cy2 = 5kN and direction along −50.2o from positive x-axis.

292
Ans a): The FBD of the panel is drawn below

e
ki
oo
pC
ar
Sh

b): Considering moment balance equation about the axis AD, we get
−−→  −−→  0.1
Mx = 0 ⇒ T AB × n̂BG .î + T DC × n̂CG .î − × 400 = 0 . . . . . . (1)
−−→ −−→ 2
where T is in Newton, AB = 0.1ĵ (m), DC = 0.1ĵ (m)

r→ −

G − rB −0.05î − 0.1ĵ + 0.1k̂
n̂BG = − → −
→ =p
|rG − rB | (0.05)2 + (0.1)2 + (0.1)2
−→ −

rG − rC 0.15î − 0.1ĵ + 0.1k̂
n̂CG = − →| = p
|r→
G − −
rC (0.15)2 + (0.1)2 + (0.1)2
From equation (1) we get
 0.1 × 0.1 0.1 × 0.1 
T p +p = 20
(0.05)2 + (0.1)2 + (0.1)2 (0.15)2 + (0.1)2 + (0.1)2
i.e. T = 173.65(N)

293
Ans a): Consider joint-A whose FBD is shown below

e
ki
oo
pC
ar

P
From equilibrium equations ( Fy = 0) we get
Sh

FAB sin(60o ) + 6000 = 0 . . . . . . . . . . . . (1)


2
i.e. FAB = −6000 × √ (N)
3
We further note that member BP is a zero-force member. Next consider joint B. The FBD is shown above.
The following equations of equilibrium are obtained
FAB cos(30o ) + FBD cos(30o ) = 0 . . . . . . . . . . . (2)
FAB sin(30o ) = FBC + FBD sin(30o ) . . . . . . . . . (3)
1200
We thus get FBD = −FAB = √ (N) = 6.928kN,
3
The force in member BD is 6.928kN (T).

c): The zero force members are:


BP, CO, DN, EM, FL, GK, HJ

294
Ans a): FBDs of the blocks for P=0 are shown below (knowing that blocks are in equilibrium)

e
ki
oo

b): For impending downward motion of Block B the FBDs are shown below,
pC
ar
Sh

c): The following equations pf equilibriumPare obtained from the free body diagrams shown in (b)
Block-A: FAB cos(θ) = µs NA ( FPx = 0) . . . . . . . (1)
FAB sin(θ) + 300 = NA P ( Fy = 0) . . . . . . . . .(2)
Block-B: FAB cos(θ) = NB ( Fx = 0) .P . . . . . . (3)
FAB sin(θ) + µs NB = P + 50 ( Fy = 0) . . . . . . . . .(4)
From equations (1)-(4) we get, 
sin(θ) + µs cos(θ)
P = FAB (sin(θ) + µs cos(θ)) − 50 = 300µs  − 50 = 17.06 N
cos(θ) − µs sin(θ)
The required downward force is 17.06 N.

295
Ans a): The FBD of the pulley + belt is shown below

e
ki
oo
pC

b): When the weight is moving upward with uniform speed


T = W eµx π = 1.602 kN
ar

From the FBD shown in (a) we get the following equilibrium equation after taking moment about the center
Sh

of the pulley.
M = (T − W ) × r
since M = Kt θ, we get
(T − W ) × r W (e( µx π) − 1)r
θ= = = 0.301 rad
Kt Kt
o
The pulley rotates by 17.2454 in the clockwise direction.

296
MECHANICS
MID - AUTUMN SEMESTER EXAMINATION2017-2018
1. A woman supports an 80 kg homogeneous box on a horizontal rough ledge by providing only an upward
vertical force at the corner B, as shown in the figure . We need to determine the range (Fmin , Fmax ) within
which the vertical force at box B must lie for keeping the box in equilibrium without tilting or moving it from
the horizontal position shown.

(a) Draw two separate free body diagrams corresponding to FBmin and FBmax . (4)

(b) Determine the range FBmin , FBmax . Take g = 10m/s2 (12)

e
ki
oo
pC
ar
Sh

ANS:
(a) Only verticle forcwe applied at B. The box may tilt about the point C or A.
For FBmin the box will tilt about point C. ForFBmax the box will tilt about point A.

(b)

297
FBmin : FBmin :
P P
Mc = 0(left FBD) MA = 0(right FBD)
FBmin × 0.7 − mg × 0.2 = 0 FBmax × 1 − mg × 0.5 = 0
⇒ FBmin = 72 mg = 27 × 80 × 10N ⇒ FBmax = 12 mg = 40 × 10N
⇒ FBmin = 228.57N ⇒ FBmax = 400N

The range of FB to maintain equilibrium is

( FBmmin , FBmax ) = (228.57N, 400N )

e
ki
oo
pC
ar

ANS:
Sh

(b)


r−

AB = √7 (î + ĵ + k̂)m ( as AB makes equal angle with x, y and z axes)
3

r−

AD = √4 (î + ĵ + k̂)m
3
r−−→
AB = (6ĵ + √73 k̂)m
⇒− r−
→ −−→ −−→ 7 7
BC = rAC − rAB = − √3 î + (6 − √3 ĵ
−−→ TBC 7 √7 ĵ
Forces: TBC = TBC .nBC
ˆ = − → − √3 î + (6 −
|r−
BC | 3
−−→
TDE = −TDE ĵ (parallel to y axis)
→−
F = −(2KN )k̂ (vertically downward)

Take moment about A: MA = 0 ⇒ −


P
r− → −−→ −−→ −−→ −−→ →
AD × TDE + rAB × TBC + rAB × F = 0

TBC
⇒ √43 (î + ĵ + k̂) × (−TDE ĵ) + √73 (î + ĵ + k̂) × − 7 7 7
→ − √3 î + (6 − √3 )ĵ + √3 (î + ĵ + k̂) × −2k̂ = 0
|r−
BC |

⇒ (−4TDE k̂ + 4TDE î) + 7T BC √


( 73 k̂ − √73 ĵ + (6 − √73 )k̂ − (6 − √73 î)) + 14ĵ − 14î = 0
|−
r−→
BC |
√ √
⇒ (4TDE − 7T BC
(6 − 7/ 3) − 14)î + (− 7T BC
× (7/ 3) + 14)ĵ + (−4TDE + 7T BC
× 6)k̂ = 0 = 0î + 0ĵ + 0k̂
|−
r−

BC | | −
r−→
BC | |−
r−

BC |

From the component along y we obtain:√



− 7T
−BC

→ × (7/ 3) + 14 = 0 ⇒ TBC = 1449 3 × |r−−

BC | KN
|rBC |
Now :|−r−→ 7 2
BC | = ((6 − √ ) +3
49 0.5
) = 4.491
3

298
⇒ TBC = 2.222 KN

From the component along z axis we obtain:


TBC
−4TDE + 7 − → ×6=0
|r−
BC |
TBC

⇒ TDE = 212 × |− −
→ = 21/2 × 14 3/49 KN
√ r BC |
⇒ TDE = 3 3KN = 5.196 KN

Therefore the cable tensions are : TBC = 2.222KN


TDE = 5.196KN

(a). FBD of the beam AB:

e
ki
oo
pC
ar
Sh

299
ANS: (a) FBD of the beam AB:

Fy = 0 : Ay − P = 0 ⇒ Ay = P
P P
(b) Reaction Components at A: Fx = 0 : Ax = 0
MatA = 0 : MA + M − P.L = 0 ⇒ MA = P L − M
P

e
ki
oo
pC
ar
Sh

ANS:
(a) Zero force members:
1. CD and ED(from joint D)
2. EF and CE (from joint E)
3. BH (from joint H)
(b) Forces in members CF and BC:
FBD of joint C:

F = 0 : FBC cos45 − FCF cos45 = 0 ⇒ FBC = FCF


P
P x
Fy = 0 : P − FBC sin45 − FCF sin45 = 0
P − FBC × √12 = 0 ( using FCF = FBC , sin45 = √1 )
2

⇒ FBC = P/ 2

300

⇒ FCF = P/ 2 √
Therefore √
: FBC = P/ 2(compression)
FCF = P/ 2(tension)

ANS:
(a) Free body diagrams:

e
ki
oo
pC
ar

Member BD has forces acting along at its ends through pin joints. Therefore BD is a two force member.
Sh

301
(b) Forces at pins B and C: From the fbd of member DE: MD = 0 : FC × 4 − 6KN × 5 = 0
P

⇒ FC = 30 4 = 7.5KN ( along y direction)


From the fbd of member ABC √ √
MA = 0 : FB sin45 × 2 − FC × 4 = 0 ⇒ FB = 2 2FC = 15 2KN Component of FB along x: FBx =
P

FB cos45 = 15KN
along y :FBy = FB sin45 = 15KN
Fx = 0 : Ax − FB cos45 = 0
P

⇒ Ax = FB cos45 = 15KN e
ki
oo

B sin45 − Fc = 0
P
Fy = 0 : Ay + F√
pC

⇒ Ay = Fc − FB / 2 = −7.5KN
Therefore the forces at the pins A, B, C are
ar

Ax = 15KN Ay = −7.5KN √
Sh

FBx = 15KN FBy = 15KN or FB = 15 2KN (tension)


Fc = 7.5KN (downward and along y axis)
ANS:

302
Free body diagrams of the cylinders:

From the triangle joining centers of A, B and C:

e
ki
oo

P
From the FBD of cylinder A: Fx = 0 : N1 = N2
pC

Fy = 0 : (N1 + N2 )cosθ = W or 2N1 cosθ = W − − − − − −(1) Note : since N1 = N2 from fbd of half
P

cylinders B and C , we get F3 = F4 and N3 = N4


ar

From the FBD of cylinder-B:


Sh

Fy = 0 : N3 − W2 − N1 cosθ = 0
P

N3 = W (after using equation (1) )


Fx = 0 : F3 − N1 sinθ = 0
P

or F3 = N1 sinθ = W 2 tanθ ( using eq. (1) )


To maintain equilibrium - need to avoid slip of half cylinders B and C.
⇒ F3 ≤ µN3 = µW
⇒W 2 tanθ ≤ µW ⇒ tanθ ≤ 2µ
⇒ √ d2 2 ≤ 2µ = 1
(16r −d )

d ≤ 2 2r
Since , F3 = F4 and N3 = N4 , we obtain the relation
√ considering the fbd of cylinder C.
For equilibrium , minimum distance d is dmin = 2 2r

303
ANS:

e
ki
oo
pC
ar
Sh

Oy = T1 + T2 − − − − − − − − − −(2)
P
F = 0 : Ox = 0 Fy = 0
P x
Mc = 0 : (T1 − T2 ) × 0.01 − Oy × 0.03sinθ − Ox × 0.03cosθ = 0
⇒ (T1 − T2 ) − (T1 + T2 ) × 3sinθ = 0
T1 −T2
⇒ sinθ = 3(T 1 +T2 )
− − − − − − − − − − − −(3)
(a) Since belt moves from right to left : T1 > T2
⇒ T1 /T2 = eµβ ⇒ T2 = T1 e−0.5π
sinθ = 0.2186 ⇒ θ = 12.63 deg
(b)Oy = T1 + T2 = 10(1 + e−0.5π )N = 12.08N, Ox = 0N
⇒ Force magnitude at hinge O is 12.08N

304
MECHANICS
MID - AUTUMN SEMESTER EXAMINATION2016-2017

ANSWER:
F1 + F2 = F
⇒ F2 cosβ − F1 sin30 = 0
andF2 sinβ + F1 cos30 = 10
⇒ 5cosβ = F1 sin30 and 5sinβ = 10 − F1 cos30

Square and add to get β and then find F1


Therefore solving this we get β = 30 deg and F1 = 8.67KN
e
ki
oo
pC
ar
Sh

ANSWER:
For rod ABC For rod BC

At point D there is a roller, so horizontal force at point D ⇒ Dx = 0

305
Looking at the FBD of the whole body.

⇒ MA = 0 or F = Dy = 100 ⇒ Ay = 0[ Fy = 0] also Ax = 0(
P P P
Ax = 0)
Unstretched length of spring = 1m
Compressed length = 1 − 2 × 1sinα
Ax = Ay = 0[proved]
⇒ MB = 0 or F3 × 1cosα = F × 2cosα
P

or F3 = 2F or kx = 2F or x = 2×100
20 = 10mm
⇒ x = 1 − 2sinα
or 0.001 =1 − 2sinα or 2sinα = 0.99 or α = 29.67 deg

e
ki
oo
pC
ar
Sh

ANSWER: At point E ,Ex = 0


→ Ey (8) = (4KN )(1.6) + (3KN )(3.4) [ Taking FBD of the whole body] MA = 0
P

or Ey = 2.68KN

⇒ cosθ = AC/AG = 0.8 → sinθ = 0.6 From ∆AHO, α = 90 − θ


P
Fx = 0 :

306
or Ax + 4sinθ + 3sinθ = 0
or Ax = −4.2KN
Fy = 0 : Ay + Fy − 4cosθ − 3cosθ = 0 or Ay = 2.92KN
P

(b) At point F , there is no truss to balance forces ⊥ to EG.


→ DF is zero member truss , if FDF = 0
→ At point D , there is no member to balance sorces⊥ to CE.
→ DG is zero member truss
Similar logic for GC → GC,GD,FD.
[ 3KN and 4KN are both parallel]

e
ki
oo

(c) ⇒ FED + FEF cosθ = 0


pC
ar
Sh

and Ey + FEF sinθ = 0


solving FEF = −4.47KN = FGF and FED = 3.57KN = FCD ; FHB = −4KN
→ FHB = 4KN (C)
FGF = 4.47KN (C)
FCD = 3.57KN (T )

307
ANSWER:
tanα = 4/3
⇒ sinα = 4/5 = 0.8
cosα = 3/5 = 0.6
⇒ MA = 0 [ For whole body ]
P

T (0.5 + 1.5 + 3) = Fy (3)


or Fy = 450 × (5/3) = 750N
P
Also Fx = 0 [for whole body ]
⇒ Ax = 0
P
Fy = 0 [for whole body]
⇒ Ay + Fy = 430 or Ay = −300N

e
ki
oo
pC
ar
Sh

[ diagram in reference to the above answer]

M = 0, → Ay (1.5) + Cx (2) = Cy (1.3) → Cy = −150N ( on member AC)


P
P B
ME = 0 → 450(0.5) = Cx × 2 → Cx = 112.5
ThereforeCy on member CF = 150 N and Cx = 112.5N

308
e
ki
oo

ANSWER:
pC
ar
Sh

µA = 0.25
P
MC = 0
FA cos15×
2 + F sin15×
1 − 20cos45×
1 − 20sin45 × 1 − µFA cos15 × 1 + µA FA sin15 × 2 = 0
Solving we get FA = 13.607KN

309
Fx = 0, P − µA FA cos15 − FA sin15 − 0.15N = 0
P
on wedge :
or P = 0.15N − 0.236
Fy = 0.N − FA cos15 − 0.25FA sin15 = 0
P

or N = 14.024KN ⇒ P = 1.867KN

e
ki
oo
pC
ar
Sh

N = Wcos30 [ Fy = 0] − − − − − −(1)
P
P
P + T + f = Wsin30 [ Fx = 0]———(2)
P
MC = 0
or P(3a/2) + T(a/2) = W sinθ(a)———-(3)
Also by law of friction
P 0.6π µ = 0.6( At the spool)
T =e
or T = 0.152P ——–(4)
Solving we get P = 317.3 N
From (2) f = 134.5N ⇒ Minimum coefficient of friction = f/(Wcos30) = 0.155

310
MECHANICS
END - SPRING SEMESTER EXAMINATION2017-2018

F/2
1.a) Shear stress = A
F
= 2×(25×30)×10 −6
F
−6 = 800 × 10
3 e
ki
∴ 2×(25×30)×10
or F =1.2kN
oo
pC

b) δl = shearstress
G
or δ = 0.2mm
ar
Sh

2)

1 σx
rx = E (σx − γσy ) = E

311
0
DB = (1 + rx )DB

6 √
= (1 + 80×10
70×109 )10 2
= 14.15829806mm
ry = −γσ
E
x

0
∴ φnew = 2tan−1 ( DB
AD
0 = 2tan
−1 (14.1368202/14.15829806) ≈ 89.913◦

e
ki
oo
pC
ar

3)
Sh

Using Mohr’s circle


σx = 80M P a
σy = 60M P a
τxy = 40M P a
p
∴ Radis = (10)2 + (40)2 = 41.23M P a
Center = (70M P a, 0)
∴ (σN )max = (70 + 41.23)M P a = 111.23M P a
(σN )min = (70 − 41.23)M P a = 28.77M P a
σ +σ σ −σ
σx0 = x 2 y + x 2 y cos2θ + τxy sin2θ
σx −σy
τx0 y0 = −( 2 )sin2θ + τxy cos2θ

312
q
σx +σyσ −σ
(σx0 )max = 2 ( x 2 y )2 + τxy
+ 2
q
σx +σyσ −σ
(σx0 )min = − ( x 2 y )2 + τxy
2
2

∴ σx = q 80M P a, σσ y = 60M P a q
∴ 70 + 100 + τxy 2 ≤ 100 and 70 − 2 ≥S
100 + τxy
q q
or 100 + τxy 2 ≤ 30 or 65≥ 2
100 + τxy
2 ≤ 900 or 4225 ≥ 100 + τ 2
or 100 +τxy xy
or τxy ≤ 28.3M P a or τxy ≤ 64.23M P a
∴ τxy maximum is 28.3MPa

e
ki
oo
pC
ar
Sh

4) Z = 12.5mm
E = 200GP a
γ = 0.25σθθ = 2σxx = PZr
∴ σθθ = Ptr [ normal stress is maximum for σθθ among σθθ and σxx
a) To keep σθθ constant and r constant
P/t = constant
or t = 2.5×2
1.5 mm = 16.67mm
∴ Increase in thickness = 4.17mm
b) σθθ = Ptr
6 ×r
or 200 × 106 = 1.5×10
12.5
or d =2r =3.33m

Dd Dr 1 Pr
c) d = r =E ( σθθ -γ σxx ) = (1 − (γ/2) Et
Dd 2×106 ×(5/3)
or =
d 200×109 ×12.5×10−3
(1 − 0.33
2 ) [ Here P = 2MPa]
or Dd = 3.71mm

313
5.a) total sum must be zero
∴ 300 + 200 + Tc − 400 = 0
or Tc = 100N m
e
ki
oo
pC
ar
Sh

T
J = GφL = P
τ

τ = TJP
∴ τ is maximum at the position BC
Tmax = 500N m
(500)×r
∴ τ = (1/2)πr 4
or r = 15.8mm
∴ d = 31.69mm
TL
φ = GJ
∴ φmax is at BC Tmax = 500N m
500×0.5 1.5
∴ φmax = 80×10 9 × π ×r 4 = 180 × π
2
10×0.5×180 4
or 80×109 ×π×1.5×π = r ⇒ r ≈ 16.6mm and d = 33.21mm σθθ

314
6)

e
ki
oo
pC
ar
Sh

[ Be careful about the curavatre of the quadratic]


V + 2x = 1.75kN or V = (1.75 − 2x)
M + 2x( x2 ) = 1.75x
or M = 1.75x − x2

0
V + 2 = 1.75
0
or V = −0.25kN

315
e
ki
0
M + 2(x − (1/2)) = 1.75x
oo

0
⇒ M = 1.75x − 2x + 1
pC

= 1 − 0.25x
ar

00
V + 2 = 1.75
00
Sh

⇒ V = −0.25kN
00
M + 2(x − 12 ) + 12 = 1.75x
00
⇒ M = 1/2 − 0.25x

316
7. σ = MI y e
ki
At the two maxima , x =0.4 , x=0.6
oo

At x = 0.4
3 3
σc = 20×10Izz×0.125 = 2.5×10
pC

Izz
3 1.5×103
σT = 20×10Izz×0.075 = Izz
ar

At x = 0.6
3
σT = 15×10Izz×0.125
Sh

1.875×103
= Izz
3
σc = 15×10Izz×0.075
3
= 1.125×10
Izz
∴ Compressive stresse maximum at x = 0.4 with σc = (2500/Izz )Pa
Tensile stress is maximum at x =0.6
with σT = (1875/Izz )Pa
2 3
bh3
Izz = 0.15×(0.05)
12 + (0.05)(0.15)(0.125)2 + 0.05(0.12)
12 + (0.05)(0.15)(0.125)2 [i = 12 + A1 d2 ]
−4
1.75 × 10 m 4

317
MECHANICS
END SEMESTER AUTUMN EXAMINATION 2017-18

Ans 1a): Factor of safety = 2.5


Critical Area A = (40 - 10)×15 = 450mm2 e
ki
F
oo
Critical stress, τ = F/A = MPa, where F is in Newton.
450
F/2 2F F
pC

Ans 1b): Critical shear stress = 2


= 2
= MPa
πd /4 πd 50π
ar
Sh

Ans 1c): Considering fail due to tension


Fmax 400
∴ =
450 2.5
or Fmax = 72 kN
Considering fail due to stress,
Fmax 170
⇒ = or Fmax = 10.68 kN
500 2.5

318
T Gφ τ
Ans 2a): = = G=30 GPa r = 50mm
J L r
τL 50 × 106 × 3
∴φ= = = 0.1rad = 5.73o
Gr 30 × 109 × 50 × 10−3
Tr
Ans 2b): τmax = To get τmax and T constant
J
J 1
J = π r2 4 − r1 4

we got =constant
r 2
1 (0.054 − 0.044 ) 1 3
∴ π = πr
2 0.05 2 e
ki
or diameter = 83.9mm
oo
Ans 2c): Ratio of weights  4= ratio of areas
π(d42 − d41 ) d2 − d41
∴ ratio = = ' 0.51
pC

πd2 /4 d2
ar
Sh

Ans 3): The ’q’ force can be replaced by 1 single force of magnitude qL at the center.
q = 1 kN/m

∴ ΣMA = 0: 2×1 - 1 - MA = 0 MA = 1kNm


ΣF = 0: RA = 2kN

319
Sh
ar
pC
oo
ki
e

320
.

Ans 4a):

e
ki
Also A = AN / cos 30o
oo

P sin 60o P
∴ σN = = cos2 30o
AN sec 30o A
pC

3 P
∴ 50 × 106 = ×
4 100
ar

or P = 6.67 kN
P cos 60o P
Sh

Ans 4b): τ = = sin 60o


AN sec 30o 2AN  √
1 50 × 106 × 4 3
= × √ ×
2 3 3 2
= 28.87MPa

321
F 4l
Ans 5): =E×
A l
force = weight = 1000 N, E = 1 GPa, A = 100mm2 , l = 3m
1000 4l
or, −6
= 109 ×
100 × 10 3
or, 4l = 30 × 10−3 = 80mm

e
ki
oo

Ans 6a):
pC
ar
Sh

r
Given : = 100
t
Pr
Hoop stress: σn = = 100P
t
Pr
Longitudinal Stress: σa = = 50P
2t
Assuming state of plane stress:
σa νσn 1
Axial strain: εa = − = (50P − ν × 100P )
E E E
50P
⇒ εa = (1 − 2ν) = 150 × 10−6 (Given)
E
150 × 10 E−6 −6
150 × 10 × 70 × 109
⇒P = = Pa
50(1 − 2ν) 50 × (1 − 2 × 0.33)
P = 617.65 ×103 Pa
∴ pressure before can opening is = 617.65 kPa

322
Ans 6b): τxy = 0 θ = 30o
σx = σa = 50P = 30.88 MPa
σy = σn = 100P = 61.77 MPa
   
σx + σy σx − σy
σ x1 = + cos 2θ + τxy sin 2θ
2 2
= 46.33 - 15.45cos 60o = 38.61 MPa
   
σx + σy σx − σy
σ y1 = − cos 2θ − τxy sin 2θ = 54.04 MPa
2  2
σx − σy
τx1 y1 = − sin 2θ + τxy cos 2θ = 13.37 MPa
2
σx1 = 38.61MPa , σy1 = 54.04MPa , τx1 y1 = 13.37MPa

Ans 7):
e
ki
oo
pC
ar
Sh

E = 100GPa = 100×103 MPa ν = 0.3


σx = 10MPa , σy = 20MPa
1 1
Strains: εx = (σx − νσy ) = (10 − 0.3 × 20) = 4 × 10−5
E 100 × 103
1 1
εy = (σy − νσx ) = (20 − 0.3 × 10) = 17 × 10−5
E 100 × 103
ν
εz = − (σx + σy ) = −9 × 10−5
E
Elongation : δx = εx × L = 4 × 10−5 × 200mm = 8 × 10−3 mm
δy = εy × L = 17 × 10−5 × 200mm = 34 × 10−3 mm
δz = εz × t = −9 × 10−5 × 1mm = −9 × 10−5 mm

New Volume : (200 + δx )(200 + δy )(1 + δz )mm3

323
= (200.008)(200.034)(0.99991)mm3
V1 = 40004.8 mm3

Original volume : V = 200 × 200 × 1mm3 = 40000 mm3


⇒ Percentage change in volume :
4V 4.8
= × 100 = 1.2 × 10−2 %
V 40000

*****

e
ki
oo
pC
ar
Sh

324
MECHANICS
END SEMESTER SPRING EXAMINATION 2016-17
1. A vibration isolation unit consists of two blocks of rubber bonded to a rigid metal plate AB
and to rigid supports as shown in figure. The modulus of rigidity of the rubber is G=20 MPa,
Poisson’s ratio ν=0.5 and modulus of elasticity is E=60 MPa. If a force of magnitude P=40KN is
applied to the plate AB as shown, such that the bonding remains intact at all surfaces, then find
the
a) Deflection of plate AB in the downward direction.
b) Change in volume of the two rubber blocks.
c) Maximum tensile stress developed in the rubber blocks.

e
ki
Ans 1a): Force = P/2 = 20 KN
oo

G = 20 MPa
P 20 × 103
pC

∴τ = =
2π 150 × 100 × 10−6
τ 20 × 103 1 1
ar

=θ= −6
× 6
=
G 150 × 100 × 10 20 × 10 15
Sh

Ml Ml
θ= =
l 30mm
∴M l = 2mm
Ans 1b): V = xyz
MV Mx My Mz
Thus, = + +
V x y z
taking M x and M y=0
MV Mz Ml 1
= = =
V z l 15
Ans 1c)* There should not be any tensile stress. [Clarification Required]

2. The walls of a thin-walled cylindrical pressure vessel made of epoxy are strengthened with
glass fibers oriented at β = 45o to the axis of the vessel as shown in figure 2. The pressure vessel is
subjected to internal gague pressure of 1 MPa. The walls are 1 mm thick and the diameter D and
length L are 500mm and 1000mm respectively. If a factor of safety of 2.0 is to be used them what
must be the
a) Tensile strength of the glass fibers so that they do not rupture.
b) Shear strength of the interface between the glass fibers and the epoxy.
c) Change in the circumference of the pressure vessel if the material of the pressure vessel is
taken to be isotropic with elastic properties E=20GPa and ν = 0.28 .
Pr
Ans 2a): σθθ = 2r
z( L ) + 1
1 × (0.5/2)
= MPa
0.001 × (0.5/1 + 1)
325
0.5 × 0.5
= MPa
0.001(1.5)
500
= MPa
3
Pr 1 × 0.5/2
σxx = = = 125 MPa
2t 2 × 0.001
Using Mohr’s Circle, Tensile stress at 45o rotation

e
ki
oo

σ + σxx
θθ
σ= ≈ 145.8 MPa
pC

2
Using factor of safety 2, σ 0 = 292 MPa
ar

Ans 2b): Sheer strength = (σ − σxx ) Radius of Mohr’s Circle


= 145.83 − 125 = 20.83MPa
Sh

Ans 2c): C = 2πr


MC Mr σθθ − νσxx 166.67 − 0.28 × 125
or = = = = 6.5835 × 10−3
C r E 20 × 103
∴M C = 6.5835 × 10−3 × π × 0.5 ' 10.34mm

3. A solid and a hollow cylindrical shaft made of the same material have the same length and
volume but are subjected to torsion Ts and Th respectively. Derive the relation between the ratios
of applied torque (Ts /Th ) and n(as shown in figure 3) if the maximum shear stress in both the cases
is exactly the same.

τ T
Ans 3): =
r J
rT
or τ = Given τ is same
J
ro Ts r2 Th
∴ π 4 = π 4 4 4
2 ro 2 (r2 − n r2 )

326
Ts (ro /r2 )3
or =
Th (1 − n4 )
4. A straight beam 8m long with inverted T section is transversely loaded. The cross-section of
the beam along with its centroid is shown in the figure

a) I of the cross-section about the cetroidal axis.


b) Location of the section of the beam where the tensile and the compressive bending stresses
are maximum.
c) Ratio between the maximum tensile bending stress and the maximum compressive bending
stress within the beam.
d) Location of the section where average shearing stress (shear force divided by area) vanishes.
e) Points where the bending stress is zero.
bh3
Ans 4a): I =
12 e
ki
(0.4)(0.04)3 (0.04)(0.2)3
∴ I1 = I2 =
oo

12 12
= 2.133 × 10−6 = 2.67 × 10−5
pC

∴ Inet = I1 + 0.4 × 0.04 × (0.04)2 + I2 + 0.4 × 0.04 × (0.08)2


= 1.568 × 10−4
ar

My
Ans 4b): σ = peaks at x=1, x=4, x=7
Sh

I
∴ Compressive stress is maximum at x=1, x=7
As I is constant, compare the products of My.
∴ Tensile stress is maximum at x=4.
4c): σmax tensile = 200 × 180 σmax compressive = 100 × 180
Thus, ratio = 2
4d):Probably not in syllabus.
4e): x=2,6 as M=0

5. The compound beam ABCDE shown in the figure 5 consists of two beams AD and DE
joined by a hinged connection at D. The loads on the beam consists of a 4KN force and 4KN-m
moment at B and a 2KN force at the mid-point of DE. Reaction forces at supports A,B and E have
magnitudes and direction as shown in figure 5. Draw the shear force diagram for ABCDE (with
proper labeling). Calculate bending moment at C and D and the maximum bending moment.

327
e
ki
oo
pC
ar
Sh

∴ From M2 , MC (x = 4) = -2 KN
∴ From M3 , MD (x = 6) = 0 KN

328
Probably the maximum bending moment is 2KN-m.

Q 6. is probably not in syllabus.

e
ki
oo

*****
pC
ar
Sh

329
ELECTRICAL TECHNOLOGY
Formulae
Rab × Rca
1) 4 to Y conversion: Ra =
Rab + Rbc + Rca

Ra Rb + Rb Rc + Rc Ra
2) Y to 4 conversion: = Rab
Rc
3) Source conversion:

4) For Thevenin theorem e


ki
oo
pC
ar
Sh

Isc is the short circuit current when RL is shorted.


VT h is the voltage across RL terminals when RL is open.

5) Maximum power transfer theorem is when internal resistance


is equal to load resistance.
Erms π
6) Form factor = = √ ' 1.112
s Z E ang 2 2

1 Vo
Erms = Vo2 sin2 θ.dθ = √
2π 0 2
1 π
Z
2Vo
Eavg = Vo sin θ.dθ =
π 0 π
Epeak Erms √
7)Peak factor = 2 = 1.414
=
8) For composite
p excitations; E = E1 sin(ω1 t) + E2 sin(ω2 t)
2
Erms = E1rms 2
+ E2rms

9) If XL + XC > 0 then its an inductive circuit and if XL + XC < 0 then its a capacitive circuit.
10) Maximum power transfer, when

ZL = ZT∗ h
Special cases: (i) For XT h = 0, RL = RT h , |ZL | = RT h
(ii)For XL constant (even 0) RL = [RT2 h + (XT h + XL )2 ]1/2
(iii) RL constant (explicitly), then, XL = −XT h
(VT h )2 RL
P=
2 (RL + RT h ) + (XL + XT h )2
2

11) Half powerp bandwidth, P = Po /2


∴ ω1 = −αp+ α2 + ωo2
ω2 = α + α2 + ωo2
R 1
where, α = , ωo = √
2L LC
R
∴ 4ω = ω1 − ω2 = 2α =
L
ωo 1 L ωo L
Quality factor (for series circuits) = =√ × = = ωτ
4ω LC R R
ωo ωo
Quality factor (for parallel circuits) = = =
4ω 1/RC
e
ki
omegao RC = ωo τ
oo

12)
pC
ar
Sh
13)

 2 1/2
1 R1 C − L
ωo = √
LC R22 C − L
See R1 is with L and R2 is with C.

14) Power = VL IL cos φ = 3Vph Iph cos φ where cos φ is power factor of load (balanced)

15 For balanced cases Z4 = 3ZY

16)

e
ki
oo
pC
ar


Sh

!
3(ω1 − ω2 )
φ(power factor angle)= tan−1
ω1 + ω2
If ω1 > ω2 inductive load.
If ω1 < ω2 capacitive load.

17) Hl = NI in AT(ampere turns)


B(flux density) = µH φ = BA −→flux
HR = l/µA ←− Magnetic reluctance(AT/Wb)
Hdl = N I

B2
18 Magnetic pressure = Energy per unit volume =
2µo
N 2 µ × Area
19) L = N = no. of turns
length
N2
L= R = magnetic reluctance
R√
M = ( L1 L2 k ←− coupling coefficient (k=1 means, no leakage condition)
1 2
20) L = Nφ/I Energy = LI
2
21)
Leq = L1 + L2 + 2M for,

Leq = L1 + L2 − 2M for,

1
Energy = Leq I 2
2
22)

e
ki
oo
pC
ar

Z
Sh

Energy stored = H.dB

23) Hysterisis loss = Rn Bmn fV

where, R − n = characteristic constant of core


Bm = Bmax n = steinmetz exponent
f = frequency
V = volume

21) Eddy current loss Pe = Ke f 2 Bm


2 V where K = kt2
e
t = thickness, V = volume, f = frequency
Bm = Bmax

22) Induced voltage(rms value), e1 = 2πf N1 φmax = 4.44f N1 φmax

E1 E2 M
23) = = a = turn ratio of transformer
N1 N2 N2
V2(no−load) − V2(f ull−load)
24) % voltage regulation = × 100%
V2(no−load)
Taking approximations ,
Voltage regulation = I(R cos θ + X sin θ) for θ lagging
=I(R cos θ − X sin θ) for θ leading
25)

xS cos θ
26) Efficiency(n) = all quantities are in maximum value
xS cos θ + Pi + x2 Pc
maximum
p efficiency when Iron loss = Copper loss
∴ x = Pi /Pc
1 e
ki
27) (Va )Auto = (VA )T w
1 − 1/a0
oo

V1
where a’ = auto transformer ratio =
V2
pC

θelectrical P
28) = where P = no. of poles
ar

θmechanical 2
3I 2 r2
Sh

29) Air gap power = 2 = PG


S
Rotor copper loss = SPG
Mechanical load (including mechanical loss) = (1 − S)PG
1
Mechanical load r2 (−1 + )
S
Pm (1 − S)PG PG
30) Torque, T = = =
ωr (1 − S)ωs ωs
3 VT h 2 (r20 /s)
31) T = ×
ωs r0
(rT h + 2 )2 + (XT h + X20 )2
s
r20
sTmax = q
RT2 h + (XT h + X20 )2
3VT2h 1
Tmax = ×
RT h + RT h + (XT h + X20 )2
p
2ωs
2Temax
32) Tc = For neglecting stator impedence.
S Smax
+
Smax s
33)

Peak remains constant with changing r2 . Increasing r2 shifts the peak towards left.

120 × f
34) ns (no. of rotations /minute) = where P=pole and slip rpm = sns
P
Is (line) − 4
35) =3
Is (line) − Y
36) q(t) = qα + (qo − qα )e−t/τ τ = RC
i(t) = iα + (io − iα )e−t/τ τ = L/R

37) Induced emf in stator coil = sE2

38) 200/400 V, 4000 VA −→ 400V at the supply of higher side referenced equivalent circuit.
Full load higher side referenced equivalent circuit = 10A

39) 400V, 100Hp −→ 400 volt line voltage supply ; 100 Hp is the output power.
40)

Vm
I= sin(ωt + θ − φ) + ke−R/L
|Z|  
Vm −1 ωL
∴k= sin(θ − φ) whereθ = tan
|Z|   R
ωL
∴ For transient free switching, φ = tan−1
R
When you replace L with
 C, then φ = −θ
1
∴ φ = − tan−1 for Capacitors.
ωRC

You might also like